Download as pdf or txt
Download as pdf or txt
You are on page 1of 90

The MOLECULES

of LIFE
Physical and Chemical Principles

Solutions Manual

Prepared by James Fraser and Samuel Leachman


Chapter 1
From Genes to RNA
and Proteins

Problems and Solutions

True/False and Multiple Choice

1. When two atoms approach each other closely, the 8. The central dogma of molecular biology states that
energy goes up because the nuclei of the atoms RNA is translated from proteins.
repel each other. True/False
True/False
9. Which of these types of molecules serve as a
2. Ionic interactions are stronger in water than in template for messenger RNA?
vacuum because water forms strong hydrogen a. Protein
bonds with polar molecules. b. DNA
True/False c. Transfer RNA
3. An N–H•••O=C hydrogen bond has optimal energy d. Carbohydrates
when it: e. Ribosomes
a. is bent. 10. DNA primase synthesizes DNA molecules.
b. is linear. True/False
c. has a donor–acceptor distance of 4 Å.
d. has a donor–acceptor distance of 2 Å. Fill in the Blank

4. A by-product of forming a peptide bond from two 11. When two atoms approach closer than
amino acids is water. ________________ the interaction energy goes up
True/False very sharply.
Answer: their van der Waals radii
5. Circle all of the polar amino acids in the list below:
a. Phenylalanine 12. The van der Waals attraction arises due to _______
induced dipoles in atoms.
b. Valine
Answer: mutually
c. Arginine
d. Proline 13. At room temperature, the value of __________ is
e. Leucine about 2.5 kJ•mol−1.
Answer: thermal energy
6. Proteins fold with their hydrophobic amino acids on
the surface and their hydrophilic amino acids in the 14. ________ is an operational nucleic acid, whereas
core. _________ is strictly an informational nucleic acid.
True/False Answer: RNA, DNA

7. Which of the following is not a unit of structure 15. Amino acids are zwitterions: molecules with charged
found in proteins? groups but an overall ______ electrical charge.
a. β sheets Answer: neutral
b. Loop regions 16. The ______ consists of two subunits that assemble
c. α helices around messenger RNA.
d. γ arches Answer: ribosome
2 Chapter 1: From Genes to RNA and Proteins

Quantitative/Essay b. Only iv could be broken readily by thermal


fluctuation.
17. Order the following elements from lowest c. i. Same.
electronegativity to highest: P, C, N, H, O, S. Use your ii. Slightly stronger; the Van der Waals
ordering of the atoms to rank the following hydrogen
interactions do not change, but the net
bonds from weakest to strongest:
interaction becomes stronger because of the
a. SH----O hydrophobic effect.
b. NH----O iii. Weaker.
c. OH----O iv. Weaker.
Answer: H = P < C = S < N < O; SH----O < d. ii, iii, and iv.
NH----O < OH----O
19. The diagram below shows a representation of the
18. The stabilization energy of a bond or interatomic structure of a peptide segment (that is, a short
interaction is the change in energy upon breakage portion of a larger protein chain). Hydrogen atoms
of a bond between two atoms (that is, the change are not shown. Nitrogen and sulfur atoms are
in energy when the atoms are moved away from indicated by “N” and “S.” Sidechain oxygen atoms
each other). We can classify bonds into the following are indicated by “*.”
categories, based on their dissociation energies: a. Identify each of the amino acid residues in the
Strong: > 200 kJ•mol−1 peptides.
Medium: 20–200 kJ•mol−1 b. Draw a linear chemical structure showing the
covalent bonding, including the hydrogen atoms, of
Weak: 5–20 kJ•mol−1
the entire
Question 1.19peptide.
Very weak: 0–5 kJ•mol−1
N
Consider the bonds highlighted in purple in the
diagram below. S
N
N
a. First consider the bonds in molecules isolated
from all other molecules (in a vacuum). Classify each N
N
of them into the four categories given above, based *
N

on your estimation of the bond strength. N N *


N *
b. Which of these bonds could be broken readily by N

thermal fluctuations? N

c. Next, consider what happens when these N

molecules are immersed in water (fully solvated). N

For each bond, indicate whether it becomes weaker,


stronger, or stays the same in water.


d. Which of these bonds could be broken readily by Answer:
thermal
Questionfluctuations
1.18 in water?
a. Ala-Gly-Ser-Gly-Ser-Tyr-Gly-Arg-Val-Met.
(i) (ii) b. See Figure at the top of the next page.
P O
20. With 64 possible codons and 21 options to code (20
amino acids + stop), (a) what is the average number
of codons per amino acid/stop codon? (b) Which
amino acids occur more often than expected in the
(iii) (iv) O actual codon table?
C OH
O –
C C Answer:
O C a. Average = 64 codons/21 amino acids =
H 3.05 codons per amino acid.
+
N
b. Leucine (6 codons), valine (4 codons), serine
HH (6 codons), proline (4 codons), threonine
Answer: (4 codons), alanine (4 codons), arginine
(6 codons), glycine (4 codons).
a. i. Strong.
ii. Very weak. 21. There are approximately 3 billion nucleotides in
iii. Weak. the human genome. If all of the DNA in the entire
iv. Weak. genome were laid out in a single straight line, how
PROBLEMS and solutions 3
1Q19b

NH
+
C NH

NH CH3
OH
CH2 H3C CH3 S

OH OH CH2 CH CH2

H CH3 H H H CH2 H H H CH2 H H H H H CH2 H H H CH2

C N C C N C C N C C N C C N C C N C C N C C N C C N C C N C C

O H O H O H O H O H O H O H O H O H O H O

A G S G S Y G R V M

long would the line be? Express your answer in 24. What changes to the central dogma were necessary
meters. after the discovery of retroviruses?

Answer: Answer:
(3 × 109 bp) × (3.4 × 10–10 m•bp–1) = 1.02 m The unidirectional flow from DNA to RNA was no
longer true after the discovery of retroviruses.
22. The human immunodeficiency virus (HIV) is a Retroviruses use reverse transcription to create a
retrovirus with an RNA genome. DNA copy and then generate many RNA copies.
a. Assume that each HIV contains two RNA
genomes and 50 molecules of the reverse 25. What chemical properties have led to DNA being
transcriptase enzyme. selected through evolution as the information
b. Assume that each reverse transcriptase molecule for complex life forms instead of RNA?
molecule acts on each RNA genome 10 times to Answer:
produce DNA.
DNA is inherently more stable. The 2ʹ-OH group in an
c. Assume that an integrase enzyme successfully
RNA nucleotide, which DNA lacks, can react to break
integrates 1% of the available reverse transcribed
the backbone just downstream by forming a cyclic
HIV genomes into the genome of a human host cell.
2ʹ-3ʹ phosphodiester bond and breaking the sugar–
d. Assume that each integrated copy of the viral phosphate backbone. Furthermore, because DNA
genome is transcribed 500 times/day. typically exists as a double-stranded helix, in the
event of DNA on one strand being lost or damaged it
How many HIV RNA genomes are created per day
can be replaced or repaired using the other strand as
from one infected cell?
a template. RNA, typically single-stranded, does not
Answer: have this capability.
50 reverse transcriptases × 10 replications ×
26. How do size considerations forbid G-A base pairs in a
2 genomes = 1000 DNA genomes created.
double helix?
1000 genomes created × 0.01 successfully integrated
by integrase = 10 integrated genomes. Answer:
10 copies of the genome integrated × 500 Both G and A are purines and have two rings. The
transcriptions per day = 5000 HIV genomes created double helix only has enough space for a single
from one infection per day. purine (two rings) and pyrimidine (one ring), but not
enough for four rings total.
23. What is a step in RNA processing that occurs in
eukaryotes but not in prokaryotes? 27. Why are DNA chains synthesized only in the 5ʹ-to-3ʹ
direction in the cell?
Answer:
Eukaryotes have introns, which necessitate splicing. Answer:
Additionally, eukaryotes add a poly-A tail on the 3ʹ Replication proceeds only in the 5ʹ → 3ʹ direction,
end and a methyl-G cap on the 5ʹ end. and not 3ʹ → 5ʹ, because only nucleotide
4 Chapter 1: From Genes to RNA and Proteins

triphosphates with the triphosphate group attached 29. Suppose you were told that the two strands of
to the 5ʹ carbon of the sugar are present in cells. This DNA could in fact be readily replicated in opposite
is the high-energy, reactive end of the molecule. directions (that is, one in the 5ʹ → 3ʹ direction, and
one in the 3ʹ → 5ʹ direction). Would you have to
28. Chemists are able to synthesize modified postulate the existence of a new kind of nucleotide?
oligonucleotides (that is, polymers of nucleotides) in If so, what would the chemical structure of this
Question
which1.29
the phosphate linkage is replaced by a neutral compound be?
amide linkage, as shown in the diagram below.
Answer:
phosphodiester DNA amide DNA You could postulate a nucleotide triphosphate with
the triphosphate attached to the 3ʹ sugar carbon,
O O rather than the 5ʹ. The high energy 3ʹ triphosphate
O base O base
would be susceptible to attack by the 5ʹ-OH group in
the growing strand, allowing the chain to grow in the
3ʹ → 5ʹ direction.
O H H
O– O
P
O
O N
O base H O base

O H O H

(Adapted from M. Nina et al., and S. Wendeborn,


J. Am. Chem. Soc. 127: 6027–6038, 2005. With
permission from the American Chemical Society.)
a. Such modified oligonucleotides are able to form
double-helical structures similar to those seen for
DNA and RNA. Often these double helices are more
stable than the natural DNA and RNA double helices
with the same sequence of bases. Explain why such
helices can form, and why they can be more stable.
b. Given the increased stability of such modified
nucleotides, why has nature not used them to build
the genetic material? Provide two different reasons
that could explain why these molecules are not used.

Answer:
a. The amide DNA helices lack the unfavorable
negative charge concentration present in
phosphodiester DNA.
b. If the amide DNA double helices were too
stable, it wouldn’t be possible to unwind them
for replication and transcription. Also, without a
high-energy phosphate linkage, the amide DNA
monomers can’t internally store the energy needed
for synthesis.
The MOLECULES
of LIFE
Physical and Chemical Principles

Solutions Manual

Prepared by James Fraser and Samuel Leachman


Chapter 2

Nucleic Acid Structure

Problems and Solutions

True/False and Multiple Choice Fill in the Blank

1. The tertiary structure of functional RNA molecules is 7. The ______________ structure of a nucleic acid is the
easily predicted. sequence of nucleotides in the DNA or RNA molecule.
True/False Answer: primary

2. Which of the following is not a stabilizing force for the 8. The modified RNA base in which two methyl groups are
structure and stability of double-stranded nucleic acids? added to guanine is __________.
a. base stacking Answer: N,N-dimethylguanine. The corresponding
b. hydrogen bonding nucleoside is N,N-dimethylguanosine.
c. disulfide bonds
9. B-form DNA has a C2ʹ ________ sugar pucker.
d. electrostatic forces
Answer: endo
3. H-bond acceptor, H-bond donor, H-bond acceptor,
methyl group is the pattern of potential interactions at 10. Hoogsteen base pairs, where the hydrogen-bonding
the edge of which Watson-Crick base pair? interactions utilize the Watson-Crick base-pairing edge
on one base and the major groove edge in the other
a. A-T base, can be utilized to form an RNA _______ helix.
b. G-C
c. A-A Answer: triple
d. C-G
11. Metal ions, such as K+, Na+, and Mg2+, typically interact
e. T-A
with the __________ group on the backbone of nucleic
acids.
4. Genomic DNA can become deformed from its normal
B-form by DNA binding proteins, such as the histone Answer: phosphate
proteins and the TATA-box binding protein.
True/False Quantitative/Essay
5. Which nonstandard base pair is most likely to form a 12. What physical factors force RNA to adopt only the C3ʹ
wobble base pair? endo sugar configuration, but allow DNA to adopt either
a. U-U the C2ʹ endo or the C3ʹ endo sugar configurations?
b. A-G
c. A-A Answer:
d. G-U The C2ʹ endo configuration results in close contact
e. G-G (1.9 Å) between one of the oxygen atoms of the
3ʹ phosphate group and the hydrogen atom at the
6. Classify the following RNA structural elements as 2ʹ position of the ribose ring in DNA. Compared with
secondary or tertiary structure: DNA, RNA has the 2ʹ hydrogen replaced by hydroxyl
a. coaxial helices group (OH) and the two oxygen atoms (2ʹ ribose and
b. pseudoknot 3ʹ phosphate) repel each other strongly. Rather than
c. hairpin distorting the structure away from the Watson-Crick
d. junction model, it is energetically favorable for RNA to switch to
e. adenosine platform the C3ʹ endo conformation.
f. ribose zipper
g. bulge
Answer: Secondary: c, d, g; tertiary a, b, e, f
2 Chapter 2: Nucleic Acid Structure

2Q14
Problem
13. WhatSet
type(quantitative question
of RNA interaction 2) (na_57_v1)
is shown below? Answer:

H N G
minor groove
N
O N
H N
N

H N H N H
O

major groove
H U O

Label the major and minor grooves of the Watson-Crick Interactions on minor groove: H-bond acceptor, H-bond
base pair and the potential hydrogen bonds between all donor, H-bond acceptor.
bases. Interactions on major groove: hydrogen atom, H-bond
acceptor, H-bond acceptor, H-bond acceptor.
Answer:
The G-U base pair is not a standard Watson-Crick base
This shows an A-minor motif in which the minor groove
pair.
edges of an adenine base interact with the minor
groove
2Q13 of a G-C base pair. 15. List at least three physical features that distinguish
A- and Z-form DNA.
major groove
Answer:
The main differences between A- and Z-form DNA are
as follows:
For A-form DNA, the repeating unit is one base;
however, in Z-form DNA, the repeating unit is two bases
(most often alternating pyrimidine and purines).
minor groove A-form DNA is a right-handed double helix. Z-form DNA
is a left-handed double helix.
In A-form DNA, the sugar pucker of the bases is
C3ʹ endo, whereas Z-form DNA alternates between
C2ʹ endo and C3ʹ endo.
The minor groove of A-form DNA is wide and shallow,
whereas the Z-form minor grove is narrow. The major
14. Label the atoms, the bases, the hydrogen bonds groove of A-form DNA is deep and narrow, whereas the
across bases, the major and minor grooves, and the Z-form major groove is relatively shallow.
interactions along the grooves for the following base
pair (oxygen and nitrogen atoms are not identified In A-form DNA the base pairs are tilted to the helical
explicitly): axis, whereas the Z-form base pairs are nearly
Problem set (quantitative question 2) (na_56_v1)
perpendicular to the helical axis.

16. In water, why is base stacking relatively more important


than hydrogen bonding to forming a DNA double helix?

Answer:
Water can make H bonds with the DNA base H-bond
donor and acceptors, so the energetic gain in
interactions when those H bonds are satisfied by other
nucleotides is not great. However, the many small polar
interactions along the nucleotide rings that are formed
upon base stacking are energetically more favorable
than the interactions with water.

17. How are the interactions between a nucleic acid and


Is the base pair a standard Watson-Crick base pair? Mg2+ mediated by water?
PROBLEMS and solutions 3

Answer: 22. The structure of the large ribosomal subunit from


Water molecules can form a hydration shell around Haloarcula marismortui (PDB code: 1FFK) has been
Mg2+, facilitating indirect interactions between it and solved by x-ray crystallography. The 23S RNA contains
the phosphate and functional groups in nucleic acids. 2922 nucleotides (758 A, 889 G, 739 C, and 536 U).
a. Assuming a random distribution of nucleotides,
18. Why is the R (purine) in GNRA tetraloops required how many four-mer sequences with the sequence G-N
rather than a pyrimidine? (any base)-R (purine)-A are possible?
Answer: b. There are actually 21 GNRA tetraloops in
The R position, adjacent to the A in sequence and the structure. What percentage of possible GNRA
space, is functionally significant because it provides sequences actually formed tetraloops in the structure?
a large surface area (the base has the two rings of a Answer:
purine, as opposed to one ring for a pyrimidine) against
a. Probability of G = total G/total nucleotides =
which the A can base stack.
889/2922
19. Why do most transcription factors interact with the Probability of N = 1.
major groove of B-form DNA rather than the minor Probability of R = (total A + total G)/total nucleotides =
groove? (758 + 889)/2922.
Answer: Probability of A = total A/total nucleotides = 758/2922.
The major groove has more distinguishing interactions Each four-mer has a GRNA probability = (probability of
than the minor groove. Each base pair has a unique G) × (probability of N) × (probability of R) × (probability
pattern of molecular interactions along the major, of A) = 0.0445.
but not the minor, groove. Since transcription factors Total four-mers = 2922 – 3 = 2919.
need to bind specifically to sequences of DNA, the
Probable GRNA four-mers = probability × total four-
best potential for distinguishing different sequences is
mers = 0.0445 × 2919 = 130.
to interact with the major groove. Further, the minor
groove is narrow compared with the major groove. b. If only 21 GNRA tetraloops formed out of a
The depth of the major groove is compatible with the possible 130, then16% of the sequences form a stable
interaction with protein structural elements such as an tertiary GNRA tetraloop motif.
α helix.
23. A bacterial DNA polymerase moves at approximately
20. Which of the following DNA sequences is most likely to 1000 base pairs per second when replicating DNA. The
adopt the Z-form? Why? polymerase holoenzyme is approximately
110 Å long. By what multiple of its length does the
a. GCGCGCGCATATGCGCGCGCC
polymerase move forward along the axis of the DNA
b. AGAGAGCTCTCTCTCTAAAAT
double helix in 3 seconds? Assume the DNA stays fixed
Answer: and ignore the rotational component of the motion of
Sequence a is more likely to adopt Z-form because the polymerase along the DNA.
it alternates purine and pyrimidines in a GC-rich Answer:
sequence. This alternating pattern of 2ʹ endo and 3ʹ
Each bp of DNA ~3.4 Å rise per bp.
endo puckers yields the zig-zag pattern, where the
smallest repeating unit is two base pairs, characteristic In 3 s, the polymerase moves 3000 bp.
of Z-form DNA. Length = 3000 bp × 3.4 Å•bp–1 = 10,200 Å.
Length/polymerase length = 10,200 Å/110 Å = 92.7
21. Consider a relaxed, closed-circular DNA plasmid that
times the total length of the polymerase in 3 sec.
has 1040 base pairs with writhe = 0. An intercalator is
added, such that there is one intercalator per 104 base
pairs. The effect of the intercalator is to cause the twist
between the base pairs that flank it to be reduced
to zero. Will the resulting intercalator-bound DNA be
positively or negatively supercoiled?
Answer:
Positively supercoiled. The relaxed plasmid contains
100 turns (1040 bp × 1 turn/10.4 bp) and therefore
has linking number L = 100. Because it is not initially
supercoiled, it has writhe W = 0. By the relation
W = L – T, initial twist T is 100, with each base pair
contributing ~35°. The cumulative effect of binding
10 intercalators is a reduction of the twist by a full turn
(~35° × 10 ≈ 360°) to T = 99. As L is constant, positive
supercoiling with W = 1 results.
The MOLECULES
of LIFE
Physical and Chemical Principles

Solutions Manual

Prepared by James Fraser and Samuel Leachman


Chapter 3

Glycans and Lipids

Problems and Solutions

True/False and Multiple Choice

1. In most carbohydrate monomers, the ratio of 6. An individual lectin-binding domain is likely to bind with
carbon:hydrogen:oxygen is high affinity to many diverse sugars.
a. 1:1:1 True/False
b. 2:1:1
7. In plants, the cell wall is generally higher in percent
c. 1:2:1
polysaccharide content and lower in percent lipid
d. 1:1:2
content than the cell membrane.
e. 1:1.5:1
True/False
2. Which of the following is not a lipid?
a. cholesterol
Fill in the Blank
b. sphingolipid
c. apolipoprotein 8. Simple sugars can exist in both linear and ______ forms.
d. vitamin A
Answer: cyclic
e. triglyceride
9. ________ is an acetylated polymer found in the
3. Many sugars are structural isomers of other sugars.
exoskeletons of insects and the cell walls of fungi.
True/False
Answer: Chitin
4. Most detergents will organize spontaneously into
10. Inositol is an unusual six-carbon sugar because it does
bilayers when placed in aqueous solutions.
not contain an _______ atom in its ring.
True/False
Answer: oxygen
5. Which of the following fatty acids are unsaturated?
Problems Q5 (lipids_probQ5_v1) 11. A lipid _____ is a section of membrane with both
composition and physical properties that are distinct
(A) (B) (C) (D) (E) (F) from the surrounding membrane.
O O O O O O
Answer: raft
O O O O O O
12. In eukaryotic cells, intracellular compartments bounded
by membranes are called ________.
Answer: organelles

Quantitative/Essay

13. Why are the most stable conformations of sugars


most often nonplanar with hydrogens in the equatorial
positions?
Answer:
Carbons which make four single bonds should have
bond angles near the value in a tetrahedron, 109°.
To maintain bond angles near this value the atoms in
closed ring structures cannot all lie in a plane. Instead
Answer: A, B, and E the rings “pucker,” moving one or more atoms out of
2 Chapter 3: Glycans and Lipids

the plane. There is less space for nonhydrogen atoms 18. What molecular forces drive the formation of bilayers
in the axial positions, so the conformational energy is when phospholipids are immersed in water?
lower when most of the hydroxyls are in equatorial
Answer:
positions.
The charged phospho- head groups point out to form a
14. What information can be gained more readily from favorably charged interface with water. The hydrophobic
the Fischer representation than from the Haworth alkyl chains point in and interact with each other
representation? laterally, packing against the hydrophobic chains of the
other layer. These hydrophobic forces exclude water
Answer:
from the interior region. This structure satisfies the
The Fischer projection is very convenient for comparing
interaction preferences for both segments (charged
stereochemistry of different sugar isomers, but poor
head group and uncharged tail) of the lipid molecules.
for seeing the three-dimensional relationship of atoms
and the real geometry of the molecules. The format 19. How does the packing of cholesterol against other lipids
indicating three-dimensional structure provides a better alter the biophysical properties of a membrane?
sense of conformational features, but is less easy for
Answer:
seeing relationships between stereoisomers.
The rigid core of cholesterol packs against alkyl chains
15. How do the homopolymers cellulose and glycogen differ of neighboring lipid molecules, reducing their flexibility.
at the level of: (a) the stereochemistry of the monomer, The net effect is to stiffen the membrane bilayer
(b) physical properties, and (c) the intramolecular against distortion, while retaining fluidity. A membrane
interactions that maintain those physical properties? containing cholesterol is less permeable to small
molecules, but does not substantially reduce the rate of
Answer:
diffusion of molecules within the membrane.
Glycogen contains only α-glucose, whereas cellulose
contains only β-glucose. 20. Many proteins need to be in close proximity to the
Glycogen branches off spiral structures through 1→6 membrane in order to perform their biological functions.
linkages that result in a soft, gel-like substance. The What are two strategies employed by the cell to ensure
gradual helical structure of glycogen is not stabilized by that certain proteins are anchored to the membrane?
any particular intramolecular interaction. Answer:
Cellulose consists of linear chains, assembled into Proteins can be covalently modified at a cysteine
stacked sheets that form stiff fibers. These sheets are residue or at the C-terminus with a lipid-containing
stabilized by intramolecular hydrogen bonds between molecular anchor such as a polyisoprene. These
neighboring residues. modified proteins then associate with the membrane
16. A student performs a biochemical fractionation of a via the insertion of the lipid anchor into the membrane.
cell and isolates the membrane-containing fraction. An Alternately, proteins containing specific lipid binding
analysis of the glycoproteins in the fraction reveals only domains can interact with the cytosolic portion of lipids
N-linked sugars. that are inserted into the membrane. These interactions
a. What type of amino acid residue are the sugars are generally noncovalent.
linked to? A third mechanism is for the protein to have a stretch of
b. What is the enzyme responsible for transferring the hydrophobic amino acids that insert into the membrane
first sugar to that residue? ensuring that the soluble domains of the protein are
c. What organelle has the student likely isolated? localized near the membrane.
Answer: 21. Without the aid of an enzyme, is a carbohydrate-
a. N-linked glycosylation links sugars to Asn residues. modified lipid more likely to (a) move in an undirected
b. The enzyme oligosaccharyl transferase transfers a lateral motion for 500 Å or (b) transfer across the ~40-Å
14 residue sugar to the Asn from a lipid anchor. thick cell membrane from one leaflet to the other?
c. The student has most likely isolated the Answer:
endoplasmic reticulum. (a) is more likely. Without a flippase enzyme, transfer
of a lipid molecule from one leaflet to another requires
17. The O blood type group is known as the universal
moving the head group (in this case, partially polar
donor. What property of the glycosylation pattern of
carbohydrate) across the hydrophobic region, a process
O-type individuals allows for the blood to be transfused
into individuals of any blood type without eliciting an that is energetically unfavorable and very slow. In
immune response? contrast, diffusion laterally within the membrane occurs
quite easily.
Answer:
The other three blood types (A, B, and AB) all contain 22. Scientists* have proposed a model for how yeast
the same tetrasaccharide that defines the O blood type. cells determine the length of a class of fatty acids that
Since the immune system of an individual with any are synthesized using a family of fatty acid synthase
blood type must be tolerant of the tetrasaccharide in enzymes. In different variants of the enzyme, a key
the context of “self” blood cells, no immune reaction lysine residue can be positioned at various places along
is generated by the tetrasaccharide of donated O-type an α helix of the synthase to control the length of the
blood cells. fatty acid chain that is produced.
PROBLEMS and solutions 3

Imagine a similar enzyme in which moving the position


of a catalytic lysine residue along a β sheet creates a
synthase that catalyzes four additional carbons into the
fatty acid chain. The lysine residue can only be moved
in two-residue units along the β sheet and still face
the substrate (that is, it can be moved from position
1 to positions 3 or 7, but never to positions 2 or 4).
Thus, movement from position 1 to position 3 results
in a four-carbon change in the length of the fatty acid
produced.

There are 14 residues in the β sheet. The maximum


length, a 40-carbon fatty acid, is created when the
residue is placed at the end of the β sheet (position
1). What is the shortest possible fatty acid that the
synthase can create?

*[Denic V & Weissman JS (2007) A molecular caliper


mechanism for determining very long-chain fatty acid
length. Cell 130, 663–677.]

Answer:
The catalytic residue can be moved to position 13—this
is 6 residue units.
40 carbon fatty acid – (6 residue units × 4 carbons per
residue unit) = 40 carbons – 24 carbons = 16 carbons.
The smallest fatty acid chain that can be created by this
enzyme is 16 carbons long.
The MOLECULES
of LIFE
Physical and Chemical Principles

Solutions Manual

Prepared by James Fraser and Samuel Leachman


Chapter 4

Protein Structure

Problems and Solutions

True/False and Multiple Choice

1. Globular proteins are generally embedded in the interior 7. The least restricted ϕ and ψ angles are found in
of a lipid bilayer. polypeptides in which class of secondary structure?
True/False a. right-handed α helix
b. β sheet
2. The secondary structure of a protein refers to the
c. left-handed α helix
extent and order of its α helices and β sheets.
d. loop
True/False
8. The active site in open twisted α/β domains is in a
3. Which of the following statements regarding protein crevice outside the carboxyl ends of the β strands.
domains is NOT true?
True/False
a. Secondary structural elements of a domain
generally pack so that a hydrophobic core is formed.
Fill in the Blank
b. A protein domain normally contains 50–200
residues. 9. Soluble proteins have mostly _________ sidechains
c. Protein domains are units of tertiary protein on the inside and mostly ________ sidechains on the
structure. outside.
d. Proteins are only comprised of one protein Answer: hydrophobic, hydrophilic
domain.
10. The organization of the protein subunits in multimeric
4. Match the following proteins with their quaternary proteins is known as the ___________ structure.
structure: Answer: quaternary
a. hemoglobin i. one subunit 11. SH2 domains bind to _________ and SH3 domains bind
b. RNA polymerase ii. many structurally to _________.
similar subunits
Answer: phosphorylated tyrosine residues, proline
c. myoglobin iii. many subunits of
containing peptides
varied structure
Answer: a–ii, b–iii, c–i 12. _______ residues form cis peptide bonds in proteins
with significant frequency.
5. Most protein conformational changes involve breaking
Answer: Proline
and reforming several covalent bonds along the
polypeptide chain. 13. α helices that have one hydrophobic face and one
True/False hydrophilic face are known as _________ helices.
Answer: amphipathic
6. The only genetically encoded amino acid without a
stereoisomer is:
a. alanine Quantitative/Essay
b. tryptophan
14. A group of scientists isolate a novel strain of bacteria.
c. glycine
Although the bacteria have normal nucleic acids and
d. proline
proteins, the strain has an abnormal membrane. In
e. lysine
this strain, the lipid bilayer is twice as thick (70 Å) as
2 Chapter 4: Protein Structure

that of a normal strain of bacteria. This extra thickness do not gain stability from the hydrophobic effect. While
is entirely due to longer hydrophobic tails and not backbone hydrogen bonds could contribute stabilizing
due to changes in the charged head groups of the energy to the folded state, the backbone –NH and –C=O
phospholipids that comprise the membrane. If you groups of the unfolded polypeptide hydrogen bond to
isolated a single transmembrane helix from a protein water. Upon folding, formation of secondary structure
from this strain, how long would you expect it to be? elements replaces hydrogen bonds to water with
hydrogen bonds to other parts of the protein backbone.
Answer: Thus the summed energy of hydrogen bonding does not
70 Å membrane/1.5 Å per residue of alpha helix = change upon folding.
at least 47 residues.
18. Draw an alanine-alanine dipeptide. Indicate the peptide
15. After sequencing the genome of the bacteria in Problem bond. What factors restrict the rotation about the
14, the scientists want to predict all of the membrane 4Q18 bond?
peptide
proteins.
Answer:
a. Describe the hydrophobicity index procedure for
predicting α helical membrane proteins. O O
Peptide bond
b. How would the hydrophobicity index calculation
differ for this new strain compared to normal bacteria? H2N CH C N CH C OH
Answer:
a. The hydrophobicity index at each residue CH3 CH3
represents the mean hydrophobicity value of the
surrounding contiguous set (“window”) of residues Rotations are strongly hindered because the peptide
centered on the residue in question. Clustered regions bond has partial double bond character. Evidence for
of negative hydrophobicity index indicate the presence this double bond character includes the observation
of a transmembrane helix. from high resolution crystal structures that the peptide
bond length is shorter than normal single N–C bond
b. For this strain of bacteria, since the membrane
lengths.
is thicker than normal, the “window” size should
be increased. Rather than calculating the average 19. The simplest form of the Ramachandran diagram
hydrophobicity for 19 residues, we should calculate (shown in Figure 4.20) is calculated by ignoring
the average for at least 40 residues. This represents hydrogen bonding, interactions with water, and the
a minimum contiguous stretch of residues to stretch hydrophobic effect. Why is this simple form of the
across the lipid bilayer. diagram still an effective predictor of protein backbone
conformation?
16. Using the hydrophobicity scale in Figure 4.74, calculate
the hydrophobicity index for the 19 contiguous residue Answer:
window defined by the following sequence:
 The simplest Ramachandran diagram shows which
Pro-Gly-Ala-Val-Val-Ile-Trp-Phe-Val-Val-Met-Ser-Ala-Ile- conformations are prevented due to interatomic
Ile-Phe-Tyr-Ala-Thr collisions. The repulsive energy of such collisions is
Could this segment be part of a transmembrane helix? so high that it dominates the forbidden regions of
the diagram. Taking the other forces into account
Answer: modulates the favorability of some regions of the
Pro-Gly-Ala-Val-Val-Ile-Trp-Phe-Val-Val-Met-Ser-Ala-Ile- Ramachandran diagram, but the overall forbidden
Ile-Phe-Tyr-Ala-Thr. and allowed zones are not changed.
0.4 + 4.5 + 2.1 – 2.1 – 2.1 – 4.5 – 8.8 – 7.1 – 2.1 – 2.1 –
20. How are the large loop elements of scorpion toxin
2.9 + 2.1 + 2.1 – 4.5 – 4.5 – 7.1 – 2.9 + 2.1 + 0.8
stabilized without participating in the hydrophobic
= –36.6; –36.6/19 = –1.9 core?
This segment could be part of a transmembrane helix Answer:
as the hydrophobicity index is negative and there are no
In addition to the hydrophobic core, this particular
charged residues.
protein fold is held together by four disulfide bonds. In
17. Why are isolated secondary structural elements not the loop regions, the disulfide bonds provide covalent
usually stable in isolation, even though all backbone linkages between different parts of the protein
hydrogen bonds are satisfied? backbone. Since the energy required to break these
covalent bonds is very large, these loop elements can
Answer: be stabilized without significant contributions from the
The hydrophobic effect is the major force driving hydrophobic effect.
protein folding and governing the stability of globular
proteins. Isolated secondary structural elements cannot 21. Draw a helical wheel for the following sequences:
bury hydrophobic residues away from water and thus 
 a. Leu-Asp-Lys-Ile-Val-Arg-Phe-Leu-Gln-Ser-Tyr
PROBLEMS and solutions 3

b. Leu-Asp-Leu-Lys-Arg-Ser-Glu-Leu-Asn-Tyr-Asn 23. Why are binding sites in proteins often located at


interdomain boundaries?
For each, highlight the hydrophobic residues by marking
them with an asterix (*). Do these sequences form Answer:
amphipathic helices? Mutations that confer novel binding are more likely to
be tolerated by the protein if they do not interfere with
Answer:
proper folding, making surface residues at interdomain
a. boundaries better candidates for binding sites. Also,
Y interdomain regions possess deep invaginations within
F which small molecules can bind.
I* 24. In contrast to globular proteins and regions exposed
S K
to the cytoplasm, the integral membrane portions of
membrane proteins contain very few residues not
L* ordered into secondary structure elements such as
R β sheets and α helices. Why is satisfying backbone
L* hydrogen bonds by forming secondary structure
D elements more important for protein folding in the
V* membrane than in solution?
Q Answer:
Yes, this forms an amphipathic helix. In solution, residues in loop segments can form
backbone–water hydrogen bonds and thus do not pay
b.
a large energetic penalty for failing to satisfy these
N interactions with other backbone atoms. In contrast, in
E the membrane the lipid tails do not provide hydrogen
bond donor or acceptors. Additionally, there is no
K
Y L* energetic gain from burying a hydrophobic core in the
membrane since the hydrophobic environment negates
the importance of the hydrophobic effect. Secondary
L* structure formation is an efficient mechanism
S for completely satisfying the hydrogen bonding
L*
requirements of the peptide backbone. These hydrogen
D
bonds provide the major energetic force driving folding
R in the membrane.
N
25. A scientist isolates a membrane protein that transports
sodium ions from inside the bacterium (where it is at
No, this does not form an amphipathic helix. low concentration) into solution (where the sodium ion
concentration is high).
22. The ridges and grooves of an α helix form an angle of
~25° or ~45° to the helical axis. What are the spacings a. Does this protein use active or passive transport?
between the residues that line these two types of b. What are two energy sources for accomplishing
grooves? How do these geometric considerations this transport?
constrain the packing of pairs of helices at relative
Answer:
angles of 50° and 20°?
a. This protein uses active transport since it is moving
Answer: an ion to a region of higher concentration from inside
~25° grooves are separated by four residues; ~45° the cell where the ion is at a lower concentration.
grooves are separated by three residues. In order to b. Two mechanisms for coupling energy to transport
pack the two helices using two ~25° grooves, one against a concentration gradient are to convert light
helix must be turned 180° and placed on top of the energy to a conformational or chemical change (as
other helix. In the interface between the two helices in rhodopsin) and to use ATP hydrolysis to drive a
the directions of the ridges and grooves must thus be conformational in the protein (as in the maltose
inclined by an angle of about 50° (25° + 25°) in order transporter).
for the ridges of one helix to fit into the grooves of the
other and vice versa. 26. What is the conformational change that occurs when
bacteriorhodopsin absorbs light?
To form an interface between ridges formed using a
~25° and a ~45° groove, one helix must be rotated 180°. Answer:
The helices are then on the same side of the helical axis The conformational change occurs on the covalently
and make an angle of 20° (45° – 25°) when the ridges bound cofactor retinal. When retinal absorbs a photon,
and grooves fit into each other. the bond between carbon atom 13 and carbon atom
4 Chapter 4: Protein Structure

14 is isomerized from a trans conformation to a cis


conformation.

27. Two mutated bacteriorhodopsin proteins have a Val or


Glu at the normal Asp 85 position.
a. Do you predict that either of these mutated
proteins will transport protons?
b. If not, at what stage of the proton transport “wire”
will the proton transport be blocked compared to wild
type?
Answer:
a. It is possible that the Asp-to-Glu mutation will still
transport protons because the Glu sidechain can pick
up the proton from retinal (acting as a Schiff base). In
contrast, Val does not have a charged sidechain and
cannot remove the proton from retinal, thus it will likely
be unable to transport protons.
b. In the Val mutant, the proton removed from
retinal (acting as a Schiff base) cannot be moved to
the hydrogen bonded network of water in the protein
transport wire. Without the removal of this proton,
retinal will be perpetually charged and unable to absorb
an additional proton from Asp 85 to eventually conduct
to the water portion of the proton “wire.”
The MOLECULES
of LIFE
Physical and Chemical Principles

Solutions Manual

Prepared by James Fraser and Samuel Leachman


Chapter 5
Evolutionary Variation
in Proteins

Problems and Solutions

True/False and Multiple Choice

1. The BLOSUM scoring matrix gives a measure of how 7. The core of a protein generally contains residues from
conservative a mutation is. For substitutions of aspartic the ________ class of amino acids.
acid (Asp, D), which of the following orderings correctly Answer: hydrophobic/nonpolar
places the amino acids from most conservative to least
conservative? 8. Globin proteins bind the iron-containing ________
cofactor.
a. K,L,A,C,E,S
b. E,S,K,A,C,L Answer: heme
c. L,C,A,K,S,E 9. A disulfide bond links two _______ residues.
d. A,C,E,K,L,S
Answer: cysteine
2. An environment profile in the 3D-1D profile method 10. According to the BLOSUM substitution matrix, the most
compares: conservative mutation from tryptophan (W), other than
I. the stability of the amino acid in varying solvents to itself, is to ______, which has a score of ______.
II. the burial of each amino acid in the structure
Answer: tyrosine, 2
III. the hydrophobicity surrounding each amino acid
IV. the type of secondary structure element containing 11. Many soluble human proteins can be expressed in the
each amino acid E. coli bacteria or using an in vitro translation system.
a. I, II, III, and IV How can these proteins fold without the cellular
b. I and IV machinery present in human cells?
c. II, III, and IV Answer: The “thermodynamic hypothesis” states
d. II and IV that proteins adopt native structures that optimize
thermodynamic properties. Since sequence determines
3. Protein domains can be assembled together in many structure and most proteins do not require post-
different ways, because surface sidechains can be translational modifications, external templates, or
mutated easily without losing protein stability. specific molecular chaperones to fold, many proteins
True/False can fold after translation by a different organism or even
a cell-free (in vitro) translation system.
4. In contrast to ribonuclease, some proteins cannot fold
without the assistance of proteins known as molecular 12. What level of activity (1, 10, or 100%) is predicted
chaperones. This means the thermodynamic hypothesis for ribonuclease-A when it is subject to each of the
following stepwise procedures?
of protein folding does not apply to these proteins.
a. i. denatured, then
True/False
ii. reduced, then
5. Two proteins that share more than 50% sequence iii. exposed to oxygen, then
identity over a 100-residue stretch are likely to have the iv. refolded by removing urea
same three-dimensional fold. b. i. denatured, then
ii. reduced, then
True/False
iii. refolded by removing urea while exposed
to oxygen
Fill in the Blank c. i. denatured, then
ii. refolded by removing urea
6. Two common chemical denaturants of proteins are Rationalize the predictions based on the effect
guanidinium and _______. of denaturation and reduction–oxidation of the
Answer: urea ribonuclease-A cysteine residues. Assume that all
2 Chapter 5: Evolutionary Variation in Proteins

possible unfolded conformations are equally likely and However, we must correct for the vertical ordering: it
that folding is faster than oxidation. does not matter which lysine–glutamate pair is picked
Answer: first, second, etc., just that the correct pairs are picked.
a. Low activity (~1%). (i) The protein is first denatured, This means we need to divide the total number by 6!
forming mostly random structures; however, all disulfide (6 pairs to choose first × 5 pairs to chose second...etc).
bonds remain intact. (ii) Reduction causes all disulfide 518,400/(6 × 5 × 4 × 3 × 2 × 1) = 720.
bonds to be broken. (iii) Exposure to oxygen causes the Off the 720 random structures populated during cross-
disulfide bonds to reform. They reform with random linking in experiment (b) only one will be correct. Thus
pairings, since the protein is unfolded and populating the activity expected is 1/720 = 0.0014, or 0.14%.
many random structures. (iv) The protein refolds, but 14. Use the BLOSUM substitution matrix (Figure 5.11) to
only those molecules in which the disulfides formed compute the sum of the substitution scores (Sij) and
correctly are active. the overall likelihood ratio (L) of the following short
b. High activity (~100%). (i) The protein is first alignments:
denatured forming mostly random structures; however, a. PADKTN
all disulfide bonds remain intact. (ii) Reduction causes PEEKSA
all disulfide bonds to be broken. (iii) Exposure to oxygen b. KFLASV
causes the disulfide bonds to reform, but since folding ATWDPE
is faster than oxidation, the disulfide bonds reform Answer:
correctly. a. Sequence: P A D K T N
c. High activity (~100%). (i) The protein is first Sequence: P E E K S A
denatured forming mostly random structures; however, Score: (7)+(–1)+(2)+(5)+(1)+(2)
all disulfide bonds remain intact. (ii) The protein refolds Sum of Scores = 12
and since the disulfide bonds are already intact, full Likelihood = 2(Score/2) = 2(12/2) = 64
activity is restored. b. Sequence: K F L A S V
13. A folded protein structure contains six ion pairs Sequence: A T W D P E
between lysine and glutamate. There are no other Score: (–1)+(–2)+(–2)+(–2)+(–1)+(–2)
possible ion pairs in the protein. A chemical cross- Sum of Scores = –10
linker forms a covalent bond between ion-paired lysine Likelihood = 2(Score/2) = 2(–10/2) = 1/32 = 0.3125
and glutamate sidechains. By analogy to the Anfinsen
15. Based on the BLOSUM matrix, how much more likely is
experiment, the following experiments are done:
it that:
a. folded protein → unfold with urea → remove urea a. tryptophan is substituted by a tyrosine than a
to refold → add cross-linker → remove excess cross- tryptophan is substituted by a cysteine?
linker → measure activity b. sequence (i) DPKRFL is related to sequence (ii)
EPKRFI than sequence (i) is related to sequence (iii)
b. folded protein → unfold with urea → add cross-
KGKRYA?
linker → remove excess cross-linker → remove urea to
refold → measure activity To answer this question, you must calculate the ratio
of the likelihood ratios for each case. Explain the
The cross-linker does not by itself alter the activity significance of higher likelihood.

of the protein when the correct ion pairs are formed.
A protein with incorrect ion pairs cross-linked would Answer:
be inactive. The activity measured at the beginning a. The score for W → Y substitution = 2; The
and end of experiment (a) is 100%. What percentage likelihood = 2(Score/2) = 2(2/2) = 2. The score for a
of the activity do you expect to observe at the end of W → C substitution = (–2); The likelihood = 2(Score/2) =
experiment (b)? Assume that all unfolded conformations 2(–2/2) = 0.5.
are equally likely. Lij/Lik = 2/0.5 = 4 times higher likelihood.
Answer: This means that a W → Y substitution is more
There are six ways of picking the first lysine and six conservative than a W → C subsitution.
ways of picking the first glutamate (6 × 6 = 36). b. The sum of scores for (i) → (ii) = 27; The likelihood
There are five ways of picking the second lysine and five = 11,585.2. The sum of scores for (i) → (iii) = 9; The
ways of picking the second glutamate (5 × 5 = 25). likelihood = 22.6.
There are four ways of picking the third lysine and four Lij/Lik = 11,585.2/22.6 = 512 times higher likelihood.
ways of picking the third glutamate (4 × 4 = 16). This means that (i) is more likely to be related to (ii) than
There are three ways of picking the fourth lysine and to (iii).
three ways of picking the fourth glutamate (3 × 3 = 9). 16. Proteins known as cyclophilins catalyze proline cis-trans
There are two ways of picking the fifth lysine and two isomerization. A catalytic arginine residue is invariant
ways of picking the fifth glutamate (2 × 2 = 4). in all cyclophilins. All other positions change residue
There is one way of picking the last lysine and one way identity in different cyclophilins despite the fact the
of picking the last glutamate (1 × 1 = 1). variant proteins have the same overall fold and general
The total number of ways to pick is: 36 × 25 × 16 × 9 × 4 catalytic activity. Explain, given the relationship between
× 1 = 518,400. protein sequence and structure, how catalytic activity is
PROBLEMS and solutions 3

retained even though most residues can change.
 Explain why the distribution of protein sizes has the
Answer: periodicity that is seen in the diagram and estimate a
The relationship between sequence and structure value for x.
is asymmetric. Many proteins can have the same Answer:
structure despite different (degenerate) sequences. A reasonable value for x is approximately 100–150.
However, sequences fold into only one structure This is the average size of a protein domain (which are
(following the thermodynamic hypothesis). Only normally 50–200 residues). The periodicity is observed
residues that perform specific functions, such as the because proteins are modular and are expanded by
catalytic arginine in cyclophilins and the histidine in addition of different domains. The peaks at 1x, 2x, and
helix F in globins are invariant. 3x derive from proteins containing 1, 2, or 3 domains.
17. Why is the sequence similarity generally higher when 21. The number of distinct protein folds is limited. Why
comparing two globins from mammals than when might this be so? Approximately how many folds are
comparing a globin from a mammal and a globin from a there (hundreds, thousands, millions, or billions)?
plant? Answer:
Answer: There are likely to be only thousands of folds because
It is likely that the both the mammalian and plant natural selection will favor protein sequences that will
globins derived from a single ancestral globin and have fold into stable structures. In order to fold into a stable
expanded through gene duplication. The mammalian structure, it is necessary to be built up of secondary
globins share a common ancestor with each other that structure elements and have a hydrophobic core. It is
is more recent than the mammalian globins share with probable that there are a finite number of orientations
plant globins. and combinations of secondary structure elements
that will produce a hydrophobic core of acceptable
18. How might the tolerated variation in the hydrophobic
geometry to enable protein folding.
core of the lambda repressor change if the hydrophobic
core of wild type lambda repressor were more tightly 22. How many folds are represented below? Describe what
packed? CATH class each fold belongs to and how the secondary
Answer: structure is arranged in each fold.
The protein would likely be less tolerant of Q5.12 (seq_struct_70_v1)
mutations. Proteins undergo fluctuations, even in the
hydrophobic core. They are not perfectly packed as
interlocking pieces of a puzzle. This property allows for
accommodation of differently shaped sidechains and for
toleration of mutation.
19. What characteristics define a protein domain?

Answer:
Domains have a distinct topology and a self-contained
hydrophobic core. They generally contain 50–200
residues.
A B C

20. The diagram below shows the size distribution for


globular proteins produced by the bacterium E. coli.
Q5.10 (seq_struct_71_v1)
frequency of occurrence

D E F

Answer:
There are four folds. A and E have the same Rossmann-
like mixed α/β fold. B has a unique mixed α/β fold. C has
an all β sheet structure arranged in a barrel. D and F
have a four helix bundle.
23. A threading program is used and it gives two possible
predicted folds for a particular sequence. They differ
mainly in the placement of a single helix. In predicted
1x 2x 3x fold (A), the helix is entirely within the hydrophobic
number of residues in the protein core. In predicted fold (B), the residues of the helix are
4 Chapter 5: Evolutionary Variation in Proteins

mostly exposed to solvent. The sequence of the helix is


LIVFLAIL. Explain how the 3D-1D profile method could
be used to distinguish between the two possible folds.
Answer:
All of the residues in the helix are hydrophobic and have
positive scores as α helical buried positions. In contrast,
these residues have negative scores for exposed
α helical positions. Thus, fold A correctly predicts that
the α helix should be part of the hydrophobic core of
the protein.
24. What structural features of the Rossmann domain
enable it to bind nucleotides?
Answer:
The negatively charged phosphate groups of the
nucleotides interact with the P loop at the positive pole
of the helix dipole generated by the first α helix of the
Rossmann fold.
25. Both thioredoxin reductase and glutathione reductase
use fused FAD- and NADPH-binding domains and
dimerization to accomplish their cellular functions.
Which structural feature likely evolved first, the fused
domains or dimerization?
Answer:
The fused FAD and NADPH domain structure is
conserved between both structures. However, the
relationship between the individual subunits of the
dimers is different for each protein (see Figure 5.43).
This implies that domain structure evolved before the
split of individual thioredoxin and glutathione reductase
lineages and that dimerization evolved later.
The MOLECULES
of LIFE
Physical and Chemical Principles

Solutions Manual

Prepared by James Fraser and Samuel Leachman


Chapter 6
Energy and
Intermolecular Forces

Problems

True/False and Multiple Choice

1. Which of the following properties are extensive (choose c. constant temperature


all that apply): d. reversible expansion
a. temperature e. expansion at a constant pressure followed by
b. pressure reversible compression
c. amount of heat released
d. density 5. Energy is a good indicator of the direction of
e. energy spontaneous change for macroscopic mechanical
f. molarity objects but not for molecular processes.
g. number of moles True/False
h. mass
i. volume 6. Which of the following statements about potential
energy is true?
2. A piston containing an ideal gas expands isothermally
from 7 atm pressure to 2 atm pressure. The energy of a. The potential energy of an ideal gas is equal to its
the system (that is, the contents of the piston) remains kinetic energy.
constant during this process. b. The potential energy of an atom is the work
done in moving the atom from infinity to its
True/False
present position.
3. A protein is negatively charged, but it binds a negatively c. Potential energy is always absolutely conserved.
charged small molecule faster than it binds a positively d. The potential energy of a system always increases.
charged small molecule. The most reasonable e. Potential energy is an intensive function.
explanation for this phenomenon is:
a. Even though the protein is negatively charged 7. The type of function that best describes the energy of a
overall, electrostatic focusing effects provide a hydrogen bond as the distance between the hydrogen
pathway for a negatively charged molecule to enter atom and the acceptor atom varies between 1.5 and
the active site. 2.5 Å, is:
b. The presence of water molecules screen the
electrostatic effects. Problems, multiple choice, Q7 (forces_53_v1)
c. The negatively charged molecule makes stronger (a) (b)
hydrogen bonds than the positively charged molecule.
energy (increasing)

energy (increasing)

d. Charged proteins normally bind substrates with the


same overall charge.

4. The enthalpy change for a process is equal to the heat


transferred to the system under which of the following
conditions?
a. constant volume
1.5 2.5 1.5 2.5
b. constant pressure distance (Å) distance (Å)
(c) (d)
The Molecules of Life by John Kuriyan, Boyana Konforti, and David Wemmer © Garland Science
ing)

ing)
energy (incr

energy (incr
∆U = 0 (all isothermal expansions)
V2
w = –nRT ln
V1
2 Chapter 6: Energy and Intermolecular Forces
P1
1.5 2.5 1.5 2.5 = –nRT ln
distance (Å) distance (Å) P2
(c) (d) 10
= –1 mol × 8.31 J•K–1•mol–1 × 300 K × ln
1
= –5740 J
energy (increasing)

energy (increasing)
w = –5.74 kJ
q = –w = 5.74 kJ
15. One mole of an ideal monatomic gas is initially at
300 K and 1010 J•L–1 pressure inside a cylinder with a
frictionless piston. The cylinder is an isolated (adiabatic)
system. The gas expands against zero external pressure.
1.5 2.5 1.5 2.5 When the volume expands to 24.7 L, a peg stops the
distance (Å) distance (Å)
piston. Calculate ΔU (the change in energy), q (the
heat transferred to the system), and w (the work done
Answer: Graph shown in a.
by the system) when the expansion is isothermal and
reversible.
Fill in the Blank Answer:
q = 0 (isolated system)
8. A reaction that releases energy is called an ________
w = 0 (no external pressure)
reaction.
ΔU = 0 (all isothermal expansions)
Answer: exothermic
16. One mole of a different ideal monatomic gas is initially
9. The first law of thermodynamics states that the total at 300 K and 1010 J•L–1 pressure inside a cylinder
energy of the system and surroundings remains with a frictionless piston . Calculate ΔU (the change
__________. in energy), q (the heat transferred to the system),
Answer: constant and w (the work done by the system) when the
expansion is isothermal and against a constant
10. Electrostatic interactions are governed by __________’s pressure of 101 J•L–1.
law Answer:
Answer: Coulomb For work against constant pressure,

11. An ion pair with lysine can be formed by the sidechain w = –PEXT∆V = –PEXT (V2 – V1)
of _________ or ____________.
nRT nRT
Answer: aspartic acid, glutamic acid = –PEXT –
P2 P1
12. The energy for van der Waals interactions approaches 1 mol × 8.31 J•K–1•mol–1 × 300 K
_____ as the distance between atoms goes to zero and = –101 J•L–1 –
101 J•L–1
approaches _____ at infinite distance.
Answer: infinity kcal•mol–1, 0 kcal•mol–1 1 mol × 8.31 J•K–1•mol–1 × 300 K
13. The force arising from a noncovalent interaction 1010 J•L–1
between two atoms is always ______ at the energy = –2240 J
minimum. w = –2.24 kJ
Answer: zero q = –w = 2.24 kJ

17. The complete oxidation of one mole of a sugar


Quantitative/Essay
produces carbon dioxide and water. 2000 kJ of heat is
14. One mole of an ideal monatomic gas is initially at 300 transferred from the system to the surroundings. The
K and 1010 J•L–1 pressure inside a cylinder with a rearrangement of bonds as 0.5 moles of the sugar are
frictionless piston. (Note that 1 J•L–1 = 103 Pa.) oxidized generates heat in an open test tube (101 J•L–1
pressure and 300 K temperature). What is the change in
The gas expands until the pressure is 101 J•L–1. internal energy of the system (ΔU)? What is the change
Calculate ΔU (the change in energy), q (the heat in enthalpy of the system(ΔH)?
transferred to the system), and w (the work done by Answer:
the system) when the expansion is isothermal and
ΔU = q + w. There is no work done. Therefore:
reversible.
ΔU = q = –2000 kJ•mol–1 × 0.5 mol released by the
Answer: system
∆U = 0 (all isothermal expansions) ΔU = –1000 kJ
V2 ΔH = ΔU + PΔV. Since there is no volume change,
w = –nRT ln ΔV = 0 and ΔH = ΔU = –1000 kJ.
V1
P1 18. A system is maintained at thermal equilibrium (at the
= –nRT ln
P2 same temperature) with its surroundings and has an
–1 –1 10
= –1 mol
The Molecules × 8.31
of Life J•KKuriyan,
by John •molBoyana
× 300 K × lnand David Wemmer © Garland Science
Konforti, 1
= –5740 J
w = –5.74 kJ
PROBLEMS and solutions 3

enthalpy of 50 kJ. It has 100 kJ of heat transferred to it, 21. Consider a protein with different energy levels for
which causes it to expand against a constant pressure electronic transitions and molecular vibrations. What
of 1 atm. It is then compressed back to its initial are the approximate energy differences between the
volume. All steps are at constant temperature. What is levels for each type? To what extent do the populations
its final enthalpy? of levels within each type explain why the heat capacity
Answer: of a protein is highest at the unfolding point?
50 kJ. Because all steps are at constant temperature, Answer:
ΔU = 0, and because the system undergoes no volume Electronic energy levels are far apart in energy
change, ΔV = 0 and ΔH = ΔU. The heat initially put into (103–104 kJ•mol–1). These are not generally involved in
the system to change its state is balanced by the work changes in heat capacity when a protein is unfolding,
put in to return it to its initial state. The enthalpy doesn’t since the energy absorbed is not enough to populate
change from its initial value because enthalpy is a state the higher energy levels.
function. Translational energy level spacings are very small
(10–14 kJ•mol–1). The translational movement of a
19. How much kinetic energy does a system containing
protein does not greatly change depending on whether
3 moles of an ideal gas at 300 K possess? What is the
it is folded or not. These levels are present during
heat capacity at constant volume? How much heat
unfolding, but do not contribute the increased heat
would need to be transferred to the system to raise the
capacity since both folded and unfolded proteins can
temperature by 15°C?
readily access many translational energy levels.
Answer:
3 Molecular vibration energy levels are moderate
K.E. =  nRT (1–10 kJ•mol–1). A folded protein that is heated will
2
3 adjust its arrangements and absorb energy through
= × 3 mol × 8.31 J•K–1•mol–1 × 300 K
2 fluctuations breaking many weak interactions. These
= 11,218.5 J = 11.2 kJ rearrangements are highest during unfolding, explaining
3 why the maximum heat capacity occurs at the unfolding
CV =  nR
2 temperature. Access to more molecular vibration
3 energy levels in the unfolded state explains the higher
= × 3 mol × 8.31 J•K–1•mol–1
2 heat capacity of unfolded proteins relative to folded
= 37.4 J•K–1 proteins.
q = CV × ΔT = 37.4 J•K–1 × 15 K = 561 J
22. The only two accessible conformations of a protein
20. Below are two graphs of heat capacity changes in differ by 2 kJ•mol–1. What percentage of protein
a differential scanning calorimeter experiment. In molecules will be in the higher energy state:
the experiment, protein molecules in solution are a. at 350 K?
unfolded by increasing the temperature. In both graphs, b. at 270 K?
temperature increases from left to right. Which graph
Answer:
correctly characterizes the heat capacity changes of
a protein as it is heated? Which characteristics in the a. –(Uhigh–Ulow) –2000 J•mol–1
Phigh
Problems,
graph long
lead to thisanswer, Q7 (forces_54_v1)
conclusion? = e RT = e 8.31 J•K–1•mol–1 × 350 K = 0.503
Plow
9.0 9.0
Phigh
Plow
Phigh = = 33.5 %
Phigh
+1
CP (mJ•°C–1)

CP (mJ•°C–1)

6.0 6.0
Plow

b. –(Uhigh–Ulow) –2000 J•mol–1


3.0 3.0 Phigh RT 8.31 J•K–1•mol–1 × 270 K
=e =e = 0.410
Plow
Phigh
0 0 Plow
temperature temperature Phigh = = 29.1 %
Phigh
+1
Answer: Plow
The graph on the left is correct. Both graphs correctly
indicate that the heat capacity at low temperatures,
23. Consider a ribosome translating a codon to an amino
when the protein is folded, is lower than the heat
acid. Compared to a perfect pairing, imagine that a
capacity at high temperatures, when the protein is
codon–anticodon pairing with one incorrect base pair
unfolded. However, the heat capacity of a protein peaks
is 10 kJ•mol–1 higher in energy, two incorrect base pairs
at its melting temperature because relatively more
is 20 kJ•mol–1 higher, and three incorrect base pairs is
energy goes into breaking intramolecular interactions.
30 kJ•mol–1 higher.
It then decreases at higher temperatures when fewer
unbroken interactions persist. Only the left graph How many ways are there of making a one-, two-,
indicates this correctly. or three-base-pair mismatch? What is the partition
The Molecules of Life by John Kuriyan, Boyana Konforti, and David Wemmer © Garland Science
4 Chapter 6: Energy and Intermolecular Forces

function of the codon–anticodon system at 300 K? What is the energy if the residues are a) on the protein
If energy difference is the only consideration, what surface (ε = 80) and b) in the core of the protein (ε = 2)?
percent of decoding events occur without an error (no Answer:
mismatches) at 300 K? a. 1 qiqj
U= × 1390 kJ•mol–1
Hint: There are three incorrect single-base mismatches ε rijÅ
in each of the three positions (A, B, and C)(3 × 3 = 9); 3 ×
3 = 9 incorrect two-base mismatches in three possible 1 –1
U= × 1390 kJ•mol–1 = –4.96 kJ•mol–1
combinations (AB, AC, and BC) of positions (9 × 3 = 27); 80 3.5
and 3 × 3 × 3 = 27 incorrect three-base mismatches.
b. 1 qiqj
Answer: U= × 1390 kJ•mol–1
–Ui –Uzeromismatches –Uonemismatch –Utwomismatches ε rijÅ
Q = ∑e kBT =e kBT +9×e kBT + 27 × e kBT
1 –1
U= × 1390 kJ•mol–1 = –199 kJ•mol–1
i
–Uthreemismatches
2 3.5
+ 27 × e kBT

–Ui –10 kJ–1•mol–1 –20 kJ–1•mol–1 –30 kJ–1•mol–1


26. A particular sidechain has a dihedral angle for which the
= e kBT + 9×e kBT + 27 × e kBT + 27 × e kBT
energy is governed by the following equation:
= 1 + 9 (0.018) +27 (0.0003) + 27 (0.000006) Udihedral = 2 × cos (3 × angle) kJ•mol–1
= 1 + (0.162) + (0.0081) + (0.0002)
a. What is the energy of a sidechain with an angle of
= 1.1724 60°?
–Uo –0
kBT b. With an angle of 90°?
Ntotal e kBT 100 e 100 c. What is the relative population of sidechains with
N0 = = = = 85.3 %
Q 1.1724 1.1724 an angle of 60° versus 90° at 300 K?
Answer:
Therefore, approximately 85.3% of all amino acids
are translated faithfully. The redundancy built into a. Udihedral = 2 × cos(3 × 60) kJ•mol–1 = –2 kJ•mol–1
the genetic code and other energetic and kinetic
considerations raise this number considerably. b. Udihedral = 2 × cos(3 × 90) kJ•mol–1 = 0 kJ•mol–1
24. The energy of a covalent bond as a function of –(U60–U90) –(–2000 J•mol–1 – 0 J•mol–1)
interatomic distance, r, is described by the following c. P60 RT 8.31 J•K–1•mol–1 × 300 K
=e =e = 2.23 times
equation: P90 as many
U (r ) = 2000 ( r – 1.5 ) 2
In this equation, energy is expressed in units of kJ•mol−1
and distance in Å. What is the equation that describes 27. For two atoms, separated by distances between 0 and
the force with respect to position? What is the energy 4 Å, the van der Waals energy is well described by the
when the interatomic distance is 2 Å? At the same following equation:
2
value of the interatomic distance, what is the force and  rmin 
is the force pushing the bonded atoms closer together U (r ) = ε  
 r 
or further apart? where ε = 2 kJ•mol−1. Calculate the value of rmin if the
Answer: energy at 3 Å is 1.5 kJ•mol−1.
F(r) = –dU/dr = –4000 kJ•mol–1 × Å (r – 1.5 Å) Answer:
2
U(s) = 2000 kJ•mol–1× Å–2 (2 Å – 1.5 Å)2 rmin
U(r) = ε
= 2000 kJ•mol–1× Å–2 (0.25 Å2) r
= 500 kJ•mol–1 2
rmin
1.5 kJ•mol–1 = 2 kJ•mol–1
F(2) = –4000 kJ•mol–1 × Å–2 (2 Å – 1.5 Å) 3Å
= 2000 kJ•mol–1× Å rmin2
0.75 =
9 Å2
The force is attractive. If one atom were held fixed, the
force on the second atom would accelerate it in the 6.75 Å2 = rmin2
negative direction, towards the first atom. Also, notice
rmin = 2.6 Å
that the ideal bond length is 1.5 Å, so atoms spaced 2 Å
apart would be attracted.
28. A calorimeter is used to measure the heat capacity of
25. A salt bridge between an arginine and a glutamic acid two molecules, A and B. Molecule A is a simple small
has the two bridging atoms 3.5 Å apart. Assume that molecule, whereas molecule B is a complex polymer
these bridging atoms have elementary charges of +1 (like DNA). Define heat capacity and explain why
and –1, respectively. molecule B is likely to have a higher heat capacity.

The Molecules of Life by John Kuriyan, Boyana Konforti, and David Wemmer © Garland Science
PROBLEMS and solutions 5

Answer:
The heat capacity is the amount of heat required
to raise the temperature of the system one Kelvin.
Molecule B likely has a higher heat capacity because
it has more internal energy modes (vibrational and
conformational modes) than the simpler molecule.
These low energy transitions will allow it absorb more
heat per change in temperature.
29. Water forms hydrogen bonds with proteins. How might
these hydrogen bonds alter the ability of a protein to
undergo conformational changes in water versus in the
gas phase?
Answer:
Water weakens the effective strength of hydrogen
bonds in proteins by competing for hydrogen bond
donors or acceptors. With stronger hydrogen bonds, as
in the gas phase, certain conformational changes will
require more energy and rarely occur spontaneously.
In water, however, the relative strength of protein
hydrogen bonds is closer to kBT and conformational
changes are more likely to occur.

The Molecules of Life by John Kuriyan, Boyana Konforti, and David Wemmer © Garland Science
The MOLECULES
of LIFE
Physical and Chemical Principles

Solutions Manual

Prepared by James Fraser and Samuel Leachman


Chapter 7

Entropy

Problems

True/False and Multiple Choice

1. It is easier to predict the bulk behavior of a small statements correctly characterizes the
number of molecules than a large number of molecules. system?
True/False a. It contains only blue molecules on the left side.
b. It contains only red molecules on the left side.
2. Two sets of molecules are mixed into a system at time
c. It contains a mixture of red and blue molecules
zero. After 10 seconds, equilibrium has been reached. At
throughout the system.
what time was the entropy of the system maximal?
d. It contains only blue molecules at the bottom of the
a. 10 seconds (equilibrium) system.
b. 5 seconds (half the time it takes to reach e. It has an equal number of red and blue molecules
equilibrium) on each side.
c. immediately upon mixing of the two sets of
molecules 7. A state corresponds to many different microstates.
d. prior to mixing of the two sets of molecules True/False
3. Consider a coin with two sides (H = heads; T = tails). The
probability of observing HHHHHTTTTT is equal to the Fill in the Blank
probability of observing HTHTHTHTHT.
True/False 8. The work done in a near-equilibrium expansion of an
ideal gas is _____________ than for a nonequilibrium
4. On a 10-sided die, with a side for each number from 1 to
expansion.
10, the probability of rolling a 5 is:
Answer: greater
a. 5/10
b. 1/10 9. When the volume of a system increases, its multiplicity
c. 1/9 ________.
d. 5/51 Answer: increases
e. 1/6
10. The log of the multiplicity of the system is an
5. An isolated molecular system exists in two states of
__________________ property of the system.
equal energy. State A has high multiplicity, whereas
State B has low multiplicity. Without any external agents, Answer: additive/extensive
a system in State B will spontaneously convert to State
11. The combined entropy of the system and surroundings
A.
always _______________ for a spontaneous process.
True/False
Answer: increases
6. A system is divided into two halves separated by a
removable divider. Initially, with the divider in place, 12. A drop of dye is added to a container of solvent. When
the left half has only red molecules and the right equilibrium is reached the concentration of dye will be
half has only blue molecules. The divider is removed ____________ within the container.
and equilibrium is reached. Which of the following Answer: uniform

The Molecules of Life by John Kuriyan, Boyana Konforti, and David Wemmer © Garland Science
2 Chapter 7: Entropy

Quantitative/Essay 16. Calculate the entropy of the system depicted below.


There are four types of molecules (that is, X, O, Y, and Z)
13. A coin is weighted deliberately so that the probability that can be arranged in any way in the available boxes.
of tossing heads is twice the probability of tossing tails. Problems, long answer,
(Hint: Use Equation 7.2.2.) Q3 (entropy_28_v1)
What is the probability of tossing three heads in a row?
What is the probability of tossing three tails in a row?
What is the relative probability of tossing three heads X
versus three tails?
Answer: increases Y X Y
2/3 chance of heads, 1/3 of tails.
3 heads in a row = (2/3)3 = 8/27 = 0.296.
3 tails in a row = (1/3)3 = 1/27 = 0.037. Z O Y O
Relative probability = (8/27)/(1/27) = 8 times more likely.
14. Shown below is a portion of Pascal’s triangle, with the O Z
tenth row filled in.

1 10 45 120 210 252 210 120 45 10 1 O

Answer:
The multiplicity of the system (W) is given by
a. 1Fill 10
in the
45values
120 for
210the eleventh
252 row.
210 120 45What
10 simple
1
M!/(X!O!Y!Z!(M – X – O – Y – Z)!).
rule can you use to fill in these values?
W = 25!/(2!3!4!2!14!) = 308,897,820,000
b. Using Pascal’s triangle, calculate how much more The entropy is given by kB lnW = kB ln(308,897,820,000)
likely1 it is10to get = 26.46 kB.
45 five
120 heads and 210
210 252 six tails
120 in45
a series
10 of1 11
coin tosses than getting four heads and seven tails. 17. Consider the systems below consisting of different
1 11 55 165 330 462 462 330 165 55 11 1
Answer: numbers of identical molecules (indicated by the
symbol X) and equal-sized grid boxes that are either
1 10 45 120 210 252 210 120 45 10 1 empty or occupied by a molecule. Which system has
a higher entropy? What is the difference in entropy
1 11 55 165 330 462 462 330 165 55 11 1 Problems,
between longB?answer, Q4 (entropy_29_v1)
A and

a. Simple rule: The numbers at the edges are always A.


X
1. All the other numbers are the sums of the two
numbers diagonally above.
b. According to Pascal’s triangle, the multiplicity for X
getting five heads is 462, and for four heads it is 330. So,
the ratio of the probabilities is:
X
462
= 1.4
330
X
15. Consider the following three cases of grid boxes.
Molecule X can move between and occupy any box. B.
Calculate
Problems, the multiplicity
long answer,and
Q2entropy for each case.
(entropy_27_v1) X X

a. X X X X X

b. X X
X
c. X X X
X X
Answer:
For each let X be the number of X molecules and M be
the total number of grid spaces. Answer:
For each the multiplicity can be calculated by 16!
WA =
M!/(X!(M – X)!). 12!4!
a. W = 6!/(4!(6 – 4)!) = 15; S = kB ln(W) = 2.71 kB
b. W = 6!/(2!(6 – 2)!) = 15; S = 2.71 kB 16!
WB =
c. W = 6!/(3!(6 – 3)!) = 20; S = 3.00 kB 6!10!

5×6
WAScience
The Molecules of Life by John Kuriyan, Boyana Konforti, and David Wemmer © Garland
= = 0.227
WB 12 × 11
16!
WA =
12!4!
16! PROBLEMS and solutions 3
WB =
6!10!

WA 5×6 21. In a near-equilibrium expansion, 15 kJ of work are done


= = 0.227 by a system at a constant temperature of
WB 12 × 11
300 K. To reach the same state, 5 kJ of work are done in
The multiplicity of the top (A) is lower; therefore, (B) has a nonequilibrium process. What is the entropy change
a higher entropy. for the two processes?
The difference in entropy is Answer:
WA Entropy is a state function, so the change will be the
kB ln = kB ln (0.227) = –1.48 kB
WB same in both cases:
∆S = qrev/T = 15 kJ/300 K = 50 J•K–1
18. A bag contains 20 coins, each marked by the one- 22. Considering the following system at two time points, A
letter code for an amino acid. What is the probability of Problems,
and B. long answer, Q9 (entropy_31_v1)
drawing each of the three large amino acids (Y, W, and
F) once, in any order, without returning the coins to the A. X X X X X
bag after each draw? What is the probability if coins are
returned to the bag after each draw? B. X X X X X
What is the probability of drawing the three letters
The system is divided by a movable partition. The
out in exactly the order of hydrophobicity (a unique
molecules (X) cannot move across the partition, but
ordering) without returning any coins to the bag?
they can move between the grid boxes on the same
Answer: side of the partition. At which time point does the
Large amino acids: system have the higher multiplicity? At which time point
Drawing each without replacement = 3/20 × 2/19 × is the system closer to equilibrium?
1/18 = 0.00088.
Answer:
With replacement = 3/20 × 2/20 × 1/20 = 0.00075.
At time point A:
Hydrophobicity: The multiplicity of the left side is: M!/(X!(M – X)!) =
Probability of drawing the first amino acid = 1/20, 4!/3! = 4.
second = 1/19, third = 1/18. 1/20 × 1/19 × 1/18 = The multiplicity of the right side is: M!/(X!(M – X)!) =
1/6840 = 1.46 × 10–4. 6!/(2!4!) = 15.
Problems,
19. What long answer,
is the multiplicity Q6 following
of the (entropy_30_v1)
system? The total multiplicity = mulitpicityright × multiplicityleft
= 4 × 15 = 60.
At time point B:
X X O O The multiplicity of the left side is: M!/(X!(M – X)!) =
5!/(3!2!) = 10.
X The multiplicity of the right side is: M!/(X!(M – X)!) =
5!/(2!3!) = 10.
The total multiplicity = mulitpicityright × multiplicityleft
O X = 10 × 10 = 100.
The multiplicity of time point B is higher. Since the
O multiplicity of time point B is higher, it is closer to
equilibrium than time point A.
23. Consider a coin that is weighted so that it is twice as
O likely to come up heads as tails. Answer the following
questions using the Gaussian distribution description for
the results of a series of coin tosses:
If an additional five empty grid boxes are added, does a. What is the most likely outcome of a series of 2000
the multiplicity of the system increase or decrease? coin tosses using this biased coin?
Answer: b. What is the relative probability of getting 1500
The multiplicity is M!/(X!O!(M – O – X)!) = 25!/(4!5!16!) = heads versus 1000 heads in a series of 2000 coin tosses
257,414,850. Adding additional volume always increases using this coin? Express your answer as a power of 10.
the multiplicity of the system. Answer:
20. System A has a multiplicity of 15, whereas System B has a. The mean value (μ, most likely outcome) is given
a multiplicity of 12. What is the total entropy of Systems by: μ = Mp where M is the number of events and p is
A and B? the probability of a favorable outcome.
Answer: The probability of obtaining heads is twice as large as
The entropies of the two parts will be additive. obtaining tails, so we can write:
Therefore we can calculate them separately.
p
SA = kB ln(15) = 2.71 kB; SB = kB ln(12) = 2.48 kB. p+ =1
Total entropy = 5.19 kB. 2

3
⇒ Kuriyan,
The Molecules of Life by John p =Boyana
1 Konforti, and David Wemmer © Garland Science
2
2
4 Chapter
p 7: Entropy
p+ =1
2

3 This change would not occur spontaneously because


⇒ p=1
2 the entropy is lower in the final state.
2 26. How much work is done in compressing one mole of an
⇒ p= = 0.6667
3 ideal gas from a starting volume of 1 L to a final volume
Hence, the mean value is: of 250 mL at a constant temperature of 293 K? What is
0.6667 × 2000 = 1333 the change in entropy? Assume that the process occurs
b. To calculate the relative probabilities we need to near-equilibrium (reversibly).
use the Gaussian distribution: Answer:
1  –(x – µ)2  V2
P(x) = exp  1/2  w = –nRT ln
V1
2σ  2σ2 
= –293 K × 8.31 J•K•mol–1 × ln (0.250)
where x is the number of favorable outcomes.
The standard deviation, σ, is given by: = 3375.3 J
–w –3375.3 J
σ= M × p × (1 – p) = 2000 × 0.6667 ×0.3333 = 21.1 ∆S = = = 11.5 J•K–1
T 293 K
The ratio of probabilities is given by:
– (1500 – 1333.3)2 + (1000 – 1333.3)2 27. If the increase in entropy indicates the direction of
P(1500) spontaneous change, how can a system ever undergo a
= exp  
P(1000)  2 × 21.082  process that results in a decrease in the entropy of the
system?
– 27778 + 111111 
= exp   Answer:
 889  The only way for a system to decrease its entropy
spontaneously is to couple to an external agent that
= exp(93.75) = 1040
is increasing its entropy. The external agent must
provide an external entropy increase that ensures that
24. A system consists of molecules that convert between
the summed entropy of the system plus the external
two colors (green and yellow). There are spaces for
increases.
10,000 molecules, but there are only 6000 molecules
in the system. The system starts in a state with 2000 28. In our calculations, why do we work with the natural log
green molecules and 4000 yellow molecules. What of the multiplicity?
is the entropy of the system? (Hint: Use Stirling’s Answer:
approximation.) ln(W) is an additive property of the system, and is more
Answer: convenient to work with than W because its numerical
Spaces! 10000! value remains manageable as the number of molecules
W= = in the system increases.
Green!Yellow!Empty! 2000!4000!4000!

10000!
S = kB ln
2000!4000!4000!
= kB ln((10000 ln10000 – 10000) – (2000 ln 2000 – 2000 +
4000 ln 4000 – 4000 + 4000 ln 4000 – 4000)
= 10549.2 kB

25. The system in Problem 24 converts to a final state


with 6000 yellow molecules and no green molecules.
Assuming no energy difference between yellow
and green molecules, would this conversion occur
spontaneously?
Answer:
Spaces! 10000!
W= =
Yellow!Empty! 6000!4000!

10000!
S = kB ln
6000!4000!
= kB ln((10000 ln10000 – 10000) – (6000 ln 6000 – 6000 +
4000 ln 4000 – 4000)
= 6730.1 kB

The Molecules of Life by John Kuriyan, Boyana Konforti, and David Wemmer © Garland Science
The MOLECULES
of LIFE
Physical and Chemical Principles

Solutions Manual

Prepared by James Fraser and Samuel Leachman


Chapter 8
Linking Energy and Entropy:
The Boltzmann Distribution

Problems

True/False and Multiple Choice

1. For a system converting from state 1 to state 2: Fill in the Blank


(kBlnW2 − kBlnW1) = qrev/T.
8. The Boltzmann distribution describes the energy of
True/False
molecules at _________________.
2. If the second-lowest energy level is separated from the Answer: equilibrium
ground state by 0.5kBT, then the second-lowest energy
level will not be occupied appreciably. 9. The Boltzmann constant is the gas constant (R) divided
True/False by ____________ .
Answer: Avogadro’s number
3. There are many equivalent microstates corresponding
to a particular energy distribution. 10. Kinetic energy is due to the __________ of atoms.
True/False Potential energy is due to the ___________ of atoms.
Answer: motion, configuration/relative positions
4. Which of the following definitions of entropy are
equivalent for a large system: 11. The direction of spontaneous change is governed by an
a. probabilistic definition increase in ____________, thus explaining how energy
b. thermodynamic definition can be transferred “uphill,” that is from a system of
c. statistical definition lower total energy to one with higher total energy.
d. all of the above Answer: entropy/multiplicity
5. The probabilistic definition of entropy is more accurate 12. If two systems of different temperatures are brought
than the statistical definition for small systems. together, there will be a net transfer of heat from the
True/False system of __________ temperature to the system of
_________ temperature.
6. After spontaneous heat transfer between systems, the
overall multiplicity is: Answer: higher, lower

a. lower than it was before the transfer of heat


b. zero Quantitative/Essay
c. maximized
d. minimized Assume kB = 1 J•K–1 for all problems.
7. The partition function (Q) is needed to describe: 13. A system starts with a multiplicity of 2000. Two kJ of
a. The volume of two gases on either side of a heat are transferred into the system reversibly at 298 K.
partition. What is the multiplicity now?
b. The extent to which many different energy Answer:
levels are occupied. Initial entropy = kB ln W = 1 × ln 2000 = 7.6 J•K–1
c. How the statistical definition of entropy is Change in entropy = 2000 J/298 K = 6.7 J•K–1
equivalent to the probabilistic definition of entropy. Final entropy = 14.3 J•K–1
d. The relationship between work and pressure. Final multiplicity = eS/kB = 1.64 × 106

The Molecules of Life by John Kuriyan, Boyana Konforti, and David Wemmer © Garland Science
2 Chapter 8: Linking Energy and Entropy: The Boltzmann Distribution

14. A system starts with a multiplicity of 1099. Heat is trans- 18. Consider the system and the two states from Problem
ferred out of the system reversibly at 298 K such that 17. Which state has greater multiplicity?
the final multiplicity is 105. How much heat was lost? Answer:
Answer: For State A:
Wfinal 105 W = 5!/(1!3!1!) = 20
∆S = kB ln = kB ln = –216 J•K–1
Winitial 1099 For State B:
qrev = T∆S = (298 K)(–216 J•K–1) = –64.5 kJ W = 5!/(2!3!) = 10
Therefore, system A has greater multiplicity.
64.5 kJ of heat are lost.
19. A system has 100,000 molecules at energy level 1,
10,000 molecules at energy level 2, and 1000 molecules
15. A system of noninteracting atoms has a positional
at energy level 3. What is the entropy of the system?
multiplicity of 120 and an energetic multiplicity of 5.
(Hint: Use the probabilistic definition.)
What is the total multiplicity? What are the positional,
energetic, and total entropies of the system? Answer:
Total molecules = Nt =111,000
Answer:
Probability of level 1 = N1/Nt = 100,000/111,000 = 0.90
Total multiplicity = positional multiplicity × energetic
Probability of level 2 = N2/Nt = 10,000/111,000 = 0.09
multiplicity = 120 × 5 = 600
Probability of level 3 = N3/Nt = 1000/111,000 = 0.009
Positional entropy = kB ln 5 = 1.6 J•K–1; energetic =
Entropy = –NtkB Σpi ln pi = –111,000 J•K–1 × (0.90 ln 0.90
kB ln 120 = 4.8 J•K–1
+ 0.09 ln 0.09 + 0.009 ln 0.009)
Total entropy = positional + energetic entropy =
= –111,000 J•K–1 × (–0.094 – 0.217 – 0.042) =
1.6 + 4.8 = 6.4 J•K–1
–111 kJ•K–1 × (–0.353) = 39 kJ•K–1
16. A system with 1 mole of an ideal gas changes its
20. A system with 100,000 molecules has two energy levels
volume from 10 L to 1 L by isothermal compression.
(A and B). At first, the two energy levels are populated
What is the change in entropy of the system? What is
equally. After a reversible process, energy level A is
the change in the entropy associated with the energy
populated by 65% of the molecules and the system
distribution? (Use R = 8.31 J•K–1•mol–1.) is at 293K.
Answer: a. What is the difference in energy between the two
The configurational entropy change is given by: levels?
∆S = nR ln (V2/V1) = 1 mol × 8.31 J•K–1•mol–1 × b. How much heat was added or removed from the
ln (1 L/10 L) = –19.1 J•K–1 system?
For the isothermal expansion of an ideal gas there is c. What is the change in entropy?
no change in energy. Thus, the energy distribution is
Answer:
unchanged by the process, and the associated entropy
a. P(B)  –∆G 
is also unchanged.
= exp  
17. Consider a system with five molecules and three energy P(A)  RT 
levels. The energy levels are such that a molecule at
energy level 1 contributes 1 J of energy to UTOTAL, a 0.35  –∆G 
= exp  
molecule at energy level 2 contributes 2 J of energy to 0.65  (293 K)(8.31 J•K–1•mol–1)
UTOTAL, and a molecule in energy level 3 contributes
3 J of energy to UTOTAL. Which has more internal energy, ⇒ ∆G = 1.5 kJ•mol–1
problems,
state Amultiple
or state choice,
B? Q17 (eng_ent_34_v1)
b. Initial entropy = –NtkB Σpi ln pi = –100,000 J•K–1 ×
state A state B (2 × 0.5 ln 0.5) = 69.3 kJ•K–1
Final entropy = –100,000J•K–1 × (0.65 ln 0.65+0.35 ln
3 3 0.35) = 64.7 kJ•K–1
2 2 The change in entropy (∆S) is –4.57 kJ•K–1
1 1 ∆S = qrev/T; qrev = ∆S × T = –4.57 kJ•K–1 × 293 K = –1339 kJ
c. See above.
As a check, we can verify that heat is removed from the
Answer:
system because the entropy of the system decreases
For State A during this process.
UTOTAL = ΣNiUi = 3 × N3 + 2 × N2 + 1 × N1
= 3 × 1 + 2 × 3 + 1 × 1 = 10 21. Two systems, A and B, are placed in thermal contact
and isolated from the rest of the world. Shown next are
For State B
three possible microstates for the combined system.
UTOTAL = ΣNiUi = 3 × N3 + 2 × N2 + 1 × N1
Two of these are valid microstates for the combined
=3×0+2×2+1×3=7
system, and one is not. Identify the inconsistent
Therefore, system A has greater energy. microstate and explain why it does not belong with the
other two. (Hint: Consider Figure 8.15.)

The Molecules of Life by John Kuriyan, Boyana Konforti, and David Wemmer © Garland Science
8Q22
PROBLEMS and solutions 3
problems, calculations, Q21 (eng_ent_35_v1)

microstate 1
system A system B max
5
4
energy

log (W )
3
2
1
0

microstate 2
system A system B energy
5 23. What is the entropy of the following energy distribution?
4
energy

What is the multiplicity?


3
2 N3 = 200 molecules
1 N2 = 1100 molecules
0 N1 = 10,500 molecules
Answer:
microstate 3
We first compute the entropy using the probabilistic
system A system B definition.
5 Initial entropy = –NtkB  Σpi ln pi = –11,800 J•K–1 × (0.017
4
ln 0.017 + 0.093 ln 0.093 + 0.89 ln 0.89) = –11,800 J•K–1
energy

3
2
× (–0.394) = 4651 J•K–1
1 S = kB ln W; W = eS/kB = e4651 ~ 102020.
0 24. A system has a partition function of 1.3 at 293 K. At that
temperature, an energy level is populated by 10% of the
Answer: molecules. What is the energy of that level?
Microstate 3 is inconsistent with the other two. Because Answer:
the two systems are isolated from the rest of the world, e–u/(kBT) = p × Q;
the total energy must be observed. Microstates 1 and 2 U = –kB × T × ln (p × Q) = –1 J•K–1 × 293 K × ln (0.1 × 1.3)
have the same total energy, but microstate 3 does not. = 598 J relative to ground state.
22. A system has two types of molecules (black and gray) 25. Explain how energy can move spontaneously from a
that interconvert. The log of the multiplicity of energy of system of low total energy to a system of higher total
the two types of molecules individually versus the total energy.
system energy is plotted below. Plot the log of the total
Answer:
multiplicity of energy and indicate where the maximum
problems, Spontaneous processes are governed by an increase
value forcalculations,
the combined Q10 (eng_ent_36_v1)
multiplicity occurs.
in universal entropy. If moving energy to the system
of higher total energy increases its entropy more than
it decreases the entropy of the system with low total
energy, the change will be spontaneous.
26. Why is it more common to think of temperature as
log (W )

being related to the rate of change of energy with


respect to entropy than it is to think of it as being
related to the rate of change of entropy with respect
to energy?
Answer:
While both definitions are viable, it is much more
energy intuitive to use the former definition because it
makes the direction of heat flow from higher to
Answer: lower temperatures, and because it makes all typical
Because the log of the multiplicity is additive, the log temperatures much greater than 1, rather than fractions
of the total multiplicity is the sum of the black and gray much less than 1. For example, it makes more sense to
lines. The total maximum is the same as the maximum say 37° is hotter than 25° than it does to say 0.003224
value for the gray molecule, because after that point, is hotter than 0.003354.
the log(W) of the gray molecule decreases faster than
the log(W) of the black molecule increases.

The Molecules of Life by John Kuriyan, Boyana Konforti, and David Wemmer © Garland Science
The MOLECULES
of LIFE
Physical and Chemical Principles

Solutions Manual

Prepared by James Fraser and Samuel Leachman


Chapter 9

Free Energy

Problems

True/False and Multiple Choice

1. For a system at equilibrium, Gibbs free energy is b. The free energy of the system is minimized.
maximized. c. The combined entropy of the system and
surroundings is maximized.
True/False
d. Only b and c.
2. The work done by biological systems is most commonly e. a, b, and c.
which type of work?
Fill in the Blank
a. pressure
b. chemical 7. Gibbs free energy is used to describe systems
c. home with constant ___________, while Helmholtz free
d. volume energy is used to describe systems with constant
e. heat _______________.
3. According to Le Châtelier’s principle: Answer: pressure and temperature, volume and
a. A reaction always favors the formation of as much temperature
product as possible.
8. The standard state has a pressure of ___________.
b. Reactants will only react upon the addition of
external heat or pressure. Answer: 1 atm
c. The system will adjust, if more reactants are
9. The change in free energy of a process is equal to the
added, and a new equilibrium will be established to
________________ amount of work extracted from that
balance the change.
process.
d. The Gibbs free energy is always greater than the
amount of work done plus the Helmholtz free energy. Answer: maximum
4. The value of ∆f Go for elemental oxygen is: 10. In biochemistry, the standard state for water is ____ M.
a. –15 kJ•mol–1. Excepting H+, for all other solutions, the standard state
b. 0 kJ•mol–1. is ______ M.
c. Much larger than the free energy of elemental Answer: 55, 1
hydrogen and much less than the free energy of
elemental uranium. Quantitative/Essay Problems
d. Equal to the free energy of elemental tungsten.
e. Both b and d. 11. A system is at constant volume and 29°C. The internal
energy is –40 kJ and the entropy is 31 J•K–1. What is the
5. The cell generates ATP from ADP and Pi by coupling the
value of the Helmholz free energy?
synthesis to an energetically unfavorable process.
Answer:
True/False
First convert 29°C to K by adding 273.
6. At equilibrium:
A = U – TS = –40 kJ – (302 K × 31 J•K–1) =
a. There is no change in temperature over time. –40 kJ – 9.4 kJ = –49.4 kJ

The Molecules of Life by John Kuriyan, Boyana Konforti, and David Wemmer © Garland Science
2 Chapter 9: Free Energy

12. Consider the distribution of molecules in energy levels 16. What is the expression for the equilibrium constant for
in the diagram below, where each “X” represents one the reaction A B + C?
molecule. A molecule in energy level 1 contributes one
Answer:
unit of energy to the total internal energy, a molecule
[B][C]/[A]
in energy level 2 contributes two units of energy to the
total internal energy, etc. The system is at 273 K.
17. Consider the reaction A + 2B → C. Values of ∆f Go for A,
B, and C are as follows:
Energy level 4 X
A = –34 kJ•mol–1
Energy level 3 XX
B = 84 kJ•mol–1
Energy level 2 XXXXX X C = –112 kJ•mol–1
Energy level 1 XXXXX XXXXX XXXXX What is the value of ∆Go for the reaction?
Answer:
What are the values of the (a) internal energy (U), (b) ∆G = ∆f Gproducts – ∆f Greactants = –112 kJ•mol–1 –
entropy (S), and (c) Helmholtz free energy (A) of the (–34 kJ•mol–1 + 2 × 84 kJ•mol–1) = –246 kJ•mol–1
system? Assume that kB = 1 energy unit•K–1.
18. Consider the following reactions in which A, B, C, and D
Answer: are elements in their most stable states:

a. Utotal = ΣNiUi = 1 × 4 + 2 × 3 + 6 × 2 + 15 × 1 =
A+B→Y
4 + 6 + 12 + 15 = 37 units
C+D→Z

b. W = (Total)!/(Energy level 4)! × (Energy level 3)! ...
= 24!/(1!2!6!15!) = 329,491,008 Y+Z→J
S = kB ln W = 19.6 units•K–1 Using the following table of ∆Go values, what is the
c. A = U – TS = 37 units – 273 K × 19.6 units•K–1 = value of ∆f Go for the formation for J?
–5317 units
Reaction ∆Go (kJ•mol–1)
13. A system at 275 K in state A has an enthalpy of –25 kJ
A+B→Y –70
and an entropy of 2 J•K–1. In state B, it has an enthalpy
of –20 kJ and an entropy of 10 J•K–1. Will state A convert C+D→Z 23
spontaneously to state B?
Y+Z→J 15
Answer:
GA = H – TS = –25 kJ – 275 K × 2 J•K–1 = –25.55 kJ
Answer:
GB = H – TS = –20 kJ – 275 K × 10 J•K–1 = –22.75 kJ
∆f G(Y) – (∆f G(A) + ∆f G(B)) = –70 kJ•mol–1 → ∆f G(Y) =
Since the free energy is lower in state A, it will not
–70 kJ•mol–1
convert spontaneously to state B at 275 K.
∆f G(Z) – (∆f G(C) + ∆f G(D)) = 23 kJ•mol–1 → ∆f G(Z) =
14. Assume that entropy and enthalpy changes are 23 kJ•mol–1
independent of temperature. A system in state A has an ∆f G(J) – (∆f G(Y) + ∆f G(Z)) = 15 kJ•mol–1
enthalpy of –22 kJ and an entropy of 7 J•K–1. In state B, ∆f G(J) = 15 kJ•mol–1 – 70 kJ•mol–1 + 23 kJ•mol–1 =
it has an enthalpy of –12 kJ and an entropy of 15 J•K–1. –32 kJ•mol–1
At what temperatures will state B be favored?
19. Consider the reaction, 2I + J → K, and the following
Answer:
table of enthalpies and entropies of formation at 298 K:
The state B will be favored when GB < GA
Therefore: I J K
HB – TSB < HA – TSA
(HB – HA)/(SB – SA) < T (because SB – SA > 0) ∆f Ho (kJ•mol–1) 0 –30 –300
(– 12,000 J + 22,000 J)/(15 J•K–1 – 7 J•K–1) = 1250 K ∆f So (J•K–1•mol–1) 120 200 60
1250 K < T. The temperature must be greater than
1250 K. What is the standard free-energy change (∆Go) for the
reaction?
15. A reaction has an enthalpy change of 200 kJ and an
entropy change of 250 J•K–1. Assume that entropy and Answer:
enthalpy changes are independent of temperature. At ∆G = ∆f G(K) – [2 × ∆f G(I) + ∆f G(J)]
what temperature will the Gibbs free energy be zero? = –300 kJ – 298 K × 60 J•K–1 – [2 × (0 – 298 K × 120
J•K–1) + (–30 kJ – 298 K × 200 J•K–1)]
Answer: = –317.9 kJ – (–71.5 kJ – 89.6 kJ) = –156.8 kJ
G = H – TS, G = 0. H = TS
200 kJ = T(250 J•K–1) 20. Consider the same reaction as in Problem 19. Assume
T = 200 kJ/(250 J•K–1) = 800 K that enthalpy and entropy do not vary with temperature.

The Molecules of Life by John Kuriyan, Boyana Konforti, and David Wemmer © Garland Science
PROBLEMS and solutions 3

At what temperature will the reaction begin to proceed 23. A molecular motor moves along a microtubule track
spontaneously in the opposite direction (that is, at what in steps of 100 Å displacements. The motor hydrolyzes
temperature will ∆Goreactants < ∆Goproducts)? one molecule of ATP per step. The motor operates
under conditions where the free-energy change for
Answer: ATP hydrolysis is –60 kJ•mol–1. What is the maximum
At 298 K, the reaction proceeds to products in Problem resistive force against which the motor can move
19. To proceed towards reactants: cargo?
∆Greactants < ∆Gproducts
0 < ∆Gproducts – ∆Greactants Answer:
0 < ∆f Hproducts – T∆f Sproducts – (∆f Hreactants – T∆f Sreactants) w 60000 J•mol–1 1 mol
0 < –300,000 J – T × 60 J•K–1 + 240 J•K–1 × T + 30,000 J F= = × = 9.96 × 10–12 N = 10 pN
x 100 × 10–10 m 6.022 × 1023
+ 200 J•K–1 × T
0 < –270,000 J + 380 J•K–1 × T
24. Explain why the absolute value of the work done by a
710.5 K < T
process at constant pressure can never be greater than
Therefore the temperature must be at least 710.5 K.
the absolute value of the Gibbs free-energy change for
21. The value of ∆Go for forming water from elemental the process.
oxygen and hydrogen is –237 kJ•mol–1.
Answer:
2H2 +O2 → 2H2O The maximum amount of work that can be performed
by a process at constant temperature and pressure
Calculate the standard enthalpy of formation of water
is the Gibbs free energy. It incorporates the minimum
using the entropies below. (Hint: Elemental oxygen and
energy that must necessarily be lost as heat. When the
hydrogen have the same enthalpy of formation.)
temperature isn’t held constant, additional energy is
O2 H2 H2O lost as heat, which cannot be used as work.

So (J•K–1•mol–1) 205 130 70 25. The equilibrium constant for a chemical system is
1. More reactants are added to the system. Explain
Answer: what will happen to the system as it reestablishes
Elemental oxygen and hydrogen both have an enthalpy equilibrium.
of formation = 0. Answer:
∆f G(H2O) = ∆f H(H2O) – T∆f S(H2O) If the equilibrium constant is 1, then there is always
∆f H(H2O) = ∆f G(H2O) – T(∆f S(H2O)) an even ratio of products to reactants. According to Le
∆f H(H2O) = ∆f G(H2O) – T(So(H2O) – So(H2) – ½ × So(O2)) Chatalier’s principle, as more reactants are added to the
∆f H(H2O) = –237 kJ•mol–1 + 298 K × (70 J•K–1•mol–1 – system, more product will form to equalize the ratio.
130 J•K–1•mol–1 – 205/2 J•K–1•mol–1)
∆f H(H2O) = –285 kJ•mol–1

22. A chemist wants to develop a fuel by converting water


back to elemental hydrogen and oxygen using coupled
ATP hydrolysis to drive the reaction. Given that the
value of ∆ f Go for water is –237 kJ•mol–1 and that one
mole of ATP hydrolyzed to ADP yields –30 kJ•mol–1, how
much ATP is needed to yield three moles of H2 gas?

Answer:
Reaction 1:
2H2+O2 → 2H2O
The ATP reaction:
2H2O + X ATP → 2H2 + O2 + ADP + phosphate
The molar ratio of water to hydrogen gas is 1:1,
therefore 3 moles of H2O will need to be converted.
∆Greaction1 = –711 kJ must be balanced by ATP
hydrolysis.
∆Greaction1 > ∆GreactionATP
–711 kJ > –30 kJ × (moles of ATP)
–711 kJ/–30 kJ < moles of ATP
23.7 < moles of ATP
Therefore at least 23.7 moles of ATP must be
hydrolyzed to yield 3 moles of hydrogen gas. This also
demonstrates how ATP hydrolysis can be used to drive
an otherwise unfavorable process.

The Molecules of Life by John Kuriyan, Boyana Konforti, and David Wemmer © Garland Science
The MOLECULES
of LIFE
Physical and Chemical Principles

Solutions Manual

Prepared by James Fraser and Samuel Leachman


Chapter 10
Chemical Potential and
the Drive to Equilibrium

Problems

True/False and Multiple Choice

1. Molecules move spontaneously from regions of low 7. During protein folding, the entropy of water:
chemical potential to regions of high concentration. a. increases
True/False b. decreases
c. is equal to the protein entropy change
2. The difference in chemical potential for a region d. is zero
with 500 mM of molecule B and a region with 1 M of
molecule B is equal to: Fill in the Blank
a. kBT ln(0.5)
b. 0 8. A region with a high chemical potential for molecule A
c. 1 has a _______ concentration of molecule A than a region
d. kBT ln(500) with low chemical potential.
e. kBT ln(1) Answer: greater/higher
3. The proton concentration in pure water at standard 9. To make a buffer, add a weak acid to its conjugate
state (298 K) is: _______.
a. equal to 14 Answer: base
b. always less than –7
c. the square root of the ion product 10. _____, ______, and ______ sidechains generally have
d. 107 a pKa less than 7.0. _______, ________, and ______
e. 81 kJ•mol–1 sidechains have a pKa greater than 9.
Answer: His, Asp, Glu; and Tyr, Lys, Arg
4. The melting temperature (TM) is the temperature at
which 100% of the protein molecules are unfolded. 11. The integral of the melting curve of heat capacity versus
True/False temperature yields the _____________ change of protein
unfolding.
5. Which of the following must be independent of
temperature when properly applying the van’t Hoff Answer: enthalpy change
equation?
12. The _______________ change for a reaction determines
a. Keq
the direction of spontaneous change.
b. ∆So
c. 1/T Answer: free energy or ∆G
d. pH
Quantitative/Essay
6. The pKa of a protein sidechain depends only on
the chemical identity of the sidechain, not on the 13. Two regions of an ideal dilute solution have a difference
surrounding environment. in concentration of potassium ions (K+). At 293 K, what
True/False is the difference in chemical potential between region

The Molecules of Life by John Kuriyan, Boyana Konforti, and David Wemmer © Garland Science
2 Chapter 10: Chemical Potential and the Drive to Equilibrium

1, with a concentration of 0.5 M K+, and region 2, which Answer:


has a concentration of 2 mM? a. Q = [C]/([A][B]2) = 0.5/(0.01 × 0.22) = 1250
Answer: b. The reaction will proceed towards the right
since Q < Keq.
C2
∆µ = RT ln 18. The arginine-tRNA synthetase enzyme catalyzes the
C1
reaction that charges a tRNA with the amino acid
0.002 M arginine:
= 8.31 J•K–1•mol–1 × 293 K × ln
0.5 M
ATP + arginine + tRNA AMP + PPi + arginyl-tRNA
= –13.5 kJ•mol–1
The value of the equilibrium constant is 1.13.
14. The difference in chemical potential for a particular At equilibrium, following an in vitro reaction, the
molecule between two regions of an ideal dilute concentration of ATP is 2 µM, of arginine is 500 mM, and
of arginyl-tRNA is 10 µM. The concentrations of AMP
solution is 5 kJ•mol–1. The region with the higher
and of PPi are 500 µM. What is the concentration of
chemical potential has a concentration of 200 mM.
arginyl-tRNA?
What is the concentration of the molecule in the other
region at 293 K? Answer:
Answer: [500 × 10–6] [500 × 10–6] [RtRNA*]
K = 1.13 =
C2 [2 × 10–6] [500 × 10–3] [10 × 10–6]
∆µ = RT ln
C1 [RtRNA*] = 4.52 × 10–5 = 45.2 µM
0.2 M
5000 J•mol–1 = 8.31 J•K–1•mol–1 × 293 K × ln
x 19. The pKa of a weak acid is 5. What is the pH when
the concentration of the acid form is 0.5 M and the
0.2 M concentration of the conjugate base form is 0.05 M?
ln = 2.05
x Answer:
pH = pKa + log10([base]/[acid]) = 5 + log10(0.05/0.5) =
0.2 M
= 7.79 5 + log10(0.1) = 4
x
20. The pH of a 0.15 M propionic acid/0.1 M sodium
x = 25.7 mM
propionate buffer is 4.71. What is the pKa of propionic
acid?
15. A cell with an internal calcium ion (Ca2+) concentration
of 20 µM is placed in media with a Ca2+ concentration Answer:
of 70 mM. What is the difference in chemical potential pKa = pH – log([base]/[acid]) = 4.71 – log(0.1/0.15) =
for Ca2+ ions between the inside and outside of the cell 4.71 + 0.18 = 4.89
at 310 K? 21. Consider a protein with a surface-exposed histidine
Answer: residue in a pH 4 solution. What is the fraction of protein
molecules in which this histidine residue is charged?
Cin (Assume that the pKa is 6.0.)
∆µ = µin – µout = RT ln
Cout
Answer:
20 × 10–6 M [His]/[His+] = 10(pH – pKa) = 10(4 – 6) = 10–2 = 0.01
∆µ = 8.31 J•K–1•mol–1 × 310 K × ln Fraction[His] = [His]/[His+] / ([His]/[His+] + 1) = 0.9901
70 × 10–3 M
Therefore, 99% of the sidechains will be charged.
∆µ = 21.0 kJ•mol–1 22. For a protein with a surface-exposed aspartic acid,
at what pH will this residue be neutral in 75% of the
16. At equilibrium, in a test tube, the concentration of GDP protein molecules? (Assume that the pKa is 4.0.)
is 1 M, of GTP is 20 µM, and of Pi is 1 M. What is the
Answer:
equilibrium constant of the reaction, GTP GDP + Pi?
[Asp–]/[Asp] = 0.25/0.75 = 1/3
Answer: pH = pKa + log(1/3)
[1][1] pH = 4 – 0.48
K= = 50000 pH = 3.52
[20 × 10–6]
23. A histidine is involved in an interaction with a glutamic
17. The reaction, A + 2B C, has an equilibrium constant of acid that stabilizes the charged form of the histidine,
2000. During a reaction, the concentration of A is 0.01 such that the value of ∆Go for deprotonation is 15
M, of B is 0.2 M, and of C is 0.5 M. kJ•mol–1 at pH 7.0 and 293 K (calculated using the
a. What is the reaction quotient (Q)? biochemical standard state). What is the pKa of this
b. In what direction will the reaction proceed? histidine?

The Molecules of Life by John Kuriyan, Boyana Konforti, and David Wemmer © Garland Science
Answer: 27. In the hydrophobic core of a folded protein, there are
[His]/[His+] = e(–∆G/RT) = e(–15,000/(293 × 8.31)) = 0.002118 three alanine and five phenylalanine residues that are
pKa = pH – log([His]/[His+]) = 7 – log10(0.002118) = 9.67 buried, and do not interact with water. Assume:
24. At the TM of a protein (55°C), the value of ∆Hounfolding is • In solution, waters can take on seven energetically
15 kJ•mol–1. What is the value of ∆Sounfolding? equal states.
Answer: • Two waters are ordered around each alanine in the
∆Sounfolding = ∆Hounfolding/TM = 15 kJ•mol–1/328 K = unfolded state.
45.7 J•mol–1•K–1 • Six waters are ordered around each phenylalanine
in the unfolded state.
25. A protein has a ∆Hunfolding value of 140 kJ•mol–1 at 25°C. • In the unfolded state, waters are ordered around
The value of ∆CP is 7.5 kJ•K–1•mol–1. The value alanine or phenylalanine residues and can take on only
of ∆Hounfolding at the TM is 230 kJ•mol–1. What is the two energetically equal states.
value of TM?
What is the difference in the entropy of the water due
Answer:
to the burying of these residues as this protein folds?
∆CP (298 K – TM) = ∆Hounfolding @ 25 – ∆Hounfolding @ TM
Answer:
298 K – TM = (∆Hounfolding @ 25 – ∆Hounfolding @ TM)/∆CP
Total water molecules = 2 × number of Ala +
TM = –[(∆Hounfolding @ 25 – ∆Hounfolding @ TM)/∆CP – 298 K]
6 × number of Phe
TM = 310 K or 37°C
=2×3+6×5
26. A lysine sidechain has four torsion angles, each of = 36 waters
which can take on three different values (60°, –60°, and Sfolded = R ln736 = 8.31 J•K–1•mol–1 × 36 × ln7 =
180°). Each unique combination of angles is called a 582 J•K–1•mol–1
rotamer. For example, a lysine residue where the first, Sunfolded = R ln236 = 8.31 J•K–1•mol–1 × 36 × ln2 =
second, third, and fourth torsion angles are all 60° is 207 J•K–1•mol–1
one unique rotamer, whereas a residue where the first, ∆So = Sfolded – Sunfolded = 375 J•K–1•mol–1
second, and third torsion angles are 60° and the fourth 28. Why do proteins denature at cold temperatures?
torsion angle is 180° is a second rotamer.
Answer:
In contrast, a serine sidechain has only one torsion While the physical mechanism behind cold denaturation
angle, which can take on three different values. is not yet understood, the phenomenon can be
Assume that all possible dihedral angles are allowed predicted from the curvature of protein stability curves.
at each angle for residues at this surface-exposed The constant curvature, arising from the difference in
position. heat capacity between the folded and unfolded states,
means that because ∆Go = 0 at the TM, it must also
a. What is the difference in molar entropy between a equal zero at some other point.
protein with a surface-exposed lysine and an otherwise
identical protein with a serine mutation at that position? 29. How do hydrophobic interactions provide favorable
b. Why might the simplification that each lysine entropy for protein folding?
torsion angle is able to adopt any of the three staggered Answer:
positions, independent of the conformation at other Since water molecules cannot hydrogen bond with
torsion angles, lead to an overestimate of the number of hydrophobic groups on the protein, the rotation of
low-energy conformations that lysine can adopt? water around these groups is restricted. When the
Answer: hydrophobic groups collapse into the core of the
a. protein, water no longer surrounds them. Thus, the
WK = 34 water that previously was restricted around the
hydrophobic groups is released to the bulk solvent and
WS = 31 free to move between many configurations (increasing
the entropy of the system).
WK 34
∆S = R ln = 8.31 J–1•K–1•mol–1 × ln
WS 31

∆S = 27.4 J–1•K–1•mol–1

b. Some rotamers might be disallowed because


they sterically clash with other residues, so not all
independent conformations of torsion angles might be
possible.

The Molecules of Life by John Kuriyan, Boyana Konforti, and David Wemmer © Garland Science
4 Chapter 10: Chemical Potential and the Drive to Equilibrium

The Molecules of Life by John Kuriyan, Boyana Konforti, and David Wemmer © Garland Science
The MOLECULES
of LIFE
Physical and Chemical Principles

Solutions Manual

Prepared by James Fraser and Samuel Leachman


Chapter 11

Voltages and Free Energy

Problems

True/False and Multiple Choice

1. An oxidative half-reaction involves the release of 7. Which of the following statements about the passive
electrons from a molecule. spread of a voltage perturbation in a neuron are true:
True/False i The voltage spike is not regenerated.
ii The potassium conductance of the membrane
2. Which of the following is a common consequence of a changes locally.
protein coordinating a metal: iii The sodium conductance of the membrane remains
a. The free energy of the reduced and oxidized form constant.
of the metal are altered. iv It only occurs in non-myelinated axons.
b. The metal is protected from some chemical
reactions. a. None of the above.
c. The protein undergoes a conformational change. b. All of the above.
d. It enables the transfer of electrons over long c. (i) and (iii).
distances. d. (i), (ii) and (iii).
e. All of the above. e. (ii) and (iv).
3. Standard reduction potentials in biochemistry
correspond to solutions with [H+] = 1 M. Fill in the Blank
True/False 8. The oxidized and reduced forms of a molecule are
4. Which of the following conformational changes is not known as a ____________.
thought to occur as a K+ channel closes due to a voltage Answer: redox couple
change:
9. The energy from _______ is used to produce reduced
a. Helix S6 becomes straighter.
compounds in photosynthesis.
b. Helix S4 moves downward.
c. Charged residues on helix S4 form new interactions. Answer: light
d. The selectivity filter changes size.
10. Sodium and potassium pumps hydrolyze ________ to
e. The C-terminus of helix S4 forms a 310 helix.
move ions in and out of the cell.
5. Most ion channels have little selectivity because ions Answer: ATP
are small relative to the size of the channel pore.
True/False 11. By decreasing the capacitance of the surface of the
axon, ______________ greatly facilitates the transmission
6. Which feature allows mitochondria and chloroplasts, of action potentials.
unique from other organelles, to maintain large proton Answer: myelination
gradients:
a. They contain membrane spanning proteins. 12. A _________________ can build up charge on two
b. They have a second internal membrane. conducting surfaces with an insulator sandwiched
c. There are only one of each per cell. between them.
d. Their membranes consists of a lipid bilayer. Answer: capacitor
The Molecules of Life by John Kuriyan, Boyana Konforti, and David Wemmer © Garland Science
2 Chapter 11: Voltages and Free Energy

Quantitative/Essay Answer:
a. The reaction is
13. How does phosphorylation contribute to increasing the NADPH → NADP+ + H+ + 2e–, E = 0.32
diversity of redox active molecules in the cell? 1/2 O2 + 2H+ + 2e– → H2O, E = 0.815
Answer: NADPH + H+ + 1/2 O2 → H2O + NADP+, E = 1.135
Organic compounds that undergo redox reactions can ΔG = –υFΔE = –2 × 96,500 C•mol–1 × 1.135 J•C–1
be phosphorylated on parts of the molecule that are = 219 kJ•mol–1
not important for the redox reaction. For example, ΔGwasted = ΔG – ΔGATP = 219 kJ – 30 kJ = 189 kJ
NAD+/NADH can be converted to NADP+/NADPH.
b. 189/30 = 6.3 more ATPs could be produced.
The phosphorylated and unphosphorylated forms of
the compound bind specifically to different subsets
18. Consider the structure of the potassium channel and
of cellular proteins, which increase the diversity of
answer the following questions.
potentially active redox molecules in the cell.
a. Given that the four K+ binding sites are roughly
14. A new microorganism is isolated from a lake and is isoenergetic, why are usually only two of these sites
placed into a solution of KCl. The voltage difference occupied at any given time?
across its membrane is measured at 120 mV. How much b. Based on your answer to (a), how does the “knock-
energy is required to move a proton from the negative on” model explain the very high conductance
side of the membrane to the positive side? of potassium channels?
Answer: c. What establishes the directionality of K+ ion flow
∆U = q∆E through a potassium channel?
= 1.602 × 10–19 C × 0.12 J•C–1 Answer:
= 1.922 × 10–20 J a. The binding sites are close enough such that
electrostatic repulsion of K+ ions in adjacent sites is
15. An electrochemical cell couples manganese and copper, highly unfavorable. In contrast, occupancy of alternating
each in the presence of its 2+ ion. binding sites (1 and 3, or 2 and 4) is favorable.
a. What is the combined redox reaction? b. Two ions cannot simultaneously occupy adjacent
b. What is the the value of ΔGo for the reaction? binding sites within the channel, due to electrostatic
Answer: repulsion. In the knockon model for ion movement,
a. Mn → Mn2+ + 2e–, E = 1.18 V (positive, the when an ion enters a vacant binding site within the
standard reaction is reversed) channel, it causes the ion in the next binding site
2e– + Cu2+ → Cu, E = 0.340 V (if occupied) to hop into the next available site. For
Cu2+ + Mn → Mn2+ + Cu, E = 1.52 V example, if ions are present initially in sites 2 and 4, the
b. ΔG = –υFΔE entry of an ion into site 1 at one end of the channel will
= –2 × 96,500 C•mol–1 × 1.52 J•C–1 cause the site 2 ion to move to site 3, and the site 4 ion
= –293,360 J to exit the channel at the other end.
= –293.4 kJ•mol–1 c. The directionality is established by the
electrochemical gradient. There is no directionality
16. A researcher assembles an electrochemical cell with preference for the channel itself.
silver and an unknown metal in a 1 M solution of its ion.
She measures the free energy of the combined reaction 19. A scientist measures the potential across the
under standard conditions to be –150 kJ•mol−1. What membrane of a cell. At room temperature, the pH
is the potential of the oxidative half reaction if one outside the cell is 7.4 and the pH inside the cell is 7.1.
electron is transferred by the unknown metal to silver What is the membrane potential for protons across the
under standard conditions? bilayer?
Answer:
Answer:
ΔE = –ΔG/υF = 150,000 J/96,500 C•mol–1 × 1 EH = RT/F ln ([H+]out/[H+]in) = 59mV × (–7.4 + 7.1) =
= 1.55 V
–17.7 mV
Eunknown = ΔE – EAg = 1.55 V – 0.8 V = 0.75 V

17. A new ATP-producing protein is discovered that couples 20. A squid axon is immersed in seawater during a
ATP production to the oxidation of NADPH by oxidative laboratory experiment, and the resting potential across
phosphorylation. Assume that the value of ΔGo for ATP the axonal membrane is −76 mV, at room temperature.
synthesis is 30 kJ•mol−1. If this protein only produces The concentrations of Na+ and K+ in the seawater and
1 molecule of ATP per reaction that consumes one inside the squid axon are given in the table below:
NADPH:
a. How much free energy is wasted, under standard [K+] (mM) [Na+] (mM)
conditions?
b. How many more ATP molecules could be created inside squid axon 150 30
by a perfectly efficient electron transport chain from seawater 5 200
one NADPH?

The Molecules of Life by John Kuriyan, Boyana Konforti, and David Wemmer © Garland Science
PROBLEMS and solutions 3

a. What are the membrane potentials for Na+ and K+ RK = C/t = 1 μF/cm2/0.12 sec = 120,000 ohm•cm–2
under these conditions? Next, find the current:
b. Do you expect the resting membrane potential to RK = EM/IM
be equal to the sum of the membrane potentials for Na+ IM = EM/RK = 0.1 V/120,000 ohm•cm–2
and K+? Explain your answer. = 8.33 × 10–7 C•sec–1•cm–2
Answer: Finally, solve for flow:
a. EK = 0.0257 V × ln ([K+]out/[K+]in) = 25.7mV × –3.4 = IM = flow × F × number of channels/NA
–87.4 mV Flow = IM × NA/F × number of channels
ENa = 0.0257 V × ln ([Na+]out/[Na+]in) = 25.7mV × 1.9 = = (8.33 × 10–7 C•sec–1•cm–2 × 6.022 × 1023)/(96,500 ×
48.8 mV 25,000)
b. The resting potential is influenced by the relative = 2.08 × 108 C•sec–1
conductance of the two ions across the membrane.
Since the membrane is more permeable to K+ than Na+, 24. Immediately prior to and subsequent to an action
the actual potential is weighted more heavily to the K+ potential, there is little conductance through sodium
potential (see also equation 11.76). channels; however, conductance is blocked through
different mechanisms before and after the action
21. A scientist grows a tiny synthetic nerve cell with a potential passes. Explain the structural mechanisms
surface area of 3.1 × 10−7 cm2. In its culture medium, underlying the closed states of the channel before and
the cell has a resting membrane potential of 75 mV. after the action potential passes.
During an action potential that depolarizes the cell to 0 Answer:
mV, 6.2 × 10−12 C of charge move across the membrane. Prior to the action potential, the voltage sensor is
What is the capacitance of the cell per unit area of the in the closed conformation, which is established by
cell? interactions of the positively charged residues on the
Answer: voltage sensor and the negative electric field on the
q = CV inside of the membrane. This causes the helices that
C = q/V = 6.2 pC/75 mV = (6.2 × 10–12 C)/(0.075 V) comprise the channel to be closed.
= 8.267 × 10–11 F After the action potential, the electric field is not
C = CAA as negative, so the voltage sensor is in the open
CA = C/A = 8.267 × 10–11 F/3.1 × 10–7 cm2 conformation, but flexible cytosolic inactivation
= 2.67 × 10–4 F•cm–1 = 267 μF•cm–2 domains bind to the base of the channel and prevent
ion conductance.
22. A cell has the intracellular and extracellular
concentrations of K+ and Na+ as shown below, at
room temperature. If the resting conductance of its K+
channels is 30 times greater than its Na+ channels, what
is the resting potential of the cell (assuming no other
ions contribute significantly)?

[K+] (mM) [Na+] (mM)

intracellular 200 15

extracellular 12 360

Answer:
E = (gNa/(gNa + gK)) × ENa + (gK/(gNa + gK)) × EK
= (1/(1 + 30)) × 0.0257 V × ln (360/15) + (30/(1+30)) ×
0.0257 V × ln (200/12)
= 2.6 mV – 70.0 mV
= –67.3 mV

23. A region of an axon has a time constant of 120 msec.


The capacitance of the membrane is 1 μF•cm−2 and
there are 25,000 potassium channels per cm2 of surface
area of the membrane. How many ions does a typical
potassium channel conduct per second when the
membrane potential is 100 mV?
Answer:
First solve the resistance:
t = RK × C

The Molecules of Life by John Kuriyan, Boyana Konforti, and David Wemmer © Garland Science
The MOLECULES
of LIFE
Physical and Chemical Principles

Solutions Manual

Prepared by James Fraser and Samuel Leachman


CHAPTER 12
Molecular Recognition: The
Thermodynamics of Binding

Problems

True/False and Multiple Choice

1. Which of the following is not a process governed by 5. A linear ligand binding curve indicates a simple binding
molecular recognition? equilibrium, whereas a hyperbolic binding curve
a. The fidelity of DNA replication. indicates that the system contains a receptor that
b. Passive diffusion. is allosteric or contains multiple binding sites with
c. Active transport. different affinities.
d. Translation by the ribosome. True/False
e. Transcription by RNA polymerase.
6. Some inhibitors of HIV reverse transcriptase are
2. Noncovalent interactions, such as ionic and hydrophobic nucleotide analogs that displace the natural nucleotide
interactions, are generally much weaker than covalent substrates in the active site.
interactions.
True/False
True/False
7. Which of the following would lead to a decrease in
3. A protein binds to the DNA sequence AAAAA with a
binding affinity?
10-nM affinity (that is, the value of the dissociation
constant is 10 nM). The same protein binds to the RNA a. Releasing more protein-bound water molecules
sequence AUAAUA with a 15-nM affinity and to a lipid upon binding.
with a 10-mM affinity. Which of the following statements b. Decreasing the number of rotational degrees of
is true: freedom of the ligand upon binding.
c. Adding more hydrophobic interactions to the
a. The protein binds to RNA and DNA with high
ligand–protein interface.
affinity, but low specificity.
d. Increasing the number of hydrogen bonds between
b. The protein has high specificity for lipids over RNA
the ligand and protein.
and DNA.
c. The protein binds with low affinity for RNA and
DNA, but high specificity. Fill in the Blank
d. The protein binds with low affinity and low
specificity, for all three targets. 8. A _______ is a small molecule that binds to a
macromolecule receptor.
4. The value of KD corresponds to:
a. The reciprocal of KA. Answer: ligand
b. The concentration of ligand at half saturation of
receptor. 9. In general, higher affinity is achieved by increasing the
∆ Go
c. e RT. __________ of a ligand, while increasing specificity relies
E <1><-><1t-> on increasing _______________.
e. All of the above. Answer: hydrophobicity, hydrogen bonds

The Molecules of Life by John Kuriyan, Boyana Konforti, and David Wemmer © Garland Science
2 CHAPTER 12: Molecular Recognition: The Thermodynamics of Binding

10. A typical drug has a dissociation constant for its 16. At 298 K a variant of the protein streptavidin binds to
receptor in the __________ to __________ range. biotin with a dissociation constant of 150 pM. What are
the values of KA and ∆Gbind
o
?
Answer: nanomolar, picomolar
Answer:
11. Because they bind to the same site as ATP, many kinase KA = 1/KD
inhibitors are __________ with respect to ATP. = 1/(1.5 × 10–10)
Answer: competitive = 6.7 × 109
∆Gobinding = RT ln(KD)
12. Without ___________, some drugs cannot enter or exit +t,–1tNPM–1 × 298 K × ln(1.5 × 10–10)
the active site of a kinase domain. oL+tNPM–1
Answer: breathing motions/conformational changes 17. Enough estrogen is dissolved into a solution containing
the estrogen receptor that the total concentration of
Quantitative/Essay estrogen is 3 nM. The value of KD for the interaction
between estrogen and its receptor is 5 nM. What
13. The binding between the drug cyclosporin and the fraction of the protein is bound to estrogen? Assume
protein cyclophilin is measured, giving a KD value of that the concentration of estrogen is much greater than
1.5 nM. What is the the value of ∆Gbind at 298 K?
o
that of the protein.

Answer: Answer:
∆Gobinding = RT ln(KD) f = [L]/([L] + KD)
+t,–1tNPM–1 × 298 K × ln(1.5 × 10–9) = 3/(3 + 5)
oL+tNPM–1 = 0.375
37.5 percent of the protein will be bound to estrogen.
14. Shown below is a binding isotherm for an enzyme called
a phosphatase and a lead compound known as JF99. 18. A binding assay is performed with a DNA-binding
There is a tryptophan near the binding site, and the protein and a small DNA duplex. When the
fluorescence from this residue increases upon binding concentration of the free DNA duplex is 10 nM, 8%
the compound (shown on the y axis). At the highest of the protein is bound to DNA. Assume the protein
concentration of JF99 tested (1 μM), the fluorescence concentration is much lower than that of DNA.
signal is 490 units (not shown on the graph). Estimate What is the value of KD for the interaction? What
the value of KD. are the units of KD? Explain your answer.
400 Answer:
fluorescence signal (arbitrary units)

KD = [L]/f – [L]
350
= 10 nM/0.08 – 10 nM
300 = 125 nM – 10 nM
= 115 nM
250
200 19. Using a purified sample of the protein cyclophilin, the
Scatchard plot shown below is obtained when a form of
150 cyclosporin is added. What is the value of KD?
100
0.6
[bound cyclosporin]/[free cyclosporin]

50
0 0.5
0 10 20 30 40 50 60
[JF99] (nM) 0.4
Answer:
At the KD the protein will have the half-maximal 0.3
fluorescent signal. Since the maximal signal is ~500,
we look for the ligand concentration that gives a signal 0.2
of ~250. The estimated KD is ~20 nM.
0.1
15. What is the value of ∆Gbind for the interaction between
o

the phosphatase and the inhibitor, based on the data 0


shown in previous question, at 298 K? 0 0.01 0.02 0.03 0.04 0.05 0.06
Answer: [bound cyclosporin] (nM)
If the KD is ~20 nM.
∆Gobinding = RT ln(KD) Answer:
+t,–1tNPM–1 × 298 K × ln(2 × 10–8) KD = –1/slope = –1/((0.5 – 0.1)/(0.01 – 0.05 nM)) =
oL+tNPM–1 100 pM

The Molecules of Life by John Kuriyan, Boyana Konforti, and David Wemmer © Garland Science
PROBLEMS AND SOLUTIONS 3

20. The drug jafrasitor (molecular weight 540 daltons) binds 23. The IC50 value for a drug called razundib for a Src kinase
the histone deacetylase enzyme Sir2 with a dissociation is 15 nM. The Src kinase is known to bind ATP with a
constant of 0.1 nM. What mass of jafrasitor should be dissociation constant of 0.05 mM. If the concentration
administered to a patient with a blood volume of 5.5 L of ATP in the cell is 0.75 mM, what is the value of KI for
such that Sir2 is at least 91% inhibited? razundib with respect to the Src kinase?
Answer: Answer:
From the “universal” binding curve, we can see that KI = IC50 × (KD/(KD + [L]))
approximately 91% saturation occurs at 10 × KD, which = 15 nM × (0.05/(0.8))
is 1 nM. = 0.94 nM
Mass = concentration × volume × molar mass
ONPMt-–1¨-¨HtNPM–1 24. An NMR analysis of the the drug fluvastatin indicates
= 2.97 × 10–6 g = 2.97 μg that it loses rotational and translational degrees
of freedom upon binding to HMG-CoA reductase.
21. How do the conformational changes during kinase Additionally, many internal bond rotations are restricted.
activation enable imatinib to distinguish between Abl Yet, an isothermal titration calorimetry experiment
and the hundreds of other related kinases in the cell? indicates that the binding reaction has a favorable
change in entropy. What is the likely source of the
Answer:
favorable binding entropy change?
When active, kinases are relatively similar structurally
especially in the ATP binding pocket. However, inactive Answer:
kinases are quite different from each other. Imatinib The binding surface for fluvastatin is a large
binds to the inactive conformation of Abl, which is hydrophobic cavity that traps many water molecules
distinct from the inactive conformations of most other when it is not occupied by fluvastatin. Fluvastatin
kinases. displaces these water molecules and the increases in
the rotational and translational degrees of freedom for
22. Using 1 mL of partially purified cell lysate, which the water molecules. This increase in entropy more
contains 1 mg of total protein, the following Scatchard than offsets the entropy loss of binding fluvastatin.
plot is obtained for the binding of the protein FKBP
(molecular weight 20,000 daltons) and the drug
rapamycin. What is the purity of the lysate with respect
to FKBP?
0.6
[bound rapamycin]/[free rapamycin]

0.5

0.4

0.3

0.2

0.1

0
0 0.002 0.004 0.006 0.008 0.01 0.012 0.014 0.016 0.018 0.02
[bound rapamycin] (nM)
Answer:
First calculate the concentration of FKBP.
KD = –1/Slope = –1/((0.5 – 0.1)/(0.01 – 0.018)) = 20 pM
When [L]bound = 0.01 nM, [L]bound/[L]bound = 0.5,
0.5 = –1/(0.020 nM) × 0.01 nM + [P]/20 pM
[P] = 1 × 20 pM = 20 pM
QNPMt-–1¨ HtNPM–1 = 4 × 10–7Ht-–1, which is
4 × 10–7PGNHtN-–1

The Molecules of Life by John Kuriyan, Boyana Konforti, and David Wemmer © Garland Science
The MOLECULES
of LIFE
Physical and Chemical Principles

Solutions Manual

Prepared by James Fraser and Samuel Leachman


Chapter 13
Specificity of Macromolecular
Recognition

Problems

True/False and Multiple Choice

1. The affinity of a macromolecular interaction reflects the 6. Interface residues that do not contribute greatly to
strength of an interaction for one receptor relative to all binding affinity are also generally unimportant for
other possible receptors. specificity.
True/False True/False

2. Which of the following is an attribute of the FGF–FGFR 7. Which of the following is not an important aspect of
family of interactions? protein–nucleic acid recognition?
a. Each FGF ligand has high specificity for an FGF a. Insertion of arginine sidechains into the minor
receptor. groove of the nucleic acid.
b. Because there are 18 FGF and 4 FGFR genes, there b. Conformational changes of the protein and nucleic
are 72 potential interactions. acid.
c. Signaling specificity is enhanced through c. Stacking interactions between serine
selective expression of only a few receptors per sidechains and nucleic acid bases.
tissue type. d. Stacking interactions between tyrosine sidechains
d. FGFRs are soluble serine/threonine kinases. and nucleic acid bases.
e. FGF proteins have highly diverse folds. e. Water-mediated hydrogen bonds between protein
and nucleic acid.
3. Specificity depends on the concentration of ligand,
whereas affinity is independent of ligand concentration.
Fill in the Blank
True/False

4. Most of the binding energy for SH2 domain–peptide 8. _____________ residues, identified by mutating residues
interactions is contributed by: to alanine, contribute a larger than expected energy to
a. Amino–aromatic interactions. the interaction affinity of human growth hormone with
b. The phosphorylation of the peptide tyrosine. human growth hormone receptor.
c. The amino acid in the peptide + 1 position. Answer: Hot spot
d. The amino acid in the peptide + 3 position.
9. The kinase Zap-70 contains two _______ domains that
5. Which of the following statements regarding interfacial bind to two __________ residues in its targets.
waters is true?
Answer: SH2, phosphotyrosine
a. They contribute favorably to the entropy of protein–
protein recognition. 10. Proteins distinguish double-helical RNA and DNA by
b. They help to optimize the packing at the differences in _______ shape.
contact surface.
c. They contribute only to affinity, but not to the Answer: groove
specificity of protein–protein recognition.
11. Protein–protein interfaces typically bury approximately
d. There are typically fewer water-mediated hydrogen
_____________ in surface area.
bonds than direct hydrogen bonds at protein–protein
interfaces. Answer: ~1000 Å
The Molecules of Life by John Kuriyan, Boyana Konforti, and David Wemmer © Garland Science
2 Chapter 13: Specificity of Macromolecular Recognition

12. Examining complexes of nucleic acids and their binding b. α = ([tetO]/(1 + (KD,TetO/[L]))/([nonspecific]/(1 +
proteins reveals a high _____________ in both shape and (KD,nonspecific/[L])))
charge. = ([1 × 10–9/(1 + (1 × 10–6/1 × 10–8)))/(5 × 10–2/(1 +
(1 × 10–5/1 × 10–8)))
Answer: complementarity
= 1.98 × 10–7
Quantitative/Essay c. The low specificity values are tuned through
selection to ensure that a switch in transcription can
(Assume T = 300 K and RT = 2.5 kJ•mol–1 for all questions.) be achieved. If the equilibrium constants were used
without considering competing nonspecific sites,
13. A single-molecule microscopy experiment is performed then the tetO sites would almost always be bound. In
on a slide containing 1000 molecules each of green, red, essence the nonspecific sites titrate the TetR away from
cyan, and yellow fluorescent proteins (GFP, RFP, CFP, the specific site allowing switch-like behavior despite
and YFP, respectively). The experiment measures the the low specificity values.
number of antibodies bound to each type of protein,
finding that 900 molecules of GFP, 10 of RFP, 1 of CFP, 16. A zinc finger protein is isolated from a yeast cell. The
and 3 of YFP are bound at 1 nM concentration of value of KD for its binding site is 3 µM. In the presence
antibody. of glucose, the protein dimerizes and recognizes an
a. What is the specificity of the antibody for GFP? inverted repeat binding site.
b. What is the KD of the GFP–antibody interaction? a. What is the expected value of KD if the binding is
Answer: additive?
a. α = [RGFP•L]/([RRFP•L] + [RCFP•L] + [RGFP•L]) b. The dimeric KD is measured at 5 nM. Why does this
= 900/(10 + 1 + 3) = 64.3 value deviate from the expected KD?
b. KD = [RGFP] × [L]/[RGFP•L] Answer:
= 100 × 10–9 M/900 a. KD,dimer = (KD,monomer)2
= 111 pM = (3 × 10–6)
= 9 × 10–12
14. A scientist wants to engineer an antibody to distinguish = 9 pM
between two proteins (Cyclophilin A and Cyclophilin b. The simple additivity of free energy is probably
B) with a specificity of 500 at 1 nM concentration for reduced because energy is used to induce
each protein. Her starting material is an antibody that conformational changes in the DNA and protein to
binds with 10 nM KD to both proteins. She finds that
stabilize the dimer.
she can easily make mutations that decrease the
affinity for Cyclophilin B without affecting the affinity for 17. A tryptophan residue near the periphery of a protein–
Cyclophilin A. When she achieves the desired specificity, protein interface is mutated to alanine and changes the
what is the KD for Cyclophilin B? KD of binding from 1 nM to 40 µM at 300 K.
Answer: a. How much binding energy was contributed by that
α = (1/(1 + KD,CypA/[L]))/(1/(1 + KD,CypB/[L])) residue?
KD,CypB = α × ([L] + KD,CypA) – [L] b. Explain whether or not the tryptophan residue is
KD,CypB = 5.5 μM likely a hot spot residue.
Answer:
15. A tetracycline repressor (TetR) protein, which is
a. ∆∆G = ∆GTrp – ∆GAla = RT ln(KD,Trp) – RT ln(KD,Ala)
present in E. coli at 10–8 M concentration, binds to
the tetO site with a KD of 10–10 in the absence of the = RT ln(KD,Trp/KD,Ala) = RT ln(10–9/4 × 10–5) = –27 kJ•mol–1
drug tetracycline and a KD of 10–6 in the presence of b. Non-hotspot residues typically contribute
tetracycline. There is one tetO site in the E. coli genome, 0–5 kJ•mol–1 of binding energy, whereas hotspot
giving a concentration of 10–9 M per cell. There are residues often contribute more than 20 kJ•mol–1. Since
2 × 105 nonmatching sites per cell (a concentration of the Trp contributes 27 kJ•mol–1 it is likely a hotspot
5 × 10–2 M), to which TetR binds nonspecifically with a residue.
KD of 10–5.
18. A protein–protein interface has a 10 nM affinity at
a. What is the specificity for the tetO site over the
nonmatching sites when no tetracycline is present? 300 K. A series of mutants are made in which each
b. What is the specificity for the tetO site over the residue at the interface is replaced by alanine. A lysine
nonmatching sites when tetracycline is present? residue at the center of the interface is mutated, and
c. How can TetR repressors switch transcription with found to contribute 4 kJ•mol–1 to the binding free
such low specificity values? energy.
Answer: a. What is the new KD?
a. α = ([tetO]/(1 + (KD,TetO/[L]))/([nonspecific]/(1 + b. Explain whether or not the lysine residue is a hot
(KD,nonspecific/[L]))) spot residue.
= (1 × 10–9/(1 + (1 × 10–10/1 × 10–8)))/(5 × 10–2 /(1 + (1 × Answer:
10–5/1 × 10–8))) a. ∆∆G = –4 kJ•mol–1 = ∆GLys – ∆GAla = RT ln(KD,Lys) –
= 1.98 × 10–5 RT ln(KD,Ala)

The Molecules of Life by John Kuriyan, Boyana Konforti, and David Wemmer © Garland Science
PROBLEMS and solutions 3

RT ln(KD,Ala) = RT ln(KD,Lys) – 4 kJ•mol–1 = RT ln(10–8) + 4 b. Insertion of loop residues that change the relative
kJ•mol–1 = –42 kJ•mol–1 spacing of helix A and helix B?
KD,Ala = e(–16.8) = 5.0 × 10–8 = 50 nM Answer:
b. Since the Lysine residue contributes only a. The binding affinity of either complex would likely
4 kJ•mol–1 of binding energy, it is likely not a hotspot be reduced by mutating an Arg to an Ala. The positively
residue. charged Arg residue can make ionic contacts to the
negatively charged phosphate backbone. This mutation
19. The transcription factor FraJ binds a poly-A DNA
would likely have a negligible effect on specificity
sequence with a 10 nM KD and a poly-G DNA sequence
between RNA and DNA.
with a 27 µM KD. Mutation of a critical Phe residue
b. The dsRB domain recognizes the distinct
to Ala results in a loss of 20 kJ•mol–1 in binding free
groove spacing between RNA and DNA. A mutation
energy for the poly-A sequence, but only a loss of
that changed that spacing would definitely have
4 kJ•mol–1 on binding to the poly-G sequence. What is
a detrimental effect on RNA recognition. If the
the change in specificity for the poly-A sequence over
mutation changed the spacing so that it was more
the poly-G sequence at 10–8 M concentration of FraJ
complementary to DNA grooves, it would switch the
at 300 K?
specificity dramatically.
Answer:
For poly-A: 22. A DNA-binding domain binds the sequence
∆∆G = –20 kJ•mol–1 = ∆GPhe – ∆GAla = RT ln(KD,Phe) – GATCGCAATATCGATCGATC with a 25 nM affinity. A
RT ln(KD,Ala) mutation of an Arg to Ala in the protein or a mutation
–20/2.5 = ln(10–8) – ln(KD,Ala) of the underlined “T” to “G” in the DNA sequence
–8 = –18.4 – ln(KD,Ala) both result in a 9 kJ•mol–1 loss of binding free energy.
(KD,Ala) = e(∆G/RT) Simultaneous mutation of both the protein and the DNA
KD,Ala = e(–10.4) = 3.0 × 10–5 also results in a 9 kJ•mol–1 loss of binding free energy.
For poly-G: a. What is the effect on the KD of any of these
∆∆G = –4 kJ•mol–1 = ∆GPhe – ∆GAla = RT ln(KD,Phe) – mutations?
RT ln(KD,Ala) b. What does the double mutant result suggest about
–4/2.5 = ln(2.7 × 10–5) – ln(KD,Ala) the structural basis for the protein–DNA interaction?
–1.6 = –10.5 – ln(KD,Ala) Answer:
(KD,Ala) = e(∆G/RT) a. ∆∆G = –9 kJ•mol–1 = ∆GArg/G/Arg+G – ∆GAla =
KD,Ala = e(–8.9) = 1.3 × 10–4 RT ln(KD,Arg/G/Arg+G) – RT ln(KD,Ala)
αPhe = (1/(1 + KD,poly-A/[L]))/(1/(1 + KD,poly-G/[L])) –9/2.5 = ln(2.5 × 10–8) – ln(KD,Ala)
= (1/(1 + 10–8/10–8))/(1/(1 + 2.7 × 10–5/10–8)) –3.6 = –17.5 – ln(KD,Ala)
= 1350.5 KD,Ala = 9.1 × 10–7
αAla = (1/(1 + KD,poly-A/[L]))/(1/(1 + KD,poly-G/[L])) b. The lack of additivity suggests that the Arg and T
= (1/(1 + 3 × 10–4/10–8))/(1/(1 + 1.4–4/10–8)) might interact directly, as removing either side of the
= 4.49 interaction has the same effect as removing both sides.
Given that the T occurs in an AT-rich stretch of DNA
The change in specificity is 1346.
and that Arg residues can recognize such stretches
20. A protein–protein interface comprises 22 residues at through a narrowed minor groove, it is likely that this
the contact surface. From structures of the isolated mechanism contributes to the specific recognition of
proteins, it is expected that completely burying these this stretch of DNA.
residues would cause a surface area reduction of 23. Each subunit of a homodimeric transcription factor can
~2000 Å2. However, a structure of the interface reveals individually recognize a DNA half-site with a
that only 1200 Å2 of surface area is buried. Why is there 5 µM KD. The dimeric form of the transcription factor
a discrepancy between the expected and measured recognizes the full inverted repeat DNA site with a
surface area reductions? 50 nM KD. How much free energy is used to induce the
Answer: conformational changes of the protein and DNA during
Because the shape complementarity is not perfect, the binding of the dimeric transcription factor?
many residues are not completely buried in the bound Answer:
complex. Additionally, residues that interfacial waters, We expect
which are important for providing hydrogen bonding KD,dimer = (KD,monomer)2
networks at interfaces, are not normally counted as = (5 × 10–6)
part of the buried surface area. = 2.5 × 10–11
21. Consider the dsRB domain and its potential for
∆∆Gconf = RT ln(KD, dimer measured) – RT ln(KD, dimer expected)
= RT ln(50 × 10–8) – RT ln(2.5 × 10–11)
interacting with DNA and RNA (see Figure 13.32). What
= 19.0 kJ•mol–1
is the predicted effect on the specificity and affinity of
recognition for the two types of nucleic acids of: 
 24. A complex of seven transcription factors binds a DNA
a. An Arg to Ala mutation at the binding interface? 
 enhancer element. The binding is cooperative. What

The Molecules of Life by John Kuriyan, Boyana Konforti, and David Wemmer © Garland Science
4 Chapter 13: Specificity of Macromolecular Recognition

are two molecular mechanisms that the transcription


factors might use to achieve this cooperativity?
Answer:
The transcription factors may make contact with
each other facilitating the assembly of the context.
These protein–protein interactions would increase the
apparent cooperativity of the protein–DNA interactions.
Second, some of the transcription factors may
recognize a distorted DNA structure. If one transcription
factor stabilizes this distorted structure (and in essence
pays the energetic penalty for distorting it away from
ideal geometry) then subsequent binding events can
rely on the distorted DNA for specific interactions
without contributing energy to induce a conformational
change in the DNA.

The Molecules of Life by John Kuriyan, Boyana Konforti, and David Wemmer © Garland Science
The MOLECULES
of LIFE
Physical and Chemical Principles

Solutions Manual

Prepared by James Fraser and Samuel Leachman


Chapter 14

Allostery

Problems

True/False and Multiple Choice

1. Which of the following can result in an allosteric c. low pH


modulation of activity? d. myoglobin
a. Covalent modification such as phosphorylation or e. CO2
acetylation.
b. Oligomerization. 5. Bacterial chemotaxis involves only random, Brownian
c. Binding of a ligand. movement.
d. Stabilizing an alternative conformation. True/False
e. All of the above.
6. Because of the importance of the F helix in oxygen
2. The transcription of a gene is controlled by a binding, the allosteric hemoglobin proteins of all
transcription factor binding either glucose or lactose. organisms are tetrameric assemblies.
When there is 0.2 mM of either glucose or lactose in
the cell, the gene is transcribed at about 10% of the True/False
maximum. When there is more than 2 mM glucose in 7. Which of the following statements about the Hill
the cell, the gene is fully induced. However, when there coefficient are true?
is 2 mM lactose in the cell, the amount of transcription
is approximately half of the maximum. At 40 mM of i. It is the steepness of the log-log binding isotherm
either glucose or lactose in the cell, the gene is fully at the half saturation point.
induced. The transcriptional regulation is likely: ii. Allosteric systems have a Hill coefficient of exactly 1.
iii. The maximum value for a dimeric protein is 2.
a. Ultrasensitive with respect to both glucose and
lactose. a. Only (i) is true.
b. Ultrasensitive with respect to lactose but not b. Both (i) and (iii) are true.
glucose. c. Both (ii) and (iii) are true.
c. Graded with respect to glucose. d. All of the statements are true.
d. Ultrasensitive with respect to glucose but not
lactose. Fill in the Blank
e. Hyperbolic with respect to glucose but not lactose.
8. When a system is ultrasensitive, the response to
3. In allosteric proteins, ligand binding can only result in an input is _______ than expected from the graded,
positive cooperativity. hyperbolic response.
True/False
Answer: sharper
4. Which of the following is not a known allosteric effector
of hemoglobin? 9. Oxygen biases the equilibrium of hemoglobin towards
the _____ state.
a. oxygen
b. bisphosphoglycerate Answer: R or relaxed

The Molecules of Life by John Kuriyan, Boyana Konforti, and David Wemmer © Garland Science
2 Chapter 14: Allostery

10. The extent of flagellar clockwise rotation is governed by a. What fraction of the protein has ligand bound at
the ___________________ of the protein CheY. 10 nM concentration of free ligand?
b. A single point mutation abolishes all cooperativity
Answer: phosphorylation
in the protein such that the protein binds its ligand
11. The __________ histidine residue of hemoglobin senses with an apparent KD equal to 25 nM. What is the
the effective size of the heme iron atom. fraction of the protein that has ligand bound at 10 nM
concentration of free ligand?
Answer: proximal
Answer:
12. An effect of colocalization is to greatly increase the a. KD2 = KD1 × (2/nH – 1)2
local __________ of two proteins. = 25 × (2/1.9 – 1)2
= 1.56 nM
Answer: concentration
f/(1 – f) = ([L]/KD1 + ([L]/KD1) × ([L]/KD2))/(1 + [L]/KD1)
= (10/25 + (10/25) × (10/1.56))/(1 + 10/25)
Quantitative/Essay = 2.11
f = 2.11 – 2.11f
13. A dimeric enzyme, glucokinase, has a binding site for 3.11 = 2.11
glucose in each subunit. The KD for the first binding f = 2.11/3.11
event is 1 mM and the KD for the second event is 10 µM. f = 0.68
a. What is the Hill coefficient? b. f/(1 – f) = [L]/KD
b. Is this protein positively or negatively cooperative = 10/25
with respect to glucose binding? = 0.4
Answer: f = 0.4/1.4
a. nH = 2/(1 + √(KD2/KD1)) = 0.29
= 2/(1 + √(0.01/1))
= 1.8 18. The first and second binding sites of a positively
b. nH > 1, therefore it is positively cooperative. cooperative allosteric dimeric protein have KD values of
100 mM and 10 µM, respectively.
14. A dimeric hemoglobin is isolated from a fish. Each a. Sketch the binding isotherms as log[f/(1 – f )] versus
subunit contains a binding site for a xenon gas atom. log([L]).
The KD for the first binding event is measured to be b. What is the value of the Hill coefficient?
23 nM. The KD for the second binding event is measured Q14.18a
to be 3.5 µM. Answer:
a.
a. What is the Hill coefficient?
b. Is this protein positively or negatively cooperative KD1 8
with respect to xenon binding? KD2 7
Answer: isotherm 6
a. nH = 2/(1 + √(KD2/KD1)) 5
= 2/(1 + √(3.5/0.023)) 4
= 0.15 3
b. nH < 1, therefore it is negatively cooperative. 2
15. A dimeric enzyme with two identical binding sites has a 1
Hill coefficent of 1.3. If the KD of the first binding site is
–7 –6 –5 –4 –3 –2 –1 –1 1 2 3
200 nM, what is the KD of the second binding site?
–2
Answer:
–3
KD2 = KD1 × (2/nH – 1)2
–4
= 200 × (2/1.3 – 1)2
= 58 nM –5
–6
16. What is the ratio of bound to unbound receptor [f/(1 – f )]
for a dimeric protein with two binding sites (KD1 = 20 b. nH = 2/(1 + √(KD2/KD1))
nM; KD2 = 2 nM) at 15 nM concentration of ligand? = 2/(1 + √(10–5/10–1))
Answer: = 1.98
f/(1 – f) = ([L]/KD1 + ([L]/KD1) × ([L]/KD2))/(1 + [L]/KD1)
19. The first and second binding sites of a negatively
= (15/20 + (15/20) × (15/2))/(1 + 15/20)
cooperative allosteric dimeric protein have KD values of
= 3.64
100 µM and 10 mM, respectively.
17. A dimeric allosteric protein is isolated. A scientist a. Sketch the binding isotherms as log[f/(1 – f )] versus
determines that the value of KD for the first binding site log([L]).
is 25 nM and that the Hill coefficient is 1.6. b. What is the value of the Hill coefficient?

The Molecules of Life by John Kuriyan, Boyana Konforti, and David Wemmer © Garland Science
PROBLEMS and solutions 3

Q14.19a
Answer: AAC displays positive cooperativity at low temperatures
a. and negative cooperativity at high temperatures. At
7 10°C, AAC has KD1 = 330 µM and KD2 = 130 µM. At 37°C,
KD1
KD2 6 AAC has KD1 = 200 µM and KD2 = 230 µM.
isotherm 5 a. What is the Hill coefficient at each temperature?
4 b. What is the change in fraction bound at 50 µM
3 ligand between 10°C and 37°C?
2 c. Describe a reasonable mechanism underlying this
1 unusual behavior.
Answer:
–7 –6 –5 –4 –3 –2 –1 –1 1 2 3 a. For 10°C
–2 nH = 2/(1 + √(KD2/KD1))
= 2/(1 + √(130/330))
–3
=1.23
–4
For 37°C
–5
nH = 2/(1 + √(KD2/KD1))
b. nH = 2/(1 + √(KD2/KD1)) nH = 2/(1 + √(230/200))
= 2/(1 + √(10–2/10–4)) = 0.97
= 0.18 b. For 10°C
f/(1 – f) = ([L]/KD1 + ([L]/KD1) × ([L]/KD2))/(1 + [L]/KD1)
20. Two homologs of a protein are isolated. Homolog A = (50/330 + (50/330) × (50/130))/(1 + 50/330)
is a monomer that binds glucose with a KD of 4 mM. = 0.18
Homolog B is a positively cooperative dimer. The KD of f = 0.18/1.18
the first binding site of homolog B is measured at = 0.15
4 mM. At 1 mM concentration of glucose, it is found that For 37°C
homolog B binds twice as much glucose as homolog A. f/(1 – f) = ([L]/KD1 + ([L]/KD1) × ([L]/KD2))/(1 + [L]/KD1)
What is the Hill coefficient of homolog B? = (50/200 + (50/200) × (50/230))/(1 + 50/200)
Answer: = 0.243
For homolog A: f = 0.243/1.243
f/(1 – f) = [L]/KD = 1/4 = 0.25 = 0.20
f = 0.25/(1 + 0.25) = 0.2 Thus, 4% more enzymes are bound at the higher
Since homolog B binds twice as much glucose, f = 0.4; temperature. Even though AAC is negatively
f/(1 – f) = 0.667 cooperative at higher temperatures, more of the protein
f/(1 – f) = 0.667 = ([L]/KD1 + ([L]/KD1) × ([L]/KD2))/(1 + [L]/ is bound to ligand. This is driven by the favorable
KD1) change in KD1 at higher temperatures.
0.667 = (1/4 + (1/4) × (1/KD2))/(1 + 1/4) = (0.25 + 0.25 × c. One possibility is that binding to the first subunit
(1/KD2))/(1.25) destabilizes the second subunit by disrupting the
KD2 = 0.429 mM interface between them. At high temperatures this
nH = 2/(1 + √(0.429/4)) effect is enough to partially unfold the second subunit,
= 1.51 overriding the intrinsic cooperativity of the native dimer.
Hint: See LA Freiburger et al. and AK Mittermaier (2011)
21. A cyclist is interested in cheating in a race by delivering
Nat. Struct. Mol. Biol. 18, 288–294.
more oxygen to his muscles. The cyclist reasons that
since bisphosphoglycerate (BPG) stabilizes the “T” state 23. How does CO2 directly and indirectly stabilize the “T”
of hemoglobin, which reduces its affinity for oxygen, state of hemoglobin in venous blood?
that reducing the BPG concentration in his blood
cells should be good for his performance. How might Answer:
removing BPG have a detrimental effect on the delivery CO2 directly stabilizes the “T” state by reacting with the
of oxygen to his muscles? N-terminal amino groups of the protein. This alters the
interfacial interactions between the α and β subunits,
Answer: stabilizing the “T” state relative to the “R” state. CO2
BPG acts as an allosteric effector for hemoglobin. indirectly stabilizes the “T” state by reducing the pH
Removal of BPG will result in a sharper switch between within red blood cells. The reduction in pH leads to
saturated and empty hemoglobin. Without this effect, the Bohr effect where β subunit His 146 becomes
the binding isotherm will not match the difference in protonated. This disrupts interactions centered around
oxygen concentration between the arterial and venous the ion pair with β Asp 94 and an interaction to α
blood and likely lead to suboptimal delivery of oxygen to Lys 40. Without these interactions, the “T” state is
his muscles. stabilized.
22. Due to a fascinating coupling between folding and 24. Two proteins are modified by myristoylation, which
ligand binding, the dimeric acetyl transferase enzyme targets them to the plasma membrane in a cell at 25°C.

The Molecules of Life by John Kuriyan, Boyana Konforti, and David Wemmer © Garland Science
4 Chapter 14: Allostery

This changes their effective local concentration from 10


nM to 1 µM. Assume that any favorable mutation would
decrease the value of ΔGo for binding by 4 kJ•mol–1.
How many favorable mutations would have to occur in
the absence of colocalization to result in an equivalent
effective affinity as observed when the two proteins are
colocalized?
Answer:
∆∆G = RT ln(10–8 × 10–8)/(10–6 × 10–6) = 2.5 × ln (0.001) =
–23 kJ•mol–1
If each mutation added 4 kJ• mol–1, the at least six
mutations would be required to result in as high an
effective affinity.

25. A scientist finds two pathways, both of which depend


on distinct kinase activities, which activate a stress
response in yeast. One pathway responds to elevated
levels of salt and the other responds to elevated levels
of caffeine. Both pathways result in transcription of the
chaperone Hsp90. Below is a table listing the relative
concentrations of salt or caffeine and the measured
transcription of Hsp90. Explain which pathway likely
contains a single kinase and which pathway likely
contains a kinase cascade similar to the MAP kinase
pathway.

[NaCl] Hsp90 [Caffeine] Hsp90


(mM) transcription (mM) transcription

1 0 1 0

2 1 2 0

3 1 3 1

4 40 4 1.1

5 100 5 2

10 100 10 10

100 100 100 50

1000 100 1000 100

Answer:
Only the salt response is ultrasensitive (in that it
undergoes a complete activation in a ~2-fold rather
than ~100-fold concentration change). Ultrasensitivity
resulting from the coupling of multiple kinases converts
a graded input signal into a sharp switch. This situation
is observed only for the salt response, which likely has
a kinase cascade, and not for the caffeine response,
which is more hyperbolic.

The Molecules of Life by John Kuriyan, Boyana Konforti, and David Wemmer © Garland Science
The MOLECULES
of LIFE
Physical and Chemical Principles

Solutions Manual

Prepared by James Fraser and Samuel Leachman


Chapter 15
The Rates of Molecular
Processes

Problems

True/False and Multiple Choice

1. What is the order of this elementary reaction: 7. After excitation, the intensity of light emitted by a
A + 2B → 1C sample of fluorescent molecules drops exponentially
a. 2 with time.
b. 7
True/False
c. 4
d. 3
e. 8 Fill in the Blank
2. The addition of a catalyst increases the rate of the
8. The symbol “‡” is used to represent the ______________
reaction but not the equilibrium constant.
of the reaction.
True/False
Answer: transition state
3. For a reaction with a larger ΔG compared to a reaction
with a smaller ΔG, 9. The rate law gives the relationship between the rate
a. The reaction with the larger ΔG is always the faster of a reaction and the ________ of the chemical species
reaction. involved in the reaction.
b. The reaction with the larger ΔG is always the
slower reaction. Answer: concentrations
c. The two rates are equal.
d. It is impossible to decide which reaction is 10. The rate constant for a _____-order elementary reaction
faster. is M–1•sec–1.
Answer: second
4. For elementary reactions of zero and second order
with a rate constant of 1 (with appropriate units),
11. The rate of product formation at the end of a pathway is
the second-order reaction half-life is a shorter period of
determined by the rate of the ______ step.
time.
True/False Answer: slowest

5. The most basic step used to describe a reaction process 12. Catalysts can work by changing the reaction
is: mechanism, ________, or _________.
a. The transition state.
b. An elementary reaction. Answer: decreasing the activation energy, increasing
c. A unimolecular reaction. the pre-exponential factor/[increasing the rate of
d. The steady state. collisions with favorable orientation]
e. The equilibrium rate.

6. The probability of a bimolecular reaction occurring is


Quantitative/Essay
related to the rate of collisions between the two species 13. How do the kinetics of hydrolysis of ATP to ADP and
of molecules. phosphate by water make it a good energy reservoir for
True/False the cell?

The Molecules of Life by John Kuriyan, Boyana Konforti, and David Wemmer © Garland Science
2 Chapter 15: The Rates of Molecular Processes

Answer: Apply the rate constant, k, the final mass (1.5 μg) [A],
Hydrolysis of ATP yields a large amount of energy and starting mass (15 μg) [A0] to the first order rate
(~30 kJ•mol–1) but the forward rate is very slow. equation:
Unless an enzyme catalyst is present, water does not [A] = [A0]e–kt
spontaneously react frequently with ATP. Due to this t = –ln([A]/[A0])/k
slow rate of hydrolysis, very little energy is lost unless a t = –ln(1.5 μg/15 μg)/0.053 hour–1
catalyst initiates the reaction. t = –ln(0.1)/0.053 hour–1
t = 43 hours
14. Problem
Below is a15.14
two-dimensional energy versus reaction
coordinate diagram for the reaction AB + C → A + BC. 16. A reaction is half complete after 20 minutes. After
40 minutes the reaction is two-thirds complete. When
will the reaction be 90% complete?
Answer:
First we must determine the order of the reaction. The
reaction cannot be first order because the half-life is
not constant (it halves in 20 minutes, and then in the
next 20 minutes only decreases to one-third). It cannot
r1

be a zeroth order reaction, where we would expect the


half life to decrease as the reaction proceeded, since as
the reactants decrease the half life increases. We can
test whether the reaction is second order:
t1/2 = 1/k[A0]
20 min = 1/k[1]
k = 1/20 min = 0.05 min–1
r2 Test for second order kinetics when 2/3 complete
(A40 = 1/3):
Circle the approximate area that represents 1/[A0] – 1/[A40] = –k × t
the transition state and justify your answer. If r1 1 – 1/(1/3) = –0.05 min–1 × t
corresponds to the A-B distance, and r2 to the B-C –2 = –0.05 min–1 × t
distance, indicate what molecules would be present t = 40 min, which agrees with the statement, confirming
in the regions corresponding to the left hand and right second-order kinetics
hand sides of the reaction above. When the reaction is 90% complete, [A] = 1/10
Answer: 1/[A0] – 1/[A] = –k × t
The transition state is the highest point of energy on the 1 – 10 = –0.05 × t
15Q14 –9/–0.05 min–1 = t
trajectory between reactants and products.
t = 180 minutes
17. A protein (P) can either fold properly into the native
Right side state (N) or aggregate into a misfolded form (A). Both
A + BC processes obey first-order kinetics. The branching ratio
([N]/[A]) is 9 and the effective rate constant, keff , is 15
sec–1. What is the rate constant for native state folding?
Answer:
r1

From the branching ratio:


kN/kA = 9
Transition Left side kA = kN/9
state AB + C
keff = (kN + kA)
keff = (kN + kN/9)
keff = 10kN/9
(15 sec–1 × 9)/10 = kN
r2 kN = 13.5 sec–1
18. Upon excitation, a modified green fluorescent protein
15. Iodine-123 is important for medical imaging studies and emits photons that yield an initial intensity of 10,000
follows first-order decay kinetics. A 15-µg sample of units in the fluorimeter. After 2 nanoseconds, the
I-123 has decayed to 7.5 µg after 13 hours. After how signal has decayed to 300 units. If the rate constant for
much time will it decay to only 1.5 µg? fluorescent production of light (kf) is 0.1 nsec–1, what
are the values of the rate constant for heat production
Answer:
(kh) and the fluorescence lifetime (τf)?
It has decayed to half the original mass after 13 hours,
which therefore corresponds to the half-life (t1/2). Answer:
To find the rate constant: I2nsec/I0nsec = kf[F*]0e–(kf+kh)t/kf[F*]0e–0 = e–(kf+kh)(2 nsec)
k = ln(2)/t1/2 = 0.693/13 hour = 0.053 hour–1 300/10000 = e–(0.1 nsec–1 + kh)(2 nsec)

The Molecules of Life by John Kuriyan, Boyana Konforti, and David Wemmer © Garland Science
PROBLEMS and solutions 3

0.03 = e–(0.1 nsec–1 + kh)(2 nsec) are conducted on the isomerization of an alanine-
ln(0.03) = –(0.1 nsec–1 + kh)(2 nsec) proline peptide. At 25°C (298 K) the observed
0.1 nsec–1 + kh = 1.75 nsec rate constant is 0.05 sec–1 and the value of EA is
kh = 1.65 nsec–1 calculated to be 60 kJ•mol–1. What is the value of the
τf = 1/(kf + kr) preexponential factor (A)? Similar measurements are
= 1/(0.1 nsec–1 + 1.65 nsec–1) performed on a phenylalanine-proline peptide at 25°C,
= 0.570 nsec with a measured rate constant of 0.005 sec–1. Assuming
an identical preexponential factor as the alanine-proline
19. An experiment is performed to measure the affinity of peptide, what is the activation energy for this peptide?
the human proline isomerase hCypA to the inhibitor
cyclosporin, analogous to the experiments described for Answer:
imatinib in this chapter. At 25 µM cyclosporin, the value ln (k) = –EA/RT + ln(A)
of kobs is measured to be 12.55 sec–1. Given that the off- ln(k) + EA/RT = ln(A)
rate is 0.01 sec–1, what is the on-rate? ln(0.05) + 60,000/(8.314 × 298) = ln(A)
A = e21.22 = 1.64 × 109 sec–1
Answer: Assuming that A = 1.64 × 109 sec–1 for the
kobs = kf[L] + kr phenylalanine–proline peptide
12.55 sec–1 = kf[25 μM] + 0.01 sec–1 EA = –(ln(k) – ln(A)) × RT = –(ln(0.005) – ln(1.64 × 109)) ×
12.54 sec–1/25 μM = kf 8.314 × 298 = 65.7 kJ•mol–1
kf = 0.501 μM–1•sec–1
kf = 0.501 × 10–6 M–1•sec–1 23. A relaxation experiment probes the homodimerization
approximate values from M.A. Wear et al. and of an oligonucleotide. At 10-nM oligonucleotide, the
M.D. Walkinshaw, Anal. Biochem. 345, 214–226, 2005. apparent rate constant (kapparent) is 2.06 sec–1. At
20. A homologous proline isomerase, vCypA, is isolated 100-nM oligonucleotide, the apparent rate constant
from a deadly virus. The kinetics of cyclosporin binding (kapparent) is 6.34 sec–1. What are the association and
to vCypA were measured. The off-rate is 0.1 sec–1 and dissociation rate constants?
the on-rate is 0.05 µM–1•sec–1. What concentration of Answer:
cyclosporin was used to yield kobs = 120 sec–1? kapp2 = kr2 + 8kfkr[N]
Answer: At 10 nM:
kobs = kf[L] + kr 4.24 = kr2 + 80kfkr
120 sec–1 = 0.05 μM–1•sec–1[L] + 0.1 sec–1 (4.24 – kr2)/80 = kfkr
[L] = 119.9 sec–1 / 0.05 μM–1•sec–1 Substitute into 100 nM
[L] = 2398 μM = 2.398 mM 40.2 = kr2 + 8 × (4.24 – kr2)/80 × [100]
40.2 = kr2 + (4.24 – kr2) × 10
21. Several experiments indicate that the vCypA is 40.2 = kr2 + 42.4 – 10kr2
essential for viral replication. Use the parameters 9 kr2 = 2.24
calculated in Problems 19 and 20, and assume that kr2 = 0.249
the concentration of cyclosporin would be much larger kr = 0.500 sec–1
than the concentration of either vCypA or hCypA. For Substitute this value to solve for kf
each protein, calculate the value of KD for cyclosporin. 4.24 = 0.25 + 80kf × 0.5
Given the ratio of the two KD values, explain whether
4 = 40kf
cyclosporin could be an effective treatment for this
kf = 0.100 sec–1•nM–1
virus.
Answer: 24. A protein folding reaction has two intermediate
For hCypA: states, each of which individually obeys Arrhenius-
KD = kr/kf = 0.01 sec–1/0.501 × 10–6 M–1•sec–1 = type behavior. At low temperatures, forming the first
2 × 10–8 M–1•sec–1 intermediate is rate-limiting. At high temperatures,
forming the second intermediate is rate-limiting.
For vCypA:
KD = kr/kf = 0.1 sec–1/0.05 × 10–6 M–1•sec–1 = a. Does the protein folding reaction obey Arrhenius-
2 × 10–6 M–1•sec–1 type behavior over all temperatures?
The ratio of KDs: b. Forming which intermediate has a higher activation
KD-hCypA / KD-vCypA = 2 × 10–8 M–1•sec–1/2 × energy?
10–6 M–1•sec–1 = 0.01
Answer:
The ratio of cyclosporine-bound to unbound hCypA is a. No. The rate-determining step changes as the
100 times greater than for vCypA. This suggests that temperature changes, so the apparent rate constant
the drug might not be a very effective treatment for will change as well. Above the temperature where
the virus, since it will preferentially bind to the human forming the second intermediate is rate-determining,
protein rather than the viral one. the slope of the temperature dependence will be
22. The activation energy for proline isomerization of dominated by the activation energy of forming the
a peptide depends on the identity of the preceding second intermediate. Below that temperature, the
residue and obeys Arrhenius rate behavior. Experiments slope will be dominated by the activation energy of

The Molecules of Life by John Kuriyan, Boyana Konforti, and David Wemmer © Garland Science
4 Chapter 15: The Rates of Molecular Processes

forming the first intermediate. This generates curvature


in the Arrhenius plot that is uncharacteristic of linear
Arrhenius-type behavior.
b. Formation of the first intermediate has a higher
activation energy. At low temperatures, forming the
first intermediate is slower than forming the second.
As the temperature is increased, the rate of forming
the first intermediate increases faster than the rate of
forming the second, such that at high temperatures,
formation of the second intermediate is slower.
Because forming the first intermediate has greater
temperature dependence, it must necessarily have a
higher activation energy.

The Molecules of Life by John Kuriyan, Boyana Konforti, and David Wemmer © Garland Science
The MOLECULES
of LIFE
Physical and Chemical Principles

Solutions Manual

Prepared by James Fraser and Samuel Leachman


Chapter 16
Principles of Enzyme
Catalysis

Problems

True/False and Multiple Choice

1. The initial reaction velocity for an enzyme reaction 6. An enzyme inhibitor is observed to alter the KM but not
reaches a maximum at high substrate concentration the Vmax of a reaction. This inhibitor is most likely:
because the free enzyme can no longer regenerate at a. A noncompetitive inhibitor.
the end of each reaction cycle. b. A competitive inhibitor.
True/False c. An allosteric inhibitor.
d. A substrate-dependent noncompetitive inhibitor.
2. The turnover number for an enzyme obeying Michaelis– e. A covalent inhibitor.
Menten kinetics is:
a. k2. 7. Due to its extremely slow dissociation kinetics, the
b. kcat/KM. protein bovine pancreatic trypsin inhibitor (BPTI) has
c. k1/k–1. broad specificity and inhibits more proteases than
d. (k1 + k2). protease inhibitors that are small molecules.
e. ΔG‡. True/False

3. Catalytic antibodies are generally less efficient than


natural enzymes that catalyze the same reactions. Fill in the Blank
True/False
8. In the schemes for the catalyzed reactions considered
in this chapter, S, E, and P refer to _______________,
4. A metabolic enzyme generates the amino acid ______________, and ______________, respectively.
methionine. For a given substrate concentration, an
experiment conducted in the presence of high initial Answer: substrate, enyzme, product
concentrations of methionine generates less new
methionine than an experiment conducted with no 9. The specificity constant or catalytic efficiency is the
initial methionine present. This is likely an example of: ratio between __________ and _________.
a. A ping-pong mechanism of substrate binding. Answer: kcat, KM
b. A proximity effect.
c. Substrate strain. 10. In a plot of initial velocity versus substrate
d. Product inhibition. concentration, an allosteric enzyme displays a
e. A reaction intermediate. ____________ curve, whereas a non-allosteric enzyme
that obeys Michaelis–Menten kinetics displays a
5. Which of the following is not a commonly observed ____________ curve.
feature of proteases?
a. The catalytic triad in the active site. Answer: sigmoidal, hyperbolic
b. Exclusively hydrophobic residues in the active 11. The geometry of competitive inhibitors commonly
site. mimics the _____________ of the reaction that the
c. A cysteine residue in the active site. enzyme normally catalyzes.
d. Metal ions coordinated in the active site.
e. A pair of acidic residues in the active site. Answer: transition state

The Molecules of Life by John Kuriyan, Boyana Konforti, and David Wemmer © Garland Science
Figure Q16A
2 Chapter 16: Principles of Enzyme Catalysis

12. Proteins are not the only polymers that act as catalysts. a.
Catalytic ______ molecules are also essential for cells, 2.5
including playing an essential role in protein synthesis.
Answer: RNA/ribozyme 2.0

1/v (M–1•sec)
Quantitative/Essay 1.5
13. At 25°C, an enzyme accelerates a reaction by a factor
of 105 over the uncatalyzed reaction in water. If the 1.0
effect of the enzyme is solely to reduce the energy of
the transition state, by what amount does it reduce the
energy of the transition state (EA)?
0.5

Answer:
kcat/kuncat = e(‒(ΔG
o‡cat ‒ ΔGo‡uncat)/RT)
Figure Q16B0
o‡cat ‒ ΔGo‡uncat)/8.314 × 298) 0 0.2 0.4 0.6 0.8 1.0 1.2
105 = e(‒(ΔG
–1
1/[S] (M )
ln(10 ) = (‒(ΔG cat ‒ ΔGo‡uncat)/2.5)
5 o‡

ΔGo‡cat ‒ ΔGo‡uncat = –28.5 kJ•mol–1


b.
14. Show that the equations plotted in Lineweaver–Burk
and Eadie–Hofstee plots are equivalent.
0.7

Answer: 0.6
Start with the Lineweaver–Burk equation:
1/v0 = 1/Vmax + KM/Vmax[S] 0.5
1/v (M–1•sec)

Multiply all by Vmax,


Vmax/v0 = 1 + KM/[S] 0.4
Multiply all by v0,
0.3
Vmax = v0 + v0KM/[S]
Rearrange for v0, 0.2
v0 = –v0KM/[S] + Vmax
(which is the Eadie–Hofstee equation). 0.1

15. In a reaction, E + S ← → E•⋅ S → E + P , calculate

k1 k2
Figure Q16C
k 0
0 0.2 0.4 0.6 0.8 1.0 1.2
−1

the value of KM if the forward rate constant (k1) for


–1
1/[S] (M )
E•S formation is 4.3 × 106 sec–1•M–1, the reverse rate
constant (k–1) for E•S dissociation is 2.4 × 102 sec–1, and
the turnover number (k2) is 1.2 × 103 sec–1. c.
4.5
Answer:
KM = (k–1 + k2)/k1 = (2.4 × 102 sec–1 + 1.2 × 103 4.0
sec–1)/4.3 × 106 sec–1•M–1 3.5
= 3.3 × 10–4 M
3.0
1/v (M–1•sec)

16. Presented below are Lineweaver–Burk plots for


2.5
enzymatic reactions with (red) and without (blue)
inhibitor. What type of inhibition is occurring in each 2.0
case? 1.5
1.0
0.5
0
0 0.2 0.4 0.6 0.8 1.0 1.2
1/[S] (M–1)

The Molecules of Life by John Kuriyan, Boyana Konforti, and David Wemmer © Garland Science
PROBLEMS and solutions 3

Answer: Substitute slope into:


a. Competitive inhibition. 1/v = slope × 1/[S] + 1/Vmax
b. Substrate-dependent noncompetitive inhibition. 0.5 = 0.8 × 0.5 + 1/Vmax
c. Noncompetitive inhibition. 0.1 = 1/Vmax
Vmax = 10 mM•sec–1
17. The table below lists initial velocities measured for an KM/Vmax = 0.8, K*M = 8 mM
enzymatic reaction at different substrate concentrations
c. The slope changes but the y intercept does not.
in the presence and absence of an inhibitor. The enzyme
This is reflected in the apparent KM increase upon
concentration is identical in both reactions.
addition of inhibitor and the constant Vmax between the
two reactions. These observations are indicative of a
[S] (mM) v, no inhibitor 
 v, with inhibitor 
 competitive inhibitor.
(mM•sec–1) (mM•sec–1)
d. For a competitive inhibitor, the apparent KM with
1 2.50 1.11 inhibitor is related to the KM without inhibitor:
K*M = KM(1 + [I]/KI)
2 4.00 2.00 8 mM = 3 mM (1 + 0.5 mM/KI)
5 6.25 3.85 8 mM = 3 mM + 1.5 mM/KI
5 mM = 1.5 mM/KI
10 7.69 5.56 KI = 1.5 mM/5 mM
KI = 0.3 mM
20 8.70 7.14
18. The table below lists initial velocities measured for an
a. Graph a Lineweaver–Burk plot. enzymatic reaction at different substrate concentrations
b. What are the apparent values of Vmax and KM for in the presence and absence of an inhibitor. The enzyme
each experiment? concentration is identical in both reactions.
c. What is the inhibition mechanism?
d. If the concentration of inhibitor is 0.5 mM, what is
[S] (mM) v, no inhibitor v, with inhibitor 

(mM•sec–1) (mM•sec–1)
the value of KI?
Answer: 1 1.000 0.923
Q16.17a
a. Red line is with inhibitor, blue line is with no
5 1.154 1.053
inhibitor.
10 1.176 1.071
1.0
no inhibitor
with inhibitor 50 1.195 1.087
0.8
1/v (mM–1•sec)

100 1.198 1.089

0.6
a. Graph a Lineweaver–Burk plot for each set of data.
b. What are the apparent values of Vmax and KM for
0.4 each experiment?
c. What is the inhibition mechanism?
0.2 d. If the concentration of inhibitor is 10 nM, what is
the value of KI?
0.0 Answer:
0.0 0.2 0.4 0.6 0.8 1.0 1.2 a. Red line is with inhibitor, blue line is with no
Q16.18a
1/[S] –1
(mM ) inhibitor.
1.2
b. First calculate the slope of the lines—the data are
nearly perfectly linear, so we will just use the first two 1.0
data points to calculate the relevant parameters.
For no inhibitor:
1/v (mM–1•sec)

0.8
Slope = KM/Vmax = (y1 – y2)/(x1 – x2) = (0.4 – 0.25)/
(1 – 0.5) = 0.3
0.6
Substitute slope into:
1/v = slope × 1/[S] + 1/Vmax
0.25 = 0.3 × 0.5 + 1/Vmax 0.4
0.1 = 1/Vmax
Vmax = 10 mM•sec–1 0.2 no inhibitor
with inhibitor
KM/Vmax = 0.3, KM = 3 mM
For with inhibitor: 0.0
Slope = KM/Vmax = (y1 – y2)/(x1 – x2) = (0.9 – 0.5)/(1 – 0.5) 0.0 0.2 0.4 0.6 0.8 1.0 1.2
= 0.8 1/[S] (mM–1)
The Molecules of Life by John Kuriyan, Boyana Konforti, and David Wemmer © Garland Science
4 Chapter 16: Principles of Enzyme Catalysis

b. For no inhibitor: Q16.19a


Answer:
Slope = KM/Vmax = (y1 – y2)/(x1 – x2) = (1 – 0.85)/(1 – 0.1) a.
= 0.167 0.16
Substitute slope into:
1/v = slope × 1/[S] + 1/Vmax 0.14
1 = 0.167 × 1 + 1/Vmax 0.12
Vmax = 1.2 mM•sec–1

1/v (µM–1•sec)
KM/Vmax = 0.167, KM = 0.167 × 1.2 = 0.2 mM 0.10
With inhibitor:
0.08
Slope = KM/Vmax = (y1 – y2)/(x1 – x2) = (0.95 – 0.92)/(0.2 –
0.02) = 0.167 0.06
Substitute slope into:
1/v = slope × 1/[S] + 1/Vmax 0.04
0.95 = 0.167 × 0.2 + 1/Vmax no inhibitor
0.02 with inhibitor
Vmax = 1.09 mM•sec–1
KM/Vmax = 0.167, KM* = 0.167 × 1.09 = 0.182 mM 000
c. The y intercept changes, but the slope does not. 0.0 0.02 0.04 0.06 0.08 0.10 0.12
This is reflected in the apparent KM decrease upon 1/[S] (µ M )
–1
addition of inhibitorand the apparent Vmax decrease,
but the maintenance of a constant ratio between the b. First calculate the slope of the lines—the data are
two constants. These observations are indicative of a nearly perfectly linear, so we will just use the first two
substrate-dependent noncompetitive inhibitor. data points to calculate the relevant parameters.
d. 10 nM is 0.00001 mM, K*M = KM/(1 + [I]/KI) For no inhibitor:
K*M = KM/(1 + [I]/KI) Slope = KM/Vmax = (y1 – y2)/(x1 – x2) = (0.112 – 0.096)/
0.182 mM = 0.2 mM/(1 + 0.00001 mM/KI) (0.1 – 0.05) = 0.32
0.00001 mM/KI = 0.2 mM /0.182 mM – 1 Substitute slope into:
KI = 0.1 μM or 100 nM 1/v = slope × 1/[S] + 1/Vmax
0.112 = 0.32 × 0.1 + 1/Vmax
19. The table below lists initial velocities measured for an 0.08 = 1/Vmax
enzymatic reaction at different substrate concentrations Vmax = 12.5 μM•sec–1
in the presence and absence of an inhibitor. The enzyme KM/Vmax = 0.32, KM = 4 μM
concentration is identical in both reactions. For no inhibitor:
Slope = KM/Vmax = (y1 – y2)/(x1 – x2) = (0.144 – 0.112)/
[S] (µM) v, no inhibitor 
 v, with inhibitor 
 (0.1 – 0.05) = 0.64
(µM•sec–1) (µM•sec–1) Substitute slope into:
1/v = slope × 1/[S] + 1/Vmax
10 8.93 6.94
0.112 = 0.64 × 0.05 + 1/Vmax
20 10.42 8.93 0.08 = 1/Vmax
Vmax = 12.5
30 11.03 9.87 KM/Vmax = 0.64, K* M = 8 μM

100 12.02 11.57 c. The slope changes but the y intercept does not.
This is reflected in the apparent KM increase upon
200 12.25 12.02 addition of inhibitor and the constant Vmax between the
two reactions. These observations are indicative of a
a. Graph a Lineweaver–Burk plot for each set of data. competitive inhibitor.
b. What are the values of Vmax and KM for each d. For a competitive inhibitor, the apparent KM with
experiment? inhibitor is related to the KM without inhibitor:
c. What is the inhibition mechanism ? K*M = KM (1 + [I]/KI)
d. If the concentration of inhibitor is 100 nM, what is 8 = 4(1 + 0.1/KI)
the value of KI? 8 = 4 + 0.4/KI
4 = 0.4/KI
KI = 0.4/4
KI = 0.1 μM or 100 nM

20. An experiment with 10 nM of an enzyme obeying


Michaelis-Menten kinetics yields a Vmax of 7 × 10–3
M•sec–1.
a. What is the turnover number (k2)?
b. The experiment is repeated in the presence of a
noncompetitive inhibitor and the Vmax is reduced to

The Molecules of Life by John Kuriyan, Boyana Konforti, and David Wemmer © Garland Science
PROBLEMS and solutions 5

5 × 10–4 M•sec–1. What fraction of the enzyme is bound Answer:


to the inhibitor? Recognize v0 = Vmax/(1 + KM/[S]), so a concentration
Answer: near KM will show half the velocity of the highest
a. Vmax = k2[E] velocity observed, assuming it is close to Vmax.
7 × 10–3 M•sec–1 = (k2)(1 × 10–8 M) a. ~3–5 × 10–3 M
k2 = 7 × 105 sec–1 b. ~4 × 10–9 M
c. ~200 × 10–3 M
b. Vmax = k2(1 – f)[E]
(1 – f) = 5 × 10–4 M–1•sec–1/(7 × 105 sec–1 × 1 × 10–8 M)
22. When the bi-substrate analog PALA is added to the
1 – f = 0.07
enzyme ATCase at low concentration it increases the
f = 0.93
rate of reaction of aspartate and carbamylphosphate.
Therefore 93% of the enzymes are bound to the However, at higher concentrations it decreases the
inhibitor. reaction rate. How can PALA act as both an activator
and inhibitor of ATCase?
21. Given the following three data tables of substrate
concentrations and initial velocities for enzymes that Answer:
obey Michaelis–Menten kinetics, estimate KM for each PALA mimics the two substrates of ATCase. Its
enzyme in molar units. affinity is greater than the substrate, and therefore
a. outcompetes substrate for the active site. At moderate
concentrations (where not every site on ATCase is
[S] (mM) v0 (mM•sec–1) occupied by PALA), the equilibrium of ATCase is shifted
to favor the active conformation. Since the active
1 266.7 conformation has a higher affinity for substrate, binding
sites that are not occupied by PALA are able to increase
3 553.8
the reaction velocity. At high concentrations of PALA
5 705.9 all binding sites become occupied by inhibitor and
turnover decreases.
50 1121.5

500 1191.7 23. In the search for the catalytic mechanism of an enzyme,
three mutations of charged residues to alanine (which
5000 1199.2 is uncharged) are made and compared with the wild
type (WT) enzyme. At otherwise identical conditions
and concentrations of enzymes, the following initial
b. velocities (µM•sec–1) are measured as a function of pH.

[S] (nM) v0 (mM•min–1)


pH WT Arg55Ala Glu63Ala Lys113Ala
4 123.5
4 1.3 × 103 1.3 × 103 1.3 × 102 1.5 × 103
5 137.4
5 8.7 × 105 8.7 × 105 1.3 × 102 9.5 × 105
6 148.5
6 8.1 × 104 8.1 × 104 1.3 × 102 8.1 × 104
10 177.3
7 5.3 × 103 5.3 × 103 1.3 × 102 5.2 × 103
100 240.2

1000 249.0
a. Explain which residue likely acts as a general acid/
base during catalysis?
b. What is a possible mechanism for the slightly
c. increased reaction velocities observed in the Lys113Ala
mutants at lower pH?
[S] (mM) v0 
(M•hour–1) Answer:
1 0.00 a. Glu63 likely participates directly in catalysis. All
other mutations have much higher catalytic efficiency
10 0.01 (comparable to wild type). Additionally, Glu63Ala no
longer displays any pH dependency when mutated to an
100 0.07
uncharged residue.
200 0.10 b. Having a Lys near the catalytic Glu will shift the pKa
down and make it less likely to donate a hydrogen to
1000 0.17 the reaction. Removing this charge with the Lys113Ala
5000 0.19 mutant shifts the pKa slightly and leads to increased
reactivity and increased reaction velocities.

The Molecules of Life by John Kuriyan, Boyana Konforti, and David Wemmer © Garland Science
6 Chapter 16: Principles of Enzyme Catalysis

24. Why is triose phosphate isomerase considered to be an


example of a “perfect enzyme”?
Answer:
Triose phosphate isomerase is considered a “perfect”
enzyme because it catalyzes its reaction so fast that it
is diffusion controlled.

The Molecules of Life by John Kuriyan, Boyana Konforti, and David Wemmer © Garland Science
The MOLECULES
of LIFE
Physical and Chemical Principles

Solutions Manual

Prepared by James Fraser and Samuel Leachman


Chapter 17

Diffusion and Transport

Problems

True/False and Multiple Choice

1. Bacterial movement towards nutrients resembles a c. solute flux is higher in acetone.


biased random walk. d. it is easier to concentrate water than it is acetone.
True/False
2. The concentration gradient of bicoid protein in Fill in the Blank
Drosophila embryos is established by:
a. passive diffusion of protein molecules. 8. ________ transport moves molecules through diffusion,
b. an mRNA gradient. while __________ transport drives movement through
c. spatially biased maternal deposition of mRNA. the coupling to chemical energy.
d. all of the above.
 Answer: Passive, active
3. The distance moved in two-dimensional diffusion 9. The width of the Gaussian distribution describing a
has a square-root dependence on time ( t ). In three one-dimensional random walk ____________ with an
dimensions, the distance covered through diffusion has increasing number of time steps.
a cubed-root dependence on time ( 3 t ). Answer: increases or broadens
True/False
10. Viscosity is a measure of a fluid’s ___________ to
4. In passive transport with no barriers, if the
particles moving through it.
concentration of molecules is high in one area but low
in another area, then Answer: resistance
a. the molecules will have a net flux to the area of low
11. An oblate or prolate object will have increased friction
concentration.
relative to a _______ object that occupies the same
b. the concentration will tend to be equal in both
volume.
areas after a long period of time.
c. the molecules will not move unless ATP is added. Answer: spherical
d. both a and b.
12. The velocity at which friction balances applied force is
e. none of the above.

called the _____________.
5. A spherical virus with a radius of 30 nm will move more Answer: terminal velocity
slowly by diffusion than a protein dimer with a radius
of 5.5 nm.
Quantitative/Essay
True/False

6. The rate of diffusion increases as friction increases. 13. The protein cyclophilin is a monomer with a diffusion
True/False constant of 1.2 × 10–8 cm2•sec–1 in water at 25°C.
Cyclophilin binds HIV capsid and the complex has a
7. Water has a higher viscosity than acetone because diffusion constant of 7 × 10–9 cm2•sec–1. What is the
a. it has a greater molecular mass. difference in the typical (r.m.s.) distance that cyclophilin
b. hydrogen bonds allow for the transfer of alone will travel versus the cyclophilin-capsid complex
momentum. in 10 seconds?

The Molecules of Life by John Kuriyan, Boyana Konforti, and David Wemmer © Garland Science
2 Chapter 17: Diffusion and Transport

Answer: is capable of making weak non-sequence-specific


rcyclophilin – rcyclophilin–capsid = √(6Dcyclophilint) – contacts with genomic DNA. Thus rather than searching
√(6Dcyclophilin–capsidt) the entire nucleus (in three dimensions) a DNA binding
= √(6 × 1.2 × 10–8 cm2•sec–1 × 10 sec) – √(6 × 7 × protein can slide along the DNA (in one dimension)
10–9 cm2•sec–1 × 10 sec) through these weak interactions until the recognition
= 8.5 × 10–4 cm – 6.5 × 10–4 cm sequence is located. Additionally, the oligomeric
= 2.0 × 10–4 cm structure of the lac repressor enables weak contacts
to be made across DNA strands. In eukaryotic cells,
14. There are approximately 250,000,000 hemoglobin nucleosome structure brings many DNA segments
molecules per red blood cell. How many collisions will nearby further reducing the three-dimensional space
an oxygen molecule (O2; mass of 32 amu = 5.3 × that needs to be searched.
10–26 kg) of radius 0.21 nm have with hemoglobin of
radius 3 nm in a blood cell of volume 10–16 m3 in 1 sec 18. Below are traces from dynamic light scattering
at 25°C? Assume both molecules are spheres, and that experiments over the same length of time using purified
the viscosity is that of pure water. wild-type and mutant kinase protein. The wild-type
protein exists primarily as a dimer, whereas the mutant
Answer:
protein is primarily a monomer. Explain which trace
collision rate = 4πr0 (DO2 + Dmyo) CO2 + Cmyo [Equation Problem 17.18
came from which protein.
17.46]
r0 = collision radius = rO2 + rmyo = (0.21 + 3.0) × 10–7 cm
D = kBT/6πρr
collision rate = 4 × 3.21 × 10–2 cm × (1.38 × 10–16 gcm2
sec–2 K–1 × 298 K)/6 × 10–2 g cm–1 sec –1)
× [(1/0.21 × 10–7 cm) × (1/3.0 × 10–7 cm)] × 1/10–8 cm3
× (2.5 × 108/10–8 cm3) = 1.1 × 1010 sec–1 cm–3
so in the cell volume of 10–8 cm3 there will be
110 collisions Problem 17.18

15. What is the typical (r.m.s.) distance traveled by a


spherical HIV virus (radius 120 nm) in 1 hour through the
bloodstream (viscosity = 3 cP) at 37°C?
Answer:
DHIV = kBT/(6πηr)
= 1.38 × 10–23 J•K–1 × 310 K/(6π × 3 cP × 120 nm)
= 4 × 10–21 kg•m2•sec–2/(0.57 g•cm–1•sec–1 × 1.2 ×
10–5 cm)
= 4 × 10–21 m2•sec–1/(6.84 × 10–9) Answer:
= 6.3 × 10–13 m2•sec–1 Because it is larger, the dimer will cause slower
rrms = √(6 × 6.3 × 10–13 × 60 sec/min × 60 min/hour) fluctuations in the intensity of light as it diffuses slowly
= 0.117 mm into or out of the scattering volume. The smaller
monomer will cause faster fluctuations in the intensity
16. Why is the diffusion-limited rate of collisions essentially of light. Therefore the top DLS trace is the wild-type
constant at 1010 M–1•sec–1—that is, why is it essentially protein, and the bottom trace is the mutant protein.
independent of the size of the molecules involved? Problem 17.19
19. The same proteins from
Answer: the previous problem are
The diffusion limited collision rate is given by analyzed by equilibrium
k = 4π(Da + Db)(ra + rb). The molecular size and the ultracentrifugation. The
log(A280)

diffusion coefficient have a reciprocal relationship logarithm of the absorbance


(D ~ 1/r). Thus any increase in molecular size slows the is plotted versus squared
diffusion rate (Da) by a factor (r) which is canceled by distance from the top of the
the increase in the radius term (ra). measurement cell at
10,000 rpm for each
17. What physical and chemical properties of DNA and protein. r2 (cm2)
the lac repressor account for the observation that the a. Explain which sample
association rate is faster than the three-dimensional is the mutant and which is the wild type.
diffusion limit? b. Is there monomer–dimer exchange in either the
Answer: wild-type or mutant kinase?
The association rate can be faster because of restrictive Answer:
diffusion, which effectively generates a one-dimensional a. Equation 17.69 indicates that the slope of ln(c)
movement in a three-dimensional search. Most DNA with respect to (r2) is proportional to ω2(1 – ρsolve/
binding proteins have a positively charged face that ρprotein)/2RT × M, but everything except M (mass) is
The Molecules of Life by John Kuriyan, Boyana Konforti, and David Wemmer © Garland Science
PROBLEMS and solutions 3

constant between the two experiments. Therefore, the 24. How much resistive force does a 50-nm vesicle
steeper slope is the larger M, which is from the dimeric experience if it is transported by dynein at 1 µm•sec–1
protein. The wild-type kinase is the red line, while the in the cytoplasm (η = 0.2 g•cm–1•sec–1)?
mutant kinase is the blue line.
Answer:
b. Both lines are roughly linear suggesting that only
f = 6πηrv
one molecular weight species is populated at that
= 6 × 3.14 × 0.2 g•cm–1•sec–1 × 5 × 10–6 cm ×
concentration. Thus neither the wild-type nor the
1 × 10–4 cm•sec–1
mutant kinase exchange between monomer and dimer
= 1.9 × 10–9 g•cm•sec–2
to any measureable extent.
= 19 fN
20. Sedimentation coefficients (in Svedberg units) are
often non-additive for macromolecular complexes. For
example, the assembled ribosome and proteosome
each have lower total sedimentation coefficients than
one might expect given the constituents. How might a
macromolecular assembly have a higher sedimentation
coefficient than the sum of its subunits?
Answer:
The sedimentation coefficient is related to shape. If the
constituent macromolecules were mostly spherical,
but the assembled complex was highly nonspherical,
the sedimentation coefficient would be non-additive
and increase above expectation. For example the
assembly of mostly spherical proteins into a filament-
like structure would increase the relative sedimentation
coefficient.

21. Why is it necessary to add agarose or polyacrylamide


for electrophoresis experiments?
Answer:
Agarose and polyacrylamide form a gel that prevents
convection and increases the friction factor,
allowing for a more robust measurement of relative
electrophoretic mobility.

22. Why do proteins, but not nucleic acids, need to


be covered in SDS to estimate the mass by gel
electrophoresis?
Answer:
Proteins have heterogeneous charge and therefore
would not migrate strictly in relation to mass. The
number of SDS molecules associated with the protein
is roughly proportional to the number of residues,
ensuring an even charge:mass ratio. In contrast, nucleic
acids are dominated by the charge of the phosphate
backbone and have a relatively constant charge:mass
ratio independent of sequence.

23. Dynein is a cytoplasmic motor similar to kinesin, but it


travels along microtubules in the opposite direction. A
single dynein transports a vesicle 0.6 µm along an axon
in 5 sec. Dynein steps use one cycle of ATP hydrolysis
that move it 80 Å along a microtubule filament.
Assuming all steps are forward along one filament,
what is the ATP hydrolysis rate of dynein?
Answer:
Steps = 0.6 × 10–6 m / 80 × 10–10 m = 75 steps or
75 ATPs hydrolyzed.
ATP hydrolysis rate = 75/5 sec = 15 ATPs per second
per dyenin.

The Molecules of Life by John Kuriyan, Boyana Konforti, and David Wemmer © Garland Science
The MOLECULES
of LIFE
Physical and Chemical Principles

Solutions Manual

Prepared by James Fraser and Samuel Leachman


Chapter 18

Folding

Problems

True/False and Multiple Choice

1. Unlike protein folding, RNA folding is generally d. They recognize exposed hydrophobic segments.
hierarchical. e. They can bind and release proteins several times
True/False during a folding cycle.
7. Amyloid fibrils are formed by zipper-like hydrogen bonds
2. Which of the following effects favors RNA folding? within and across β sheets.
a. Interactions between phosphate groups. True/False
b. Kinetic traps of alternative structures.
c. Neutralization of backbone charge by Fill in the Blank
counterions.
d. The entropy of the native state compared to the 8. Counterions for nucleic acids carry a ___________
unfolded state. charge.
Answer: positive
3. Which of the following is least likely to lead to RNA
unfolding? 9. In hydroxyl radical footprinting, regions with ________
a. Increasing [Mg2+] by 1 mM. solvent accessibility are likely to be cleaved.
b. Increasing the temperature to 75°C. Answer: high
c. Adding 1 M EDTA.
d. Adding 6 M urea. 10. When local secondary structure elements form quickly,
protein folding is said to follow a ______________
4. Which of the following statements is true at Tm for a mechanism. In contrast, when distant contacts are
monomeric protein or RNA? quickly formed without secondary structure formation,
a. ΔGfolding equals 0. folding is said to follow a _____________ mechanism.
b. Equal concentrations of folded and unfolded Answer: diffusion collision, nucleation condensation
protein are present.
c. Kfolding equals 1. 11. A mutation that has similar interactions in the folding
d. The unfolding curve is at its midpoint. transition state as in the folded state has a __________
e. All of the above. of 1.
Answer: ϕ-value
5. A protein with two-state folding populates two
intermediates as folding occurs. 12. Folding can be visualized as movement on a
True/False multidimensional surface known as a _______________.

6. Which of the following statements about molecular Answer: free-energy landscape


chaperones is not true?
a. They accelerate the folding of client proteins. Quantitative/Essay
b. They prevent protein aggregation. (Assume T = 300 K and RT = 2.5 kJ•mol–1 for all questions.)
c. They are required for in vitro, but not in vivo,
folding. 13. The concept of hierarchy is important for understanding

The Molecules of Life by John Kuriyan, Boyana Konforti, and David Wemmer © Garland Science
2 Chapter 18: Folding

the differences between RNA and protein folding. be caught in this misfolding trap, even though the free
How do the two processes differ in structural stability energy of the folded state is lower.
hierarchy and folding kinetics hierarchy?
15. Below is a hydroxyl radical footprint, similar to data
Answer: shown in Figure 18.51 for a 12-nucleotide segment of
Although proteins and RNA have hierarchical structure, RNA.
protein secondary structure elements are not generally
a. Which region becomes protected first?
stable in isolation. These elements are only stable when
b. Which
Figure region becomes more solvent accessible
18.Q14
incorporated into the tertiary structure of a folded
during the folding reaction?
protein. In contrast, RNA secondary structure elements
are generally stable in solution both in isolation and as
time
part of the folded structure.
Protein folding is generally non-hierarchical as
formation of secondary and tertiary structure can occur

nucleotides
simultaneously. RNA folding is kinetically hierarchical
5
in that secondary structure elements tend to form
in a distinct phase (or in distinct phases) prior to the
formation of tertiary contacts.
10
14. Below are reaction coordinate diagrams for two
RNA molecules (A and B). For each molecule of RNA, 12
we present a reaction coordinate for a misfolded
conformation (left) and a correctly folded conformation Answer:
(right). Which of the RNA folding reactions is more likely a. Bases 7–9 become protected first, then base 12.
to be caught in a misfolded kinetic trap? Explain your b. Base 4 becomes more solvent accessible during
Figure 18.Q13
reasoning. the reaction.

(A) (B) 16. How do rugged and smooth energy landscapes


differ with regard to the number of local minima,
interconversions between minima, and kinetic traps?
Answer:
free energy, G

free energy, G

Both the rugged and smooth energy landscape may


contain many local minima; however, only in the rugged
landscape do large free-energy barriers separate
unfolded unfolded unfolded unfolded
local minima and slow the rate of interconversion
misfolded misfoldedbetween wells. The slowing of interconversion between
folded wells leads tofolded
falling into kinetic traps in the rugged
landscape, which is not a factor during folding on the
reaction coordinate reaction smooth
coordinate
landscape.

(B) 17. A mutation of core residues in the E. coli protein Rop


speeds up the rate of protein folding from 0.013 sec–1
to 1.3 sec–1.
free energy, G

a. What is the effect of the mutation on the free


energy of the transition state relative to the unfolded
state?
b. Given that a simple mutation can increase the
unfolded unfolded folding rate, what does this result suggest about the
misfolded relationship between the protein folding rate and
folded natural selection? (Data based on M. Munson,
folded
K.S. Anderson, and L. Regan, Structure: Folding and
e reaction coordinate Design, 2: 77–87, 1997.)
Answer:
Answer: a. ΔΔGU→TS = RT ln (kfmut/kfWT) = –2.5 × ln (1.3/0.013)
RNA molecule A is more likely to be misfolded in a = –11.5 kJ•mol–1
kinetic trap. Although the free energies of the folded b. Proteins must satisfy many functional constraints,
states are similar for the two RNA molecules, the so a fast folding rate is not necessarily a trait
barrier height of the folding reaction for A is much experiencing strong selection. It appears that Rop
higher than for B. In addition, the barrier for folding is has experienced stronger selection for other aspects
higher than for misfolding in A. Thus, A will form the of function and that its slow wild-type folding rate is
misfolded state faster than the folded state and can sufficient to support those functions.

The Molecules of Life by John Kuriyan, Boyana Konforti, and David Wemmer © Garland Science
PROBLEMS and solutions 3

18. Shown below are two equally populated alternative slower for the mutant than for the wild type, what is the
conformations of a simplified protein with three unfolding rate of F4S?
β strands.
Figure Which conformation is likely to fold and
18.Q17 Answer:
unfold more rapidly?
ϕF4S = ΔΔGU→RTS/(ΔΔGU→RTS – ΔΔGF→RTS)
(A) –ΔΔGF→RTS = ΔΔGU→RTS/ϕF4S – ΔΔGU→RTS
RT ln (kumut/kuWT) = (–RT ln (kfmut/kfWT))/0.21 + RT ln
(kfmut/kfWT))
2.5 × ln (kumut/0.5) = (–2.5 × ln (0.5))/0.21 + 2.5 × ln (0.5)
= 6.52
kumut/0.5 = 13.57
(B) kumut = 6.78 sec–1

21. Why is the rate of protein folding limited to


106 sec–1 , even in the absence of any specific free-
energy barriers?
Answer:
Answer:
Even in the absence of any kinetic barriers, the speed of
Conformation A has a lower contact order as the
protein folding is limited by the rate of collisions causing
central strand is contiguous in sequence. A lower
small rotations around bonds that change the relative
contact order is highly correlated with faster protein
positions of different segments of the protein. These
folding. Therefore, it is likely that conformation A folds
collisions are diffusive in character, so the speed limit of
and unfolds more rapidly.
protein folding is set by diffusion.
19. The following data were obtained from a mutational
22. Shown below is a plot of relative fluorescence
analysis on the folding kinetics of the Fyn SH3 domain:
(reporting on fraction folded) versus time for a protein in
the absence (solid line) and presence (dashed line) of a
kf (sec–1) ku (sec–1) molecular chaperone.
wild type 30.2 0.5 a. How many folding intermediates occur when the
protein folds without assistance?
I28A 1.24 1.8 Figure 18.Q21
b. What is the effect of the molecular chaperone?
V55A 29.1 18.1
1
Use these data to answer the following questions:
a. What are the ϕ values of the two mutants?
fraction folded

b. Which residue forms more native interactions at


the transition state? (Data from J.G. Northey, A.A. Di
Nardo, and A.R. Davidson, Nat. Struct. Biol. 9: 389–402,
2002.)
Answer:
a. ϕI28A = ΔΔGU→RTS/(ΔΔGU→RTS – ΔΔGF→RTS)
= (–RT ln (kfmut/kfWT))/(–RT ln (kfmut/kfWT) + RT ln (kumut/
0
kuWT)) time
= (–2.5 × ln (1.24/30.2))/(–2.5 × ln (1.24/30.2) + 2.5 × ln
(1.8/0.5)) Answer:
= 0.71 a. The native protein has two well-populated folding
ϕV55A = ΔΔGU→RTS/(ΔΔGU→RTS – ΔΔGF→RTS) intermediates as indicated by the plateaus on the
= (–RT ln (kfmut/kfWT))/(–RT ln (kfmut/kfWT) + RT ln (kumut/ graph.
kuWT)) b. The effect of the chaperone is to accelerate (or
= (–2.5 × ln (29.1/30.2))/(–2.5 × ln (29.1/30.2) + 2.5 × ln bypass) the folding through the first intermediate;
(18.1/0.5)) however, it appears to have no effect on the second
= 0.01 intermediate.
b. A ϕ-value close to 1 suggests that native contacts
have formed in the transition state, whereas a value 23. How might ATP binding and hydrolysis contribute to the
close to 0 suggests that mostly unfolded (non-native) active and passive mechanisms of folding mediated by
contacts are formed. Since I28 has a higher ϕ-value, it GroEL-GroES?
likely has formed more native interactions than V55.
Answer:
20. The ϕ-value analysis of the previous problem is In an actively folding mechanism, GroEL-GroES might
continued for an additional mutant, F4S. This mutation energetically couple client protein unfolding to ATP
has a ϕ value of 0.21. If the folding rate is two times hydrolysis prior to protein folding in the “Anfinsen cage”.

The Molecules of Life by John Kuriyan, Boyana Konforti, and David Wemmer © Garland Science
4 Chapter 18: Folding

In contrast, in a passive mechanism, the nucleotide


state of GroEL–GroES triggers conformational changes
that gate client protein entry to, and exit from, the
“Anfinsen cage,” where protein folding can occur.

24. A solution x-ray scattering experiment for a protein


is performed in increasing amounts of urea. The data
show a difference in the radius of gyration at 6 M urea
and 0.1 M urea.
a. Why is the radius of gyration different between an
unfolded and folded protein even though both contain
the same number of residues?
b. How will the measured radius of gyration depend
on the concentration of urea?
Answer:
a. Radius of gyration (Rg) is related to the distance
of an average atom from the center of mass of the
protein. Because folded proteins are compact and well
packed, they tend to have a significantly smaller Rg
than unfolded proteins, which populate an ensemble of
random coil conformations.
b. Since urea is a denaturant, increasing
concentrations should result in unfolding of the
protein. The measured Rg will then tend to increase
monotonically with the concentration of urea until the
protein is completely unfolded, such that the Rg at 6 M
urea is significantly higher than at 0.1 M urea.

The Molecules of Life by John Kuriyan, Boyana Konforti, and David Wemmer © Garland Science
The MOLECULES
of LIFE
Physical and Chemical Principles

Solutions Manual

Prepared by James Fraser and Samuel Leachman


Chapter 19
Fidelity in DNA and
Protein Synthesis

Problems

True/False and Multiple Choice

1. DNA polymerases from both prokaryotes and d. The large subunit dissociates from the small
eukaryotes resemble a right hand made up of fingers, subunit.
palm, and thumb subdomains. e. EF-Tu dissociates from the ribosome.
True/False
7. The large ribsomal subunit is comprised of proteins and
2. Which of the following is not a factor that favors DNA RNA, whereas the small subunit is comprised only of
synthesis? RNA.
a. The release of pyrophosphate. True/False
b. The base-stacking interactions.
c. The change of entropy when the free dNTP is Fill in the Blank
added to the polynucleotide chain.
d. The formation of hydrogen bonds. 8. Mismatches can be removed by a 3ʹ to 5ʹ _________
e. The hydrolysis of pyrophosphate that is released. enzymatic activity.

3. The 5ʹ base in the codon is less important for codon– Answer: exonuclease
anticodon recognition and is therefore known as the
9. Conserved ______ residues bind divalent metal ions in
“wobble base.”
the palm domain.
True/False
Answer: acidic/negatively charged
4. The error rate of DNA replication in an E. coli strain that
lacked the mismatch repair system would increase by 10. DNA polymerase can add a nucleotide to the free ____
what amount? hydroxyl group of DNA or RNA.
a. ~109 Answer: 3ʹ
b. ~107–108
c. ~106 11. The “________” of the “finger-palm-thumb” subdomain
d. ~102 structure for DNA polymerase I inserts into the minor
e. ~104–105 groove of DNA.
Answer: thumb
5. Mismatched base pairs are symmetric about the
glycosyl bonds. 12. The movement of mRNA, as the tRNA shifts between
True/False sites on the ribosome, is called ______________.

6. Which of the following statements is not an effect of Answer: translocation


codon–anticodon recognition in the ribosome?
a. The 30S subunit changes from an open to closed Quantitative/Essay
conformation.
b. The aminoacyl-tRNA structure is distorted. (Assume T = 300 K and RT = 2.5 kJ•mol–1 for all questions
c. EF-Tu hydrolyzes GTP. unless otherwise stated.)

The Molecules of Life by John Kuriyan, Boyana Konforti, and David Wemmer © Garland Science
2 Chapter 19: Fidelity in DNA and Protein Synthesis

13. A scientist compares a bacterial polymerase (without 18. a. Why are fluorinated bases readily incorporated
a proofreading domain) with a yeast polymerase (also during DNA synthesis?
without a proofreading domain). The bacterial enzyme b. Draw a fluorinated thymidine analog-adenine
has lower fidelity. Surprisingly, the two enzymes have base pair and label the C1ʹ–C1ʹ distance and angles
approximately the same value of KM for correctly between the C1ʹ–C1ʹ vector and the glycosyl bonds with
matched base pairs. Explain which enzyme, the predicted values.
bacterial or yeast, would likely have a higher KM for
mismatched base pairs? Answer:
a. Shape complementarity and base-stacking
Answer: interactions are more important than the hydrogen
The yeast enzyme makes fewer errors than the bonds in determining which bases are incorporated.
bacterial enzyme. The similar KM for correct matches Since the fluorination only alters the hydrogen bonding
suggests that this difference is due to a difference and not the steric interactions, analogs are readily
solely in the affinity for incorrect matches. Since the incorporated.
bacterial polymerase makes more errors it likely has a b. The illustration is similar to Figure 19.33A with
greater affinity for incorrect base pairs. Thus, the yeast Q19.18b
base from Figure 19.34. The C1ʹ–C1ʹ distance should be
enzyme likely has the higher KM value (indicating a ~11 Å, eta angles should be ~50°.
lower affinity for incorrect bases).
14. How would inactivating a mutation of pyrophosphatase dFTP
affect the rate of DNA synthesis in the cell? H A
H 3C F H N N
Answer:
Without pyrophosphatase, the rate of DNA synthesis N N
would likely decrease. This is because pyrophosphatase N C1′
breaks down pyrophosphate near the active site that C1′ F H
could otherwise drive a reverse synthesis reaction 51º
50º
where a base would be removed by addition of
11.1 Å
pyrophosphate to the chain.

15. What significant free-energy barriers affect the rate of 19. How does the interaction between the O helix and the
DNA synthesis? incoming nucleotide change between the “open” and
Answer: “closed” forms of DNA polymerase?
The largest barrier in polymerase-catalyzed synthesis
Answer:
has been shown to be the slow conformational
In the open form, conserved residues in the O helix
change that closes the polymerase after binding
help to bind the incoming nucleotide. When the
of the dNTP to be incorporated. Other smaller but
polymerase closes, the O helix packs against the face of
significant barriers include the conformational change
the new base pair, helping to seal the new nucleotide
prior to pyrophosphate release and the release of
completely into the active site of the polymerase.
pyrophosphate itself.

16. In the “two-metal” reaction mechanism for nucleic acid 20. A scientist probes the thermodynamics of forming
phosphoryl transfer, what roles are played by a) only an RNA hairpin (H) from the unfolded state (R). She
metal ion A, b) only metal ion B, and c) both metal ions? measures ∆ SoH→R = 908 J•K–1•mol–1 and ∆ HoH→R =
295 kJ•mol–1. What are the values of a) the equilibrium
Answer: constant KH→R and b) ∆GoH→R?
a. Metal ion A activates the 3ʹ sugar hydroxyl group
by facilitating deprotonation. Answer:
b. Metal ion B promotes the bond breakage step by a. –RT lnKH→RR = ΔHH→RR – TΔSH→RR
directly ligating the pyrophosphate leaving group and = 295 kJ•mol–1 – 300 K × 0.908 kJ•K–1•mol–1
neutralizing its developing negative charge. = 22.6 kJ•mol–1
c. Both metals orient the phosphate group in the lnKH→RU = –9.0
polymerase active site and stabilize the negative KH→RU = 1.2 × 10–4
charge in the transition state by ligating the nonbridging b. ΔG = ΔH – TΔS
oxygen of the phosphate being transferred. = 295 kJ•mol–1 – 300 K × 0.908 kJ•K–1•mol–1
= 22.6 kJ•mol–1
17. Why does EDTA quench DNA polymerase reactions?
21. The Tm of a DNA duplex is measured to be 55°C at
Answer:
10 μm. When the concentration is 1000 times higher,
EDTA is a metal-chelating molecule that has higher
the Tm changes to 60°C. What are the ΔS and ΔH values
affinity for Mg2+ than DNA polymerase does. EDTA
for formation of this duplex?
outcompetes DNA polymerase for Mg2+ and thus
binds all the Mg2+ in the reaction buffer. Since Mg2+ Answer:
is required by DNA polymerase for catalysis, DNA ΔHH→RR = –R ln (C1/C2)/(1/T1 – 1/T2) =
synthesis cannot occur. –8.314 J•K–1•mol–1 × ln(0.001)/(1/328 – 1/333) =

The Molecules of Life by John Kuriyan, Boyana Konforti, and David Wemmer © Garland Science
PROBLEMS and solutions 3

1250 kJ•mol–1
Since ΔG = –RT ln(KD) = –RT ln(Ctot/4), at 55°C,
–RT ln(10 μM/4) = ΔHH→RR – TΔSH→RR; ΔSH→RR =
ΔHH→RR/T + R ln(10 μM/4) = 3.72 kJ•K–1•mol–1

22. Measurements on several variants of a DNA double


helix are performed. Although the free energy of double
helix formation does not vary significantly, the enthalpy
and entropy measurements show a large range. A
scientist observes that mutations that are favorable
with respect to binding enthalpy are entropically
disfavored. What effect has the scientist observed
and why is this phenomenon common in biological
molecules?
Answer:
The scientist is observing enthalpy–entropy
compensation. Many biological molecules, including
DNA, are highly flexible when not bound to other
molecules. As tighter, more stable interactions are
formed between two binding partners, their breathing
motions decrease. This reduction in the degrees of
freedom is unfavorable entropically. These offsetting
enthalpic and entropic effects lead to small variations in
the free energy of binding.

23. A DNA polymerase is isolated and found to have an


error rate of 1 in 106.
a. Suppose that the error rate is determined solely
by the relative stabilities of incorrect and correct base
pairs. What would the difference in free energy between
correct and incorrect nucleotides incorporated by the
polymerase have to be in order to explain the error
rate? 

b. Solution studies of isolated oligonucleotides
indicate that the energetic difference is actually
–1.2 kJ•mol–1. What is the equilibrium constant of
correct–incorrect base pair discrimination based on
these solution studies?
c. What other enzymatic activity, in addition to
nucleotide insertion, contributes to the increased
fidelity of DNA polymerase?
Answer:
a. ΔG = –RT ln (Kex) = –2.5 × ln (10–6) = –36 kJ•mol–1
b. Kex = e–ΔG/RT = e1.2/2.5 = 1.62
c. The exonuclease activity contributes to the
observed fidelity by excising incorrect bases, leading to
a higher observed fidelity.

24. Both thermodynamics and kinetics play an important


role in achieving fidelity. How is kinetic control
exercised by a) DNA polymerase and b) the ribosome?
Answer:
In DNA polymerase, the rate of extension after an error
is 200–2500 times slower, which gives the exonuclease
domain a chance to act on mismatched nucleotides.
In the ribosome, kinetic proofreading occurs through
the combination of two steps (EF-Tu GTP hydrolysis
and tRNA dissociation) that change the rate of correct
codon–anticodon recognition without altering the
equilibrium constants.

The Molecules of Life by John Kuriyan, Boyana Konforti, and David Wemmer © Garland Science

You might also like